Você está na página 1de 125

PROPERTY

PROPERTY
CASE DIGESTS

SUBMITTED BY:
GUSTAVO F. DALEN

PROPERTY

LLB 2DTABLE OF CONTENTS


TITLE

PAGE

1. LADERA V. HODGES VOL.48 NO. 12 O.G 5374--------------------------------------------------------------------- 1


2. MINDANAO BUS CO. V. CITY ASSESSOR AND TREASURER 6 SCRA 197------------------------------------ 1
3. MAKATI LEASING & FINANCE CORP. V. WEAREVER TEXTILES 122 SCRA 296 ---------------------------- 2
4. SANTOS EVANGELISTA V. ALTO SURETY & INSURANCE CO. INC. 103 PHIL. 401 ------------------------ 2
5. TSAI V. CA 366 SCRA 324 ------------------------------------------------------------------------------------------------ 3
6. SERGS PRODUCTS INC. V. PCI LEASING AND FINANCE, INC. 338 SCRA 499 ---------------------------- 3
7. BURGOS, SR. V. CHIEF OF STAFF , AFP 133 SCRA 800 ---------------------------------------------------------- 4
8. LOPEZ V OROSA, J.R AND PLAZA THEATER, INC. 103 PHIL. 98 ----------------------------------------------- 5
9. YAP V. TANADA 163 SCRA 464 ------------------------------------------------------------------------------------------ 5
10. MACHINERY &ENGINEERING SUPPLIES , INC. V. CA 96 PHIL 70 ---------------------------------------------- 6
11. FELS ENERGY INC. V PROVINCE OF BATNGAS G.R NO. 168557 ---------------------------------------------- 6
12. CHINA BANK CORPORATION V CA G.R 129644 -------------------------------------------------------------------- 7
13. LAUREL V. GARCIA 187 SCRA 797 ------------------------------------------------------------------------------------- 7
14. RABUCO V. VILLEGAS 55 SCRA 658 ----------------------------------------------------------------------------------- 8
15. MACASIANO V. DIOKNO 212 SCRA 464 ------------------------------------------------------------------------------ 9
16. REPUBLIC V. CA 281 SCRA 639 ----------------------------------------------------------------------------------------- 9
17. PROVINCE OF ZAMBOANGA DEL NORTE V. CITY OF ZAMBOANGA 22 SCRA 1334 -------------------- 10
18. CHAVES V. PUBLIO ESTATES 384 SCRA 152 ---------------------------------------------------------------------- 11
19. VILLARICO V. SARMIENTO 442 SCRA 110 ------------------------------------------------------------------------- 11
20. REPUBLIC V. T.A.N PROPERTIES, INC. G.R NO. 154953 -------------------------------------------------------- 12
21. MANILA INTERNATIONAL AIRPORT AUTHORITY V. A G.R NO 155650 -------------------------------------12
Q12
22. JAVIER V. VERDIANO II 237 SCRA 565 ------------------------------------------------------------------------------ 13
23. BUSTOS V. CA 350 SCRA 155 ----------------------------------------------------------------------------------------- 14
24. HEIRS OF ROMAN SORIANO V. CA 363 SCRA 87 ---------------------------------------------------------------- 15
25. GARCIA V. CA 312 SCRA 180 -------------------------------------------------------------------------------------------16
26. RODIL ENTERPRISE , INC. V. CA 371 SCRA 79 ------------------------------------------------------------------- 17
27. ISAGUIRRES V. DE LARA 332 SCRA 803 --------------------------------------------------------------------------- 19
28. CUSTODIO V. CA 253 SCRA 483 -------------------------------------------------------------------------------------- 19
29. ABEJARON V. NABASA 359 SCRA 47 -------------------------------------------------------------------------------- 20
30. GERMAN MANAGEMENT &SERVICES , INC. V. CA 177 SCRA 495 ------------------------------------------ 21
31. CAISIP V. PEOPLE 36 SCRA 17 ---------------------------------------------------------------------------------------- 21
32. PEOPLE V. PLETCHA, JR. 22 CA REP. 809 --------------------------------------------------------------------------- 22
33. ANDAMO V. INTERMEDIATE APPELLATE COURT 191 SCRA 195 --------------------------------------------- 23
34. BACHRACH MOTOR CO. V TALISAY-SILAY MILING CO. 56 PHIL. 117 --------------------------------------- 24
35. EQUATORIAL REALTY DEVELOPMENT, INC. V MAYFAIR THEATER 370 SCRA 56 ----------------------- 24
36. SPOUSES MACASAET V. SPOUSES MACASAET G.R NO 154391-92 ---------------------------------------- 25
37. FELICIANO V. ZALDIVAR G.R NO. 162593 --------------------------------------------------------------------------26
38. SPOUSES NUGUID V. CA G.R NO 151815 -------------------------------------------------------------------------- 29
39. IGNACIO V. HILARIO 76 PHIL. 605 ----------------------------------------------------------------------------------- 30
40. FILIPINAS COLLEGE INC. V. GARCIA TIMBANG ET AL. 164 SCRA 287 ------------------------------------- 31
41. MANOTOK REALTY INC. V. TECSON 164 SCRA 587 ------------------------------------------------------------- 32

PROPERTY
42. BERNARDO V. BATACLAN 66 PHIL. 598 ---------------------------------------------------------------------------- 33
43. HEIRS OF RAMON DURANO, SR. V. UY 344 SCRA 328 --------------------------------------------------------- 33
44. BALLATAN V. CA 304 SCRA 34 ----------------------------------------------------------------------------------------35
45. SPOUSES DEL CAMPO V. ABIESA 160 SCRA 379 ---------------------------------------------------------------- 36
46. PACIFIC FORMS, INC. V.ESGUERRA 30 SCRA 684 --------------------------------------------------------------- 36
47. PECSON V. CA 244 SCRA 407 ------------------------------------------------------------------------------------------37
48. VDA. MERCY DE ROXAS V. OUR LADYS FOUNDATION INC. G.R NO. 182378 --------------------------- 38
49. TECHNOGAS PHILIPPINES V. CA 268 SCRA 5 --------------------------------------------------------------------- 38
50. PLEASANTVILLE DEVT CORP. V. CA 253 SCRA 10 -------------------------------------------------------------- 39
51. GEMINIANO V. CA 259 SCRA 344 ------------------------------------------------------------------------------------ 40
52. ROSALES V. CASTELLTORT G.R NO. 157044 ---------------------------------------------------------------------- 41
53. AGUSTIN V. INTERMEDIATE APPELLATE COURT 187 SCRA 218 --------------------------------------------- 42
54. CUREG V. INTERMEDIATE APPELLATE COURT 177 SCRA 313 ----------------------------------------------- 43
55. VIAJAR V. CA 168 SCRA 405 ------------------------------------------------------------------------------------------- 43
56. VDA. DE NAZARENO VS, INTERMEDIATE APPELLATE COURT 257 SCRA 589 --------------------------- 44
57. HEIRS OF NAVARRO V. INTERMEDIATE APPELLATE COURT 268 SCRA 74 -------------------------------- 45
58. AVILA V. SPOUSES BARABAT G.R NO 141993 -------------------------------------------------------------------- 46
59. DEL BANCO V. INTERMEDIATE APPELLATE COURT 156 SCRA 55 ------------------------------------------- 46
60. ADLAWAN V. ADLAWAN G.R NO. 161916 -------------------------------------------------------------------------- 48
61. OESMER V. PARAISO DEVELOPMENT CORP. G.R NO 157493 ------------------------------------------------ 49
62. AGUIRRE V. CA GR. NO 122249 -------------------------------------------------------------------------------------- 49
63. BALO V. CA G.R NO 129704 ------------------------------------------------------------------------------------------- 50
64. GALVEZ V. CA G.R NO 114046 ---------------------------------------------------------------------------------------- 51
65. BALOLOY V. HUAR G.R NO 157767 ---------------------------------------------------------------------------------- 51
66. PARDELL V. BARTOLOME 23 PHIL 450 ------------------------------------------------------------------------------ 52
67. CARO V. CA 113 SCRA 10 ---------------------------------------------------------------------------------------------- 53
68. BAILON-CASILAO V. CA 113 SCRA 10 ------------------------------------------------------------------------------- 54
69. ROQUE V. INTERMEDIATE APPELLATE COURT 165 SCRA 118 ----------------------------------------------- 55
70. DELIMA V. CA 201 SCRA 641 ------------------------------------------------------------------------------------------ 56
71. AGUILAR V. CA 227 SCRA 472 ---------------------------------------------------------------------------------------- 56
72. TOMAS CLAUDIO MEMORIAL COLLEGE INC. V. CA 316 SCRA 502 ----------------------------------------- 58
73. ROBLES V. CA 328 SCRA 97 ------------------------------------------------------------------------------------------- 58
74. RIZAL CEMENT CO., INC. V. VILLAREAL 135 SCRA 15 --------------------------------------------------------- 59
75. WONG V. CARPIO 203 SCRA 118 ------------------------------------------------------------------------------------- 59
76. SOMODIO V. CA 235 SCRA 307 --------------------------------------------------------------------------------------- 60
77. MAGLUCOT- AW V. MAGLUCOT 329 SCRA 78 --------------------------------------------------------------------- 61
78. CEQUENA V. BOLANTE 330 SCRA 216 ------------------------------------------------------------------------------ 61
79. ARAGON V. INSULAR GOVERNMENT 19 PHIL 223 --------------------------------------------------------------- 62
80. CATHOLIC VICAR APOSTOLIC OF THE MOUNTAIN PROVINCE V. CA 183 SCRA 639 ------------------- 63
81. EDCA PUBLISHING & DISTRIBUTING CORP. V. SANTOS 134 SCRA 614 ----------------------------------- 63
82. DE GARCIA V. CA 31 SCRA 129 ----------------------------------------------------------------------------------------64
83. DIZON V. SUNTAY 47 SCRA 160 -------------------------------------------------------------------------------------- 65
84. LEDESMA V. CA 213 SCRA 195 --------------------------------------------------------------------------------------- 65
85. AZARCON AND ABOBO V. EUSEBIO 105 SCRA 569 ------------------------------------------------------------- 66
86. CORDERO V. CABRAL 123 SCRA 532 ------------------------------------------------------------------------------- 66

PROPERTY
87. SOLID HOMES ,INC. V. SPOUSES TAN, G.R NO 145156-57 --------------------------------------------------- 67
88. MENDOZA AND ENRIQUEZ V. DE GUZMAN 52 PHIL 164 ------------------------------------------------------ 68
89. ROBLES AND MARTIN V. LIZZARAGA HERMANOS. ETC 42 PHIL, 584 ------------------------------------- 69
90. METROPOLITAN WATERWORKS AND SEWAGE SYSTEM V. CA 143 SCRA 623 -------------------------- 70
91. SHIPSIDE INCORPORATED V. CA G.R NO 143377 --------------------------------------------------------------- 71
92. BACHRACH V. SEIFERT AND ELIANOFF 87 PHIL 483 ----------------------------------------------------------- 72
93. HEMEDES V. CA 316 SCRA 347 --------------------------------------------------------------------------------------- 73
94. FABIE V. GUITERREZ DAVID 75 PHIL 536 -------------------------------------------------------------------------- 73
95. FELIX GOCHAN AND SONS REALTY V. HEIRS OF RAYMUNDO BABA GR NO. 138945 ----------------- 74
96. VDA. DE ARANAS V. ARANAS 150 SCRA 415 ----------------------------------------------------------------------75
97. LOCSIN V. VALENZUELA 173 SCRA 454 ---------------------------------------------------------------------------- 75
98. VALISNO V. ADRIANO 161 SCRA 398 ------------------------------------------------------------------------------- 76
99. DE CASTRO V. CA G.R NO. 115838 ---------------------------------------------------------------------------------- 77
100.RONQUILLO, ET. AL. V. ROCO ET. AL 103 PHIL 84 ------------------------------------------------------------- 78
101.TANEDO V. BERNAD 165 SCRA 86 ---------------------------------------------------------------------------------- 78
102.COSTABELLA CORPORATION V. CA 193 SCRA 333 ------------------------------------------------------------ 79
103.ENCARNACION V. CA 195SCRA 74 --------------------------------------------------------------------------------- 80
104.CASE V. HEIRS OF TUASON 14 PHIL 521 ------------------------------------------------------------------------- 81
105.CHOCO V. SANTAMARIA 21 PHIL132 ------------------------------------------------------------------------------- 81
106.SECRETARY OF EDUCATION V. HEIRS OF RUFINO DULAY G.R NO.164748 ----------------------------- 82
107.SOLID MANILA CORP V. BIO HONG TRADING CO. INC. 195 SCRA 748 -----------------------------------83
108.FLORO V. LLENDO 244SCRA713 ------------------------------------------------------------------------------------ 84
109.QUIMEN V. CA 257 SCRA 163 ---------------------------------------------------------------------------------------- 85
110.GESTOPA V. CA 342 SCRA 105 -------------------------------------------------------------------------------------- 86
111.DE JESUS, ET. AL. V. HOWMART CORP.,ET.AL. 12 CA REP. 831---------------------------------------------- 87
112.LA VISITA ASSOCIATION, INC. V CA 278 SCRA 498 ------------------------------------------------------------ 88
113.ALCANTRA V. RETA, JR. 372 SCRA 364 ---------------------------------------------------------------------------- 89
114.REPUBLIC V. SILIM 356 SCRA 1 -------------------------------------------------------------------------------------- 90
115.PROSPERITY CREDIT RESOURCES, INC. V. CA 301 SCRA 52 ----------------------------------------------- 91
116.VILLANUEVA V. VELASCO 346 SCRA 99 --------------------------------------------------------------------------- 91
117.NATIONAL IRRIGATION ADMINISTRATION V. CA 440 SCRA 661 ------------------------------------------- 92
118.IMPERIAL V. CA 316 SCRA393 --------------------------------------------------------------------------------------- 93
119.HIDALGO ENTERPRISE, INC V. BALANDAN 91 PHIL ----------------------------------------------------------- 94
120.ACAP V. CA 251 SCRA 30 ---------------------------------------------------------------------------------------------- 94
121.DE LUNA V. ABRIGO 181 SRCA 150 -------------------------------------------------------------------------------- 95
122.GONZALES V. CA 358SCRA 393 ------------------------------------------------------------------------------------- 96
123.REYES V. MOSQUEDA 187 SCRA 661 ------------------------------------------------------------------------------ 97
124.LIGUEZ V. CA 102 PHIL. 577 ----------------------------------------------------------------------------------------- 98
125.PERSHING TAN QUETO V. CA 148 SCRA 54 ---------------------------------------------------------------------- 98
126.PIJARILLO V. INTERMEDIATE APPELLATE COURT 176 SCRA 340 ------------------------------------------- 99
127.CRUZ V. CA 140 SCRA 245 ------------------------------------------------------------------------------------------ 100
128.ROMAN CATHOLIC ARCHBISHOP OF MANILA V. CA 198 SCRA 300 ------------------------------------- 101
129.EDUARTE V. CA 253 SCRA 391 ------------------------------------------------------------------------------------ 101
130.QUILALA V. ALCANTARA 371 SCRA 311 ------------------------------------------------------------------------- 102
131.HEMEDES V. CA 316 SCRA 347 ------------------------------------------------------------------------------------ 103

PROPERTY
132.SIGUAN V. LIM 318 SCRA 725 -------------------------------------------------------------------------------------- 104
133.NOCEDA V. CA 316 SCRA 504 -------------------------------------------------------------------------------------- 105
134.HEIRS OF CESARIO VELASQUEZ V. CA 325SCRA 552 ------------------------------------------------------ 105

1
Ladera v. Hodge
G.R. No. 8027-R, September 23, 1952, Vol. 48, No. 12, Official Gazette
5374
Reyes, J.B.L., J.
FACTS: Paz G. Ladera entered into a contract with C.N. Hodges. Hodges promised
to sell a lot with an area of 278 square meters to Ladera, subject to certain terms
and conditions. The agreement called for a down payment of P 800.00 and monthly
installments of P 5.00 each with interest of 1% per month, until P 2,085 is paid in
full. In case of failure of the purchaser to make any monthly payment within 60
days after it fell due, the contract may be considered as rescinded or annulled.
Ladera built a house on the lot. Later on, she defaulted in the payment of the
agreed monthly installment. Hodges filed an action for the ejectment of Ladera.
The court issued an alias writ of execution and pursuant thereto, the city sherif
levied upon all rights, interests, and participation over the house of Ladera. At the
auction sale, Laderas house was sold to Avelino A. Magno. Manuel P. Villa, later on,
purchased the house from Magno.
Ladera filed an action against Hodges and the judgment sale purchasers. Judgment
was rendered in favor of Ladera, setting aside the sale for non-compliance with
Rule 39, Rules of Court regarding judicial sales of real property. On appeal, Hodges
contends that the house, being built on a lot owned by another, should be
regarded as movable or personal property.
ISSUE: Whether or not Laderas house is an immovable property.
HELD: YES. The old Civil Code numerates among the things declared by it as
immovable property the following: lands, buildings, roads and constructions of all
kind adhered to the soil. The law does not make any distinction whether or not the
owner of the lot is the one who built. Also, since the principles of accession regard
buildings and constructions as mere accessories to the land on which it is built, it is
logical that said accessories should partake the nature of the principal thing.
Mindanao Bus Company v. The City Assessor and Treasurer
G.R. No. L-17870, September 29, 1962, 6 SCRA 197
Labrador, J.
FACTS: Petitioner Mindanao Bus Company is a public utility solely engaged in
transporting passengers and cargoes by motor trucks, over its authorized lines in
the Island of Mindanao, collecting rates approved by the Public Service
Commission. Respondent sought to assess the following real properties of the
petitioner; (a) Hobart Electric Welder Machine, (b) Storm Boring Machine; (c) Lathe
machine with motor; (d) Black and Decker Grinder; (e) PEMCO Hydraulic Press; (f)
Battery charger (Tungar charge machine) and (g) D-Engine Waukesha-M-Fuel. It
was alleged that these machineries are sitting on cement or wooden platforms,
and that petitioner is the owner of the land where it maintains and operates a
garage for its TPU motor trucks, a repair shop, blacksmith and carpentry shops,
and with these machineries, which are placed therein. Respondent City Assessor of
Cagayan de Oro City assessed at P4, 400 petitioner's above-mentioned equipment.
Petitioner appealed the assessment to the respondent Board of Tax Appeals on the
ground that the same are not realty. Respondents contend that said equipments,
though movable, are immobilized by destination, in accordance with paragraph 5
of Article 415 of the New Civil Code.

2
ISSUE: Whether the equipments in question are immovable or movable properties.
HELD: The equipments in question are movable. So that movable equipments to
be immobilized in contemplation of the law, it must first be "essential and principal
elements" of an industry or works without which such industry or works would be
"unable to function or carry on the industrial purpose for which it was established."
Thus, the Court distinguished those movable which become immobilized by
destination because they are essential and principal elements in the industry from
those which may not be so considered immobilized because they are merely
incidental, not essential and principal.
The tools and equipments in question in this instant case are, by their nature, not
essential and principle municipal elements of petitioner's business of transporting
passengers and cargoes by motor trucks. They are merely incidentalsacquired as
movables and used only for expediency to facilitate and/or improve its service.
Even without such tools and equipments, its business may be carried on, as
petitioner has carried on, without such equipments, before the war. The
transportation business could be carried on without the repair or service shop if its
rolling equipment is repaired or serviced in another shop belonging to another.
Makati Leasing and Finance Corporation v. Wearever Textile Mills, Inc.
G.R. No. L-58469, May 16, 1983, 122 SCRA 29
De Castro, J.
FACTS: To obtain financial accommodations from the Makati Leasing and Finance
Corporation, the Wearever Textile discounted and assigned several receivables
with them under a receivable purchase agreement. To secure the collection of
receivables assigned, Wearever Textile executed a chattel mortgage over certain
raw materials inventory, as well as machinery described as an aero dryer
stentering range. Upon default of Wearever Textile, the Makati Leasing petitioned
for extrajudicial foreclosure of the properties mortgaged to it. When the sherif
failed to enter Wearever Textiles premises to seize the machinery, Makati Leasing
applied for a replevin. Wearever Textile contended that it cannot be a subject of
replevin or a chattel mortgage because it is a real property as it is attached to the
ground by means of bolts and that the only way to remove it is to destroy the
concrete floor.
ISSUE: Whether or not the machinery is real or personal property.
HELD: The machinery is a personal property. The Supreme Court explained that if
a house of strong materials may be considered as personal property for purposes
of executing a chattel mortgage, there is absolutely no reason why a machinery,
which is movable in its nature and becomes immobilized only by destination or
purpose, may not be likewise treated as such.
Santos Evangelista v. Alto Surety and Insurance Co., Inc.
G.R. No. L-11139, April 23, 1958, 103 Phil. 401
Concepcion, J.
FACTS: On June 4, 1949, Santos Evangelista instituted a civil case for a sum of
money. On the same date, he obtained a writ of attachment, which was levied
upon a house, built by Rivera on a land situated in Manila and leased to him. In due
course, judgment was rendered in favor of Evangelista, who bought the house at
public auction held in compliance with the writ of execution issued in said case.
When Evangelista sought to take possession of the house, Rivera refused to

3
surrender it, upon the ground that he had leased the property from the Alto Surety
& Insurance Co., Inc. and that the latter is now the true owner of said property. It
appears that on May 10, 1952, a definite deed of sale of the same house had been
issued to Alto Surety, as the highest bidder at an auction sale held. Hence,
Evangelista instituted an action against Alto Surety and Ricardo Rivera, for the
purpose of establishing his title over said house, and securing possession thereof,
apart from recovering damages. After due trial, the CFI Manila rendered judgment
for Evangelista, sentencing Rivera and Alto Surety to deliver the house in question
to Evangelista and to pay him, jointly and severally, P40.00 a month from October,
1952, until said delivery, plus costs.
ISSUE: Whether or not a house constructed by the lessee of the land on which it is
built, should be dealt with, for purposes of attachment, as immovable property or
as personal property.
HELD: The house is not personal property, much less a debt, credit or other
personal property not capable of manual delivery, but immovable property. As
explicitly held, in Ladera vs. Hodges (48 OG 5374), "a true building (not merely
superimposed on the soil) is immovable or real property, whether it is erected by
the owner of the land or by a usufructuary or lessee. The opinion that the house of
Rivera should have been attached in accordance with subsection (c) of said section
7, as "personal property capable of manual delivery, by taking and safely keeping
in his custody", for it declared that "Evangelista could not have validly purchased
Ricardo Rivera's house from the sherif as the latter was not in possession thereof
at the time he sold it at a public auction is untenable.
Tsai v. Court of Appeals
G.R. No. 120098, October 2, 2001, 366 SCRA 324
Quisumbing, J.
FACTS: On November 26, 1975, respondent Ever Textile Mills, Inc. (EVERTEX)
obtained a three million peso (P3,000,000.00) loan from petitioner Philippine Bank
of Communications (PBCom). As security for the loan, EVERTEX executed in favor
of PBCom, a deed of Real and Chattel Mortgage over the lot where its factory
stands, and the chattels located therein. On April 23, 1979, PBCom granted a
second loan to EVERTEX. The loan was secured by a chattel mortgage over
personal properties enumerated in a list attached thereto. After April 23, 1979, the
date of the execution of the second mortgage mentioned above, EVERTEX
purchased various machines and equipments.
Upon EVERTEX's failure to meet its obligation to PBCom, the latter commenced
extrajudicial foreclosure proceedings against EVERTEX. On December 15, 1982, the
first public auction was held where petitioner PBCom emerged as the highest
bidder and a Certificate of Sale was issued in its favor on the same date. On March
7, 1984, PBCom consolidated its ownership over the lot and all the properties in it.
In November 1986, it leased the entire factory premises to petitioner Ruby L. Tsai.
On May 3, 1988, PBCom sold the factory, lock, stock, and barrel to Tsai, including
the contested machineries.
On March 16, 1989, EVERTEX filed a complaint for annulment of sale,
reconveyance, and damages with the Regional Trial Court against PBCom. EVERTEX
claimed that no rights having been transmitted to PBCom over the assets of
insolvent EVERTEX, therefore Tsai acquired no rights over such assets sold to her,
and should reconvey the assets.

4
ISSUE: Whether or not the inclusion of the questioned properties in the foreclosed
properties is proper.
HELD: Yes. While it is true that the questioned properties appear to be immobile, a
perusal of the contract of Real and Chattel Mortgage executed by the parties gives
a contrary indication. In the case at bar, the true intention of PBCOM and the
owner, EVERTEX, is to treat machinery and equipment as chattels. Assuming that
the properties in question are immovable by nature, nothing detracts the parties
from treating it as chattels to secure an obligation under the principle of estoppel.
It has been held that an immovable may be considered a personal property if there
is a stipulation as when it is used as security in the payment of an obligation where
a chattel mortgage is executed over it, as in the case at bar.
Sergs Products, Inc. v. PCI Leasing and Finance, Inc.
G.R. No. 137705, August 22, 2000, 338 SCRA 499
Panganiban, J.
FACTS: Respondent PCI Leasing and Finance Inc. filed with the RTC of Quezon City
a complaint for sum of money, with an application for a writ of replevin. A writ of
replevin was issued, directing the sherif to seize and deliver the machineries and
equipment to PCI Leasing after five days and upon payment of the necessary
expenses. The sherif proceeded to petitioner's factory and seized one machinery.
Petitioner filed a motion for special protective order invoking the power of the court
to control the conduct of its officers and amend and control its processes, praying
for a directive for the sherif to defer enforcement of the writ of replevin. The
motion was opposed by PCI on the ground that the properties were personal and
therefore still subject to seizure and writ of replevin. In their reply, petitioners
asserted that the properties were immovable as defined in Article 415 of the Civil
Code, the parties' agreement to the contrary notwithstanding. Petitioners went to
the Court of Appeals via an original action for certiorari. The Court of Appeals ruled
that the subject machines were personal property as provided by the agreement of
the parties.
ISSUE: Whether or not the subject machines were personal, not real, property,
which may be a proper subject of a writ of replevin.
HELD: The contracting parties may validly stipulate that a real property be
considered as personal. After agreeing to such stipulation, they are consequently
estopped from claiming otherwise. Under the principle of estoppel, a party to a
contract is ordinarily precluded from denying the truth of any material fact found
therein. In the present case, the lease agreement clearly provides that the
machines in question are to be considered as personal properties. Clearly then,
petitioners were estopped from denying the characterization of the subject
machines as personal property. Under the circumstances, they are proper subject
of the writ of seizure. Accordingly, the petition was denied and the assailed
decision of the Court of Appeals was affirmed.
Burgos v. Chief of Staff, AFP
G.R. No. 64261, December 26, 1984, 133 SCRA 800
Escolin, J.
FACTS: On December 7, 1982, two search warrants where issued and the
premises at 19, Road 3, Project 6, Quezon City, and 784 Units C & D, RMS Building,
Quezon Avenue, Quezon City, business addresses of the "Metropolitan Mail" and
"We Forum" newspapers were searched. Office and printing machines, equipment,

5
paraphernalia, motor vehicles and other articles used in the printing, publication
and distribution of the said newspapers, as well as numerous papers, documents,
books and other written literature alleged to be in the possession and control of
Jose Burgos, Jr. publisher-editor of the "We Forum" newspaper, were seized.
ISSUE: Whether or not real properties were seized under the disputed warrants.
HELD: No. Under Article 415 (5) of the Civil Code, "machinery, receptacles,
instruments or implements intended by the owner of the tenement for an industry
or works which may be carried on in a building or on a piece of land and which
tend directly to meet the needs of the said industry or works" are considered
immovable property. In Davao Sawmill Co. v. Castillo, it was said that machinery
which is movable by nature becomes immobilized when placed by the owner of the
tenement, property or plant, but not so when placed by a tenant, usufructuary, or
any other person having only a temporary right, unless such person acted as the
agent of the owner. In the present case, petitioners do not claim to be the owners
of the land and/or building on which the machineries were placed. The
machineries, while in fact bolted to the ground, remain movable property
susceptible to seizure under a search warrant.
Lopez v. Orosa, Jr., and Plaza Theatre, Inc.
G.R. No. L-10817-18, February 28, 1958, 103 Phil. 98
Felix, J.
FACTS: Lopez was engaged in business under the name Lopez-Castelo Sawmill.
Orosa approached Lopez and invited the latter to make an investment in the
theatre business he was forming, the Plaza Theatre. Lopez expressed his
unwillingness to invest. Nonetheless, Lopez agreed to supply the lumber for the
construction of the theatre. Lopez further agreed that that the payment therefore
would be on demand and not cash on delivery basis. Lopex delivered the lumber
which was used for the construction of the Plaza Theatre. However, of the total
cost of materials amounting to P62, 255.85, Lopez was paid only P 20, 848.50, thus
leaving a balance of P 41, 771.35.
Due to Lopez demands, Orosa issued a deed of assignment over his shares of
stock of the Plaza Theatre, Inc. As there was still an unpaid balance, Lopez filed a
case against Orosa and Plaza Theatre. He asked that Orosa and Plaza theatre be
held liable solidarily for the unpaid balance, and in case defendants failed to pay,
the land and building should be sold in public auction with the proceeds to be
applied to the balance, or that the shares of stock be sold in public auction.
ISSUE: Whether or not the lien for the value of the materials used in the
construction of the building attaches to said structure alone and does not extend
to the land on which the building is adhered to.
HELD: No. While it is true that generally, real estate connotes the land and the
building constructed thereon, it is obvious that the inclusion of the building,
separate and distinct from the land, in the enumeration of what may constitute
real properties could only mean one thingthat a building is by itself an
immovable property. In view of the absence of any specific provision to the
contrary, a building is an immovable property irrespective of whether or not said
structure and the land on which it is adhered to belong to the same owner. The lien
so created attaches merely to the immovable property for the construction or
repair of which the obligation was incurred. Therefore, the lien in favor of appellant
for the unpaid value of the lumber used in the construction of the building attaches
only to said structure and to no other property of the obligors.

Yap v. Taada
G.R. No. L-32917, July 18, 1988, 163 SCRA 464
Narvasa, J.
FACTS: Goulds Pumps International (Phil.), Inc. filed a complaint against Yap and
his wife seeking recovery of P1,459.30 representing the balance of the price and
installation cost of a water pump in the latter's premises. Goulds presented
evidence ex parte and judgment by default was rendered by Judge Taada
requiring Yap to pay to Goulds the unpaid balance of the pump purchased by him
and interest of 12% per annum.
Thereafter, the water pump in question was levied by the sherif and by notice
dated November 4, 1969, scheduled the execution sale thereof. But in view of the
pendency of Yap's motion for reconsideration, suspension of the sale was directed.
It appears however that a copy of the order suspending the sale was not
transmitted to the sherif Hence, the Deputy Provincial Sherif went ahead with the
scheduled auction sale and sold the property levied on to Goulds as the highest
bidder.
Yap argues that "the sale was made without the notice required by Sec. 18, Rule
39, of the New Rules of Court," i.e., notice by publication in case of execution sale
of real property, the pump and its accessories being immovable because attached
to the ground with character of permanency (Art. 415, Civil Code).
ISSUE: Whether or not the water pump in question is an immovable property.
HELD: No. Yap's argument is untenable. The Civil Code considers as immovable
property, among others, anything "attached to an immovable in a fixed manner, in
such a way that it cannot be separated therefrom without breaking the material or
deterioration of the object." The pump does not fit this description. It could be, and
was in fact separated from Yap's premises without being broken or sufering
deterioration. Obviously, the separation or removal of the pump involved nothing
more complicated than the loosening of bolts or dismantling of other fasteners.

Machinery and Engineering Supplies, Inc. v. Court of Appeals


G.R. No. L-7057, October 29, 1954, 96 Phil. 70
Concepcion, J.
FACTS: Petitioner Machinery and Engineering Supplies filed a complaint for
replevin for the recovery of the machinery and equipment sold and delivered to Ipo
Limestone Co. An order was issued to seize and take immediate possession of the
properties specified in the order. Upon carrying out the courts order, Roco, the
companys President, along with a crew of technical men and labourers, proceeded
to the factory. The manager of Ipo Limestone Co. and Torres protested against the
seizure of the properties on the ground that they are not personal properties.
However, since the sherif contended that his duty is purely ministerial, they all
went to the factory and dismantled the equipment despite the fact that the
equipment could not be dismantled without causing damage or injuries to the
wooden frames attached to them. Consequently, they had to cut some of the
supports of the equipment which rendered its use impracticable.
ISSUE: Whether or not the machinery and equipment in question could be the
subject of replevin.

7
HELD: No. Replevin is applicable only to personal property. The machinery and
equipment in question appeared to be attached to the land, particularly to the
concrete foundation of said premises, in a fixed manner, in such a way that the
former could not be separated from the latter without breaking the material or
deterioration of the object. Hence, in order to remove the said outfit, it became
necessary not only to unbolt the same, but also to cut some of its wooden
supports. Moreover, said machinery and equipment were intended by the owner of
the tenement for an industry carried on said immovable. For these reasons, they
were already immovable pursuant to paragraphs 3 and 5 of Article 415 of the Civil
Code.
FELS Energy, Inc. v. The Province of Batangas
G.R. No. 168557, February 16, 2007
Callejo, Sr., J.
FACTS: On January 18, 1993, NPC entered into a lease contract with Polar Energy,
Inc. over diesel engine power barges moored at Balayan Bay in Calaca, Batangas.
The contract staes that NPC shall be responsible for the payment of all taxes other
levies imposed government to which POLAR may be or become subject to in
respect of the Power Barges. Subsequently, Polar Energy, Inc. assigned its rights
under the agreement to FELS Energy Inc.
On August 7, 1995, FELS received an assessment of real property taxes on the
power barges from Provincial Assessor of Batangas City. The assessed tax
amounted to P56,184,088.40 per annum. FELS referred the matter to NPC,
reminding it of its obligation under the agreement to pay all real estate taxes. NPC
sought reconsideration of the Provincial Assessors decision to assess real property
taxes on the power barges, alleging that barges are non-taxable items. In its
answer, the Provincial Assessor averred that the barges were real property for
purposes of taxation under Section 199(c) of Republic Act (R.A.) No. 7160.
ISSUE: Whether power barges, which are floating and movable, are personal
properties and therefore, not subject to real property tax.
HELD: NO. The power barges are real property and are thus subject to real
property tax. Tax assessments by tax examiners are presumed correct and made in
good faith, with the taxpayer having the burden of proving otherwise. Besides,
factual findings of administrative bodies, which have acquired expertise in their
field, are generally binding and conclusive upon the Court.
CHINA BANKING CORPORATION vs. HON. COURT OF APPEALS
G.R. No. 129644, March 7, 2000
FACTS:
In connection with a civil case filed by Metropolitan Bank against Alfonso Roxas
Chua, a notice of levy afecting the residential land of Alfonso and his wife was
issued. In 1985, the trial court rendered another decision in favor of China Banking
Corporation against Alfonso in a collection case. A certificate of sale covering one
half of the undivided portion of the property was executed in favor of Metro Bank.
In 1988, Alfonso executed Assignment of Right to Redeem to his son Paulino who
redeemed the said property on the same day. Another levy on execution in favor of
China Bank was issued on the same property. Thereafter, a certificate of sale on
execution was issued to China Bank in 1992. Paulino instituted a civil case arguing
that he has a better right over the title of China Bank because he redeemed the

8
property in 1988 while China Bank acquired its right in 1991. The trial court ruled
that the assignment was made for a valuable consideration and was executed two
years before China Bank levied the conjugal share of Chua. China Bank argued that
the assignment of right of redemption made by Alfonso to Paulino was done in
fraud of creditors and may be rescinded.
ISSUE: Whether the assignment by Alfonso to Paulino of the right of redemption
was done to defraud his creditors and therefore may be rescinded pursuant to Art.
1387.
HELD: YES, the assignment was done in fraud of creditors. The existence of fraud
with intent to defraud the creditor may either be presumed in accordance with
Article 1387 or duly proved in accordance with the ordinary rules of evidence.
Hence, the law presumes that there is fraud of creditors when:
a) There is alienation of property by gratuitous title by the debtor who has not
reserved sufficient property to pay his debts contracted before such alienation; or
b) There is alienation of property by onerous title made by a debtor against whom
some judgment has been rendered in any instance or some writ of attachment has
been issued. The decision or attachment need not refer to the property alienated
and need not have been obtained by the party seeking rescission.
In determining whether or not a certain conveyance is fraudulent the question in
every case is whether the conveyance was a bona fide transaction or a trick and
contrivance to defeat creditors, or whether it conserves to the debtor a special
right. It is not sufficient that it is founded on good considerations or is made with
bona fide intent. It must have both elements.
In the case at bar, the presumption that the conveyance is fraudulent has not
been overcome. At the time a judgment was rendered in favor of China Bank
against Alfonso, Paulino was still living with his parents in the subject property.
Paulino himself admitted that he knew his father was heavily indebted and could
not aford to pay his debts. The transfer was undoubtedly made between father
and son at the time when the father was insolvent and had no other property to
pay his creditors. Hence, it is of no consequence whether or not Paulino had given
valuable consideration for the conveyance.
Laurel v. Garcia
G.R. No. 92013, July 25, 1990, 187 SCRA 797
Gutierrez, J.
FACTS: In view of the Reparations Agreement between the Philippines and Japan,
four properties located in Japan were given to the Philippines. One of these
properties is the Roppongi property. The said property was formerly the location of
the Chancery of the Philippine Embassy until it was transferred to Nampeidai on
July 22, 1976. The Roppongi property has remained abandoned from the time of
the transfer due to lack of funds to develop the said property. Consequently,
Administrative orders were issued by the President authorizing the study of the
condition of the properties of the Philippines in Japan. Subsequently, Executive
Order 296 was issued by President Aquino allowing non-Filipinos to buy or lease
some of the properties of the Philippines located in Japan, including Roppongi.
Petitioners now contend that the Roppongi property cannot be alienated as it is
classified as public dominion and not of private ownership because it is a property
intended for public service under paragraph 2, article 420 of the Civil Code. On the
other hand, respondents aver that it has already become part of the patrimonial
property of the State which can be alienated because it has not been used for
public service for over 13 years. They further contend that EO 296 converted the
subject property to patrimonial property.

9
ISSUE: Whether or not the Roppongi property still forms part of the public
dominion hence cannot be disposed nor alienated.
HELD: Yes. The respondents failed to convincingly show that the property has
already become patrimonial. The fact that the Roppongi site has not been used for
a long time for actual Embassy service does not automatically convert it to
patrimonial property. Under Art. 422 of the Civil Code, there must be a definite and
a formal declaration on the part of the government to withdraw it from being
public. Abandonment must be a certain and a positive act based on correct legal
premises. The mere transfer of the embassy to Nampeidai is not a relinquishment
of the propertys original purpose.
The Administrative orders authorizing the study of the conditions of government
properties in Japan were merely directives for investigation but did not in any way
signify a clear intention to dispose of the properties. Likewise, EO 296 did not
declare that the properties lost their public character; it merely made them
available to foreigners in case of sale, lease or other disposition. Thus, since there
is no law authorizing its conveyance, the Roppongi property still remains part of
the inalienable properties of the State.
Rabuco v. Villegas
G.R. No. L-24916, February 28, 1974, 55 SCRA 658
Teehankee, J.
FACTS: The issue in this case involves the constitutionality of Republic Act No.
3120 whereby the Congress converted the lots in question together with another
lot in San Andres, Malate that are reserved as communal property into disposable
or alienable lands of the State. Such lands are to be placed under the
administration and disposal of the Land Tenure Administration for subdivision into
small lots not exceeding 120 square meters per lot for sale on instalment basis to
the tenants or bona fide occupants thereof and expressly prohibited ejectment and
demolition of petitioners' homes under Section 2 of the Act. Respondent contends
that the Act is invalid and unconstitutional for it constitutes deprivation of property
without due process of law and without just compensation.
ISSUE: Whether or not Republic Act No. 3120 is constitutional.
HELD: Yes. The lots in question are manifestly owned by the city in its public and
governmental capacity and are therefore public property over which Congress had
absolute control as distinguished from patrimonial property owned by it in its
private or proprietary capacity of which it could not be deprived without due
process and without just compensation. It is established doctrine that the act of
classifying State property calls for the exercise of wide discretionary legislative
power, which will not be interfered with by the courts. The Acts in question were
intended to implement the social justice policy of the Constitution and the
government program of land for the landless and that they were not intended to
expropriate the property involved but merely to confirm its character as communal
land of the State and to make it available for disposition by the National
Government. The subdivision of the land and conveyance of the resulting
subdivision lots to the occupants by Congressional authorization does not operate
as an exercise of the power of eminent domain without just compensation in
violation of Section 1, subsection (2), Article III of the Constitution, but simply as a
manifestation of its right and power to deal with state property.

10
Macasiano v. Diokno
G.R. No. 97764, August 10, 1992, 212 SCRA 464
Medialdea, J.
FACTS: The Municipality of Paranque passed an ordinance that authorized the
closure of J. Gabriel, G.G. Cruz, Bayanihan, Lt. Garcia Extension and Opena Streets
located at Baclaran, Paranaque Metro Manila and the establishment of a flea
market thereon. Thereafter, the municipal council of Paranaque issued a resolution
authorizing Paranaque Mayor Walfrido N. Ferrer to enter into a contract with any
service cooperative for the establishment, operation, maintenance and
management of flea markets and/or vending areas. By virtue of this, respondent
municipality and respondent Palanyag, a service cooperative, entered into an
agreement whereby the latter shall operate, maintain and manage the flea market
in the aforementioned streets with the obligation to remit dues to the treasury of
the municipal government of Paranaque. Consequently, market stalls were put up
by Palanyag on the said streets.
Petitioner Macasiano, PNP Superintendent of the Metropolitan Traffic Command,
then ordered the destruction and confiscation of the stalls along the
abovementioned streets. Hence, respondents filed with the trial court a joint
petition for prohibition and mandamus with damages and prayer for preliminary
injunction, to which the petitioner filed his opposition to the issuance of the writ of
preliminary injunction. The trial court upheld the validity of the ordinance in
question.
ISSUE: Whether or not an ordinance or resolution which authorizes the lease and
use of public streets or thoroughfares as sites for flea markets is valid.
HELD: No. The aforementioned streets are local roads used for public service and
are therefore considered public properties of respondent municipality. Article 424
of the Civil Code provides that properties of public dominion devoted for public use
and made available to the public in general are outside the commerce of man and
cannot be disposed of or leased by the local government unit to private persons.
Properties of the local government which are devoted to public service are deemed
public and are under the absolute control of Congress. Hence, LGUs have no
authority whatsoever to control or regulate the use of public properties unless
specific authority is vested upon them by Congress.
Republic of the Philippines v. Court of Appeals
G.R. No. 100709, November 14, 1997, 281 SCRA 639
Panganiban, J.
FACTS: Morato filed for a patent on a parcel of land located in Calauag, Quezon,
which was approved, provided that the land shall not be encumbered or alienated
within a period of five years from the date of the issuance of the patent. Later on,
the land was established to be a portion of Calauag Bay, which was five to six feet
deep during high tides and three feet deep on low tides. The water level rose
because of the ebb and flow of tides from the bay and the storms that frequently
passed through the area. Furthermore, it was observed by the Director of Lands
from his investigation, that the land of Morato was leased to Advincula for P100 per
month and it was also mortgaged to Co for P10,000. The Director of Lands filed a
suit with the contention that Morato violated the 5-year prohibitory period and thus
the patent should be cancelled and the land should revert back to the State.
ISSUE: Whether or not there is a violation of the prohibition of the patent, and
thus, the subject land should revert back to the ownership of the State.

11
HELD: Yes. The lease was an encumbrance included in the prohibitions of the
patent because it impairs the use of the land by Morato herself. As for the
mortgage, it is a legal limit on the title and if there will be foreclosure because
Morato was not able to pay her debts, the property will be auctioned. It is also a
limitation on Morato's right to enjoy and possess the land for herself.
Encumbrance, as defined, is an impairment on the use or transfer of property, or a
claim or lien on the property where there is a burden on the title. Thus, Morato
clearly violated the terms of the patent on these points. Moreover, the property
became a foreshore land because it turned into a portion of land which was
covered most of the time with water, whether it was low or high tide. Foreshore is
defined as land between high and low waters which is dry depending on the reflux
or ebb of the tides. In accordance with this land reclassification, the land can no
longer be subject to a pending patent application and must be returned to the
State.
Province of Zamboanga del Norte v. City of Zamboanga
G.R. No. L-24440, March 28, 1968, 22 SCRA 1334
Bengzon, J.P., J.
FACTS: On June 6, 1952, Republic Act 711 was approved dividing the province of
Zamboanga into two (2): Zamboanga del Norte and Zamboanga del Sur. Republic
Act 3039 was approved providing that all buildings, properties and assets
belonging to the former province of Zamboanga and located within the City of
Zamboanga are hereby transferred, free of charge, in favor of the said City of
Zamboanga.
Plaintif-appellee Zamboanga del Norte filed a complaint in the Court of First
Instance of Zamboanga del Norte against defendants-appellants Zamboanga City,
the Secretary of Finance and the Commissioner of Internal Revenue. It was prayed
that Republic Act 3039 be declared unconstitutional for depriving plaintif province
of property without due process and just compensation. Included in the properties
were the capital site and capitol building, certain school sites, hospital and
leprosarium sites, and high school playground.
ISSUE: Whether or not the properties mentioned are properties for public use or
patrimonial.
HELD: The subject properties are properties for public use. The validity of the law
ultimately depends on the nature of the lots and buildings in question. The
principle itself is simple: If the property is owned by the municipality (meaning
municipal corporation) in its public and governmental capacity, the property is
public and Congress has absolute control over it. But if the property is owned in its
private or proprietary capacity, then it is patrimonial and Congress has no absolute
control. The municipality cannot be deprived of it without due process and
payment of just compensation.
Applying the norm obtaining under the principles constituting the law of Municipal
Corporations, all those of the 50 properties in question which are devoted to public
service are deemed public; the rest remain patrimonial. Under this norm, to be
considered public, it is enough that the property be held and, devoted for
governmental purposes like local administration, public education, public health,
etc.
Regarding the several buildings existing on the lots above-mentioned, the records
do not disclose whether they were constructed at the expense of the former

12
Province of Zamboanga. Considering however the fact that said buildings must
have been erected even before 1936 when Commonwealth Act 39 was enacted
and the further fact that provinces then had no power to authorize construction of
buildings such as those in the case at bar at their own expense, it can be assumed
that said buildings were erected by the National Government, using national funds.
Hence, Congress could very well dispose of said buildings in the same manner that
it did with the lots in question.
Chavez v. Public Estates Authority
G.R. No. 133250, July 9, 2002
Carpio, J.
FACTS: In 1973, the Government through the Commissioner of Public Highways
and the Construction and Development Corporation of the Philippines (CDCP)
signed a contract to reclaim certain foreshore and ofshore areas of Manila Bay. PD
1084 was issued, creating Public Estates Authority (PEA), and PD 1085, transferring
the reclaimed lands under the MCCRRP to PEA.
In 1995, PEA entered into a Joint Venture Agreement (JVA) with AMARI, a private
corporation to develop the Freedom Islands, and the JVA was approved by
President Ramos. However, PEA and AMARI entered into the JVA through
negotiation without public bidding. A Legal Task Force was created to look into the
issue. The said task force upheld the legality of the JVA.
In 1998, Frank I. Chavez, as a taxpayer, filed a petition to compel PEA to disclose
all facts on its negotiations with AMARI, invoking the constitutional right of the
people to information on matters of public concern. He assails the sale to AMARI of
lands of the public domain as a blatant violation of the constitutional prohibiting in
the sale of alienable lands of the public domain to private corporations.
Despite the ongoing court petitions, PEA and AMARI signed an Amended Joint
Venture Agreement (Amended JVA) in 1999, and such was approved by President
Estrada. The Amended JVA seeks to convey to AMARI the ownership of 77.34
hectares of the Freedom Islands.
ISSUE: Whether AMARI has the capacity to acquire the lands held by PEA.
HELD: No. Under the 1987 Constitution, private corporations such as AMARI
cannot acquire alienable land of the public domain. Reclaimed lands comprising
the Freedom Islands, which are covered by certificates of title in the name of PEA,
are alienable lands of the public domain. PEA may lease these lands to private
corporations but may not sell or transfer ownership of these lands to private
corporations. PEA may only sell these lands to Philippine citizens, subject to the
ownership limitations in the 1987 Constitution and existing laws. Thus, the
Amended Joint Venture Agreement between AMARI and PEA was null and void.
VILLARICO V. SARMIENTO
442 scra 110
FACTS:
Villarico here is an owner of a lot that is separated from the Ninoy Aquino Avenue h
ighway by a strip of land belonging to the government. Vivencio Sarmiento had a
building constructed on a portion of the said government land and a part thereof
was occupied by Andoks Litson Corp.

13
In 1993, by means of a Deed of Exchange of Real Property, Villarico acquired a
portion of the same area owned by the government.
He then filed an accion publiciana alleging that respondents (Vivencio) on the
government land closed his right of way to the Ninoy Aquino Avenue and
encroached on a portion of his lot.
ISSUE: Whether or not VIllarico has a right of way to the NAA.
HELD: No. It is not disputed in this case that the alleged right of way to the lot
belongs to the state or property of public dominion.
It is intended for public use meaning that it is not confined to privileged individuals
but is open to the indefinite public. Records show that the lot on which the
stairways were built is for the use of the people as passageway hence, it is a
property for public dominion.
1. Public dominion property is outside the commerce of man and hence, it
cannot be:
2. Alienated or leased or otherwise be the subject matter of contracts
3. Acquired by prescription against the state
4. Cannot be the subject of attachment and execution
5. Be burdened by any voluntary easement
6. It cannot be burdened by a voluntary easement of right of way in favo rof the
petitioner and petitioner cannot appropriate it for himself and he cannot
claim any right of possession over it.
REPUBLIC V. T.A.N. PROPERTIES INC.
555 SCRA 477, G.R. NO. 154953; JUNE 26, 2008
FACTS: In 1999, T.A.N. Properties filed in the RTC of Batangas an application for the
registration of a land, located at Sto. Tomas, Batangas and with an area of
56.4007hectares. To support its application, it submitted two certificates, issued by
CENROand FMS-DENR and both certifying that the land applied for was alienable
anddisposable. The Republic of the Philippines, represented by the Director of
Lands, opposed the application on the ground that T.A.N. Properties did not prove
that theland was alienable and disposable.
ISSUE: Whether or not the applicant proved that, the land is alienable
anddisposable.
HELD: No.It is the burden of the applicant to prove that the land subject to
registration is alienable and disposable and for such the applicant must prove that
the DENR Secretary had approved the land classification and released the land of
the public domain as alienable and disposable. In the present case, T.A.N.
Properties did not provide the needed proof. Forthe documents provided by the
company, the Court cited DENR AdministrativeOrder No. 20 (DAO No. 20) and DAO
No. 38; DAO No. 20 proves that FMS-DENR has no authority to issue certificates,
classifying lands to be alienable and disposable; and DAO No. 38 provides that
CENRO can issue certificates of land classification for lands having a maximum

14
area of 50 hectares. The land applied for in the case has an area of 56.4007
hectares, thus CENRO has no jurisdiction over it. It is clear from the
aforementioned DAOs that the documents submitted by T.A.N. Properties did not
prove that the land is alienable and disposable.
Manila International Airport Authority v. Court of Appeals
G.R. No. 155650, July 20, 2006
Carpio, J.
FACTS: MIAA received Final Notices of Real Estate Tax Delinquency from the City of
Paraaque for the taxable years 1992 to 2001. MIAAs real estate tax delinquency
was estimated at P624 million. Thus, the City of Paraaque, through its City
Treasurer, issued notices of levy and warrants of levy on the Airport Lands and
Buildings. The Mayor of the City of Paraaque threatened to sell at public auction
the Airport Lands and Buildings should MIAA fail to pay the real estate tax
delinquency. City of Paraaque contends that Section 193 of the Local Government
Code expressly withdrew the tax exemption privileges of government-owned andcontrolled corporations upon the efectivity of the Local Government Code.
However, MIAA avers that airport lands and buildings are owned by the State, and
thus, exempt from tax.
ISSUE: Whether or not airport lands and buildings of MIAA are exempt from real
estate tax.
HELD: Yes. MIAA is a government instrumentality vested with corporate powers to
perform efficiently its governmental functions. MIAA is like any other government
instrumentality, the only diference is that MIAA is vested with corporate powers.
Unless the government instrumentality is organized as a stock or non-stock
corporation, it remains a government instrumentality exercising not only
governmental but also corporate powers. Thus, MIAA exercises the governmental
powers of eminent domain, police authority and the levying of fees and charges.
The airport lands and buildings of MIAA are property of public dominion and
therefore owned by the State or the Republic of the Philippines. Hence, the subject
properties are not subject to tax.
Javier v. Veridiano II
G.R. No. L-48050, October 10, 1994, 237 SCRA 565
Bellosillo, J.
FACTS: Javier filed a Miscellaneous Sales Application for lot 1641. She later
instituted a complaint for forcible entry against Babol, alleging that she was
forcibly dispossessed of a portion of said land. The case for forcibly entry was
however dismissed as it was found by the court that the occupied portion was
outside Lot 1641. The same was dismissed on appeal. Javier was eventually
granted a Miscellaneous Sales Patent and issued an OCT for lot 1641. Babol,
however had sold the property he was occupying, including a portion of 200 square
meters to Rosete. Javier demanded the surrender of the same area from Rosete
who repeatedly refused to comply. After 4 years, Javier instituted a complaint for
quieting of title and recovery of possession with damages against Babol and
Rosete. Rosete moved to dismiss the complaint on the ground of res judicata. The
CFI sustained the argument of Rosete and granted his motion to dismiss. Javier
contends that res judicata cannot apply in the instant case since there is no
identity of parties and causes of action between her complaint for forcible entry,
which had long become final and executory, and her subsequent petition for
quieting of title. Javier maintains that there is no identity of causes of action since

15
the first case was for forcible entry, which is merely concerned with the possession
of the property, whereas the subsequent case was for quieting of title, which looks
into the ownership of the disputed land.
ISSUE: Whether or not there are really diferent causes of action between the
forcible entry case and the later quieting of title case.
HELD: Yes. For res judicata to bar the institution of a subsequent action the
following requisites must concur: (1) There must be a final judgment or order; (2)
The court rendering the judgment must have jurisdiction over the subject matter;
(3) The former judgment is a judgment on the merits; and, (4) There is between
the first and second actions identity of (4a) parties, (4b) of subject matter and (4c)
of causes of action.
Javier's argument that there is no identity of parties between the two actions is
without merit. We have repeatedly ruled that for res judicata to apply, what is
required is not absolute but only substantial identity of parties. But, there is merit
in Javier's argument that there is no identity of causes of action.
"The only issue in an action for forcible entry is the physical or material possession
of real property, that is, possession de facto and not possession de jure. The
philosophy underlying this remedy is that irrespective of the actual condition of the
title to the property, the party in peaceable quiet possession shall not be turned
out by strong hand, violence or terror." A judgment rendered in a case for recovery
of possession is conclusive only on the question of possession and not on the
ownership. It does not in any way bind the title or afects the ownership of the land
or building.
On the other hand, Civil Case No. 2203-0 is in reality an action to recover a parcel
of land or an accion reivindicatoria under Art. 434 of the Civil Code, and should be
distinguished from Civil Case No. 926, which is an accion interdictal. Accion
interdictal, which is the summary action for forcible entry (detentacion) where the
defendant's possession of the property is illegal ab initio, or the summary action
for unlawful detainer (desahuico) where the defendant's possession was originally
lawful but ceased to be so by the expiration of his right to possess, both of which
must be brought within one year from the date of actual entry on the land, in case
of forcible entry, and from the date of last demand, in case of unlawful detainer, in
the proper municipal trial court or metropolitan trial court; accion publiciana which
is a plenary action for recovery of the right to possess and which should be brought
in the proper regional trial court when the dispossession has lasted for more than
one year; and, accion reivindicatoria or accion de reivindicacion which seeks the
recovery of ownership and includes the jus utendi and the jus fruendi brought in
the proper regional trial court.
Accion reivindicatoria or accion de reivindicacion is thus an action whereby plaintif
alleges ownership over a parcel of land and seeks recovery of its full possession. It
is diferent from accion interdictal or accion publiciana where plaintif merely
alleges proof of a better right to possess without claim of title.
In Civil Case No. 926 Javier merely claimed a better right or prior possession over
the disputed area without asserting title thereto. It should be distinguished from
Civil Case No. 2203-0 where she expressly alleged ownership.
Bustos v. Court of Appeals
G.R. No. 120784-85, January 24, 2001, 350 SCRA 155
Pardo, J.

16
FACTS: Paulino Fajardo died intestate on April 2, 1957. He had four (4) children,
namely: Manuela, Trinidad, Beatriz and Marcial, all surnamed Fajardo. On
September 30, 1964, the heirs executed an extra-judicial partition of the estate of
Paulino Fajardo. On the same date, Manuela sold her share to Moses G. Mendoza,
husband of Beatriz by deed of absolute sale. At the time of the sale, there was no
cadastral survey in Masantol, Pampanga. Later, the cadastre was conducted and
the property involved in the partition case was specified as Lots 280, 283, 284,
1000-A and 1000-B. The share of Manuela, which was sold to Moses, includes Lot
284 of the Masantol Cadastre and Lot 284 was subdivided into Lots 284-A and 284B. Trinidad was in physical possession of the land. She refused to surrender the
land to her brother-in-law Moses G. Mendoza, despite several demands.
On September 3, 1971, Moses filed with the Court of First Instance, Pampanga a
complaint for partition claiming the one fourth (1/4) share of Manuela which was
sold to him. During the pendency of the case for partition, Trinidad Fajardo died. On
December 15, 1984, the heirs executed an extra-judicial partition of the estate of
Trinidad Fajardo. On February 16, 1987, Lucio Fajardo Ignacio, son of Trinidad sold
Lot 284-B to spouses Venancio Viray and Cecilia Nunga-Viray.
On February 8, 1989, the Regional Trial Court, Pampanga, Macabebe, Branch 55
rendered a decision in favor of Moses G. Mendoza.In the meantime, on November
6, 1989, spouses Venancio Viray and Cecilia Nunga-Viray, buyers of Lucio Ignacio's
share of the property, filed with the Municipal Circuit Trial Court, MacabebeMasantol, Pampanga an action for unlawful detainer against spouses Bustos, the
buyers of Moses G. Mendoza, who were in actual possession as lessees of the
husband of Trinidad, Francisco Ignacio, of the subject land. The municipal circuit
trial court decided the case in favor of spouses Viray. Subsequently, the trial court
issued writs of execution and demolition, but stayed when spouses Bustos filed
with the regional Trial Court, Pampanga, Macabebe, Branch 55, a petition for
certiorari, prohibition and injunction. On December 18, 1992, the regional trial
court rendered a decision dismissing the case. On September 9, 1994, petitioners
filed a motion for reconsideration; however, on June 21, 1995, the Court of Appeals
denied the motion.
ISSUE: Whether or not petitioners could be ejected from what is now their own
land.
HELD: In this case, the issue of possession is intertwined with the issue of
ownership. In the unlawful detainer case, the Court of Appeals affirmed the
decision of the trial court as to possession on the ground that the decision has
become final and executory. This means that the petitioners may be evicted. In the
accion reinvindicatoria, the Court of Appeals affirmed the ownership of petitioners
over the subject land. Hence, the court declared petitioners as the lawful owners of
the land. In the present case, the stay of execution is warranted by the fact that
petitioners are now legal owners of the land in question and are occupants thereof.
To execute the judgment by ejecting petitioners from the land that they owned
would certainly result in grave injustice. Besides, the issue of possession was
rendered moot when the court adjudicated ownership to the spouses Bustos by
virtue of a valid deed of sale. Placing petitioners in possession of the land in
question is the necessary and logical consequence of the decision declaring them
as the rightful owners is possession. It follows that as owners of the subject
property, petitioners are entitled to possession of the same. "An owner who cannot
exercise the seven (7) "juses" or attributes of ownership-the right to possess, to
use and enjoy, to abuse or consume, to accessories, to dispose or alienate, to
recover or vindicate and to the fruits is a crippled owner.

17
Heirs of Roman Soriano v. Court of Appeals
G.R. No. 128177, August 15, 2001, 363 SCRA 87
Ynares Santiago, J.
FACTS: The land in dispute in this case is originally owned by Adriano Soriano who
died sometime in 1947. Adriano Soriano has 7 heirs whom leased the subject
parcel of land to David de Vera and Consuelo Villasista for a term of 15 years
starting July 1, 1967. The lease contract states that Roman Soriano will serve as
the caretaker of the said property during the period of lease. During the efectivity
of the lease contract, the heirs of Adriano Soriano entered into extrajudicial
settlement of his estate. As a result of the settlement, the property was divided
into two property, Lot No. 60052 which was assigned to Lourdes and Candido, heirs
of Adriano and the heirs of Dionisia another heir of Adriano. The other property, Lot
No. 8459 was assigned to Francisco, Librada, Elcociado and Roman all heirs of
Adriano. The owners of Lot No. 60052 sold the lot to spouses Braulio and Aquiliana
Abalos, and the owners of Lot No. 8459, except Roman also sold their shares to
spouses Briones.
On March 14, 1968, the de Vera spouses ousted Roman as caretaker and appointed
Isidro Versoza and Vidal Versoza as his substitute. Roman filed a case for
reinstatement and reliquidation against the de Vera spouses in CAR Case No. 1724P-68. On September 30, 1969, the Agrarian Court rendered a decision authorizing
the ejectment of Roman. On appeal, the decision was reversed by the Court of
Appeals. The deicion became final and executor. However, before it was executed,
the parties entered into a post-decisional agreement wherein the de Vera spouses
allowed Roman Soriano to sub-lease the property until the termination of the
original lease on June 30, 1982. This agreement was approved by the CAR court in
an order dated December 22, 1972.
On August 16, 1976, the Abalos spouses applied for the registration of the disputed
parcel of land. Roman Soriano and the Director of Lands acted as oppositors. On
June 27, 1983, the Land Registration Court granted the application for registration.
On April 13, 1983, after the expiration of the original lease and sub-lease in favor
of Roman Soriano, the Abalos spouses filed a case for unlawful detainer against
Roman Soriano, later, this case was dismissed on motion of the Abalos spouses. On
July 14, 1983, Elcociado, Librada, Roman, Francisco, Lourdes, Candido and the
heirs of Dionisia filed a complaint to annul the deeds of sale they executed in favor
of the Abalos spouses or should the deeds be not annulled, to allow Roman,
Elcociado and Librada to redeem their shares in the disputed land and to uphold
Roman Sorianos possession of the fishpond portion of the property as a tenantcaretaker.
After the dismissal of the case for unlawful detainer, the Abalos spouses filed on
August 22, 1984, a motion for execution of the post-decisional order embodying
the agreement of Roman Soriano and the de Vera spouses allowing the former to
sublease the property. On October 25, 1984, Roman filed a motion to suspend
hearing on the rental demanded by the Abalos spouses until after the other issues
raised in his opposition to the motion for execution are resolved. The motion to
suspend hearing on the issue of the rentals was denied and the trial court
authorized the substitution of the de Vera spouses by the Abalos spouses. Roman
Soriano's motion for reconsideration was denied on March 16, 1985. Roman filed
petition for certiorari and prohibition in the Court of Appeals but the latter denied
the petition, pending the denial of this petition, Roman Soriano died. Not satisfied
with the decision of the Court of Appeals, the heirs of Roman Soriano brought this
case in the Supreme Court.

18
ISSUE: Whether or not a winning party (ABALOS) in a land registration case can
efectively eject the possessor (SORIANO) thereof, whose security of tenure rights
is still pending determination before the DARAB.
HELD: No. The Court held that a judgment in a land registration case cannot
efectively used to oust the possessor of the land, whose security of tenure rights
are still pending determination before the DARAB. There is no dispute that Abalos
spouses' title over the land under litigation has been confirmed with finality.
However, the declaration pertains only to ownership and does not automatically
include possession, especially soin the instant case where there is a third party
occupying the said parcel of land, allegedly in the concept of an agricultural
tenant. Agricultural lessees are entitled to security of tenure and they have the
right to work on their respective landholdings once the leasehold relationship is
established. Security of tenure is a legal concession to agricultural lessees which
they value as life itself ad deprivation of their landholdings is tantamount to
deprivation of their only means of livelihood. The exercise of the right of
ownership, then, yields to the exercise of the rights of an agricultural tenant. The
Supreme Court decided to refrain from ruling whether petitioners may be
dispossessed of the subject property while petitioner's status as tenant has not yet
been declared by the DARAB.
Garcia v. Court of Appeals
G.R. No. 133140, August 10, 1999, 312 SCRA 180
Puno, J.
FACTS: Petitioner Atty. Pedro Garcia, with the consent of his wife Remedios Garcia,
sold a parcel of land situated at Bel Air II Village, Makati to his daughter Maria Luisa
Magpayo and her husband Luisito Magpayo. The Magpayos mortgaged the land to
the Philippine Bank of Communications (PBCom) to secure a loan. The Magpayos
failed to pay their loan upon its maturity, hence, the mortgage was extrajudicially
foreclosed and at the public auction sale in which PBCom bought the land. The
redemption period of the foreclosed mortgage expired without the Magpayos
redeeming the same, hence, title over the land was consolidated in favor of
PBCom.
PBCom subsequently filed a petition for the issuance of a writ of possession over
the land with the Regional Trial Court (RTC) of Makati. The RTC granted the petition.
Upon service of the writ of possession, Maria Luisa Magpayos brother, Jose Ma. T.
Garcia, who was in possession of the land, refused to honor it. Jose Garcia
thereupon filed against PBCom, the Magpayos, and the RTC Sherif the instant suit
for recovery of realty and damages wherein he contended, inter alia, that at the
time of the alleged sale to the Magpayo spouses, he was in possession of the
property; that, when his mother Remedios Tablan Garcia died, sometime in
October, 1980, he became, by operation of law, a co-owner of the property; and
that, Atty. Pedro V. Garcia, at the time of the execution of the instrument in favor of
the Magpayo spouses was not in possession of the subject property.
ISSUE: Whether or not Jose Magpayo was a co-owner of the parcel of the land in
dispute.
HELD: No. Possession and ownership are distinct legal concepts. Ownership exists
when a thing pertaining to one person is completely subjected to his will in a
manner not prohibited by law and consistent with the rights of others. Ownership
confers certain rights to the owner, one of which is the right to dispose of the thing
by way of sale. Atty. Pedro Garcia and his wife Remedios exercised their right to
dispose of what they owned when they sold the subject property to the Magpayo

19
spouses. On the other hand, possession is defined as the holding of a thing or the
enjoyment of a right. Literally, to possess means to actually and physically occupy
a thing with or without right. Possession may be had in one of two ways:
possession in the concept of an owner and possession of a holder. A possessor in
the concept of an owner may be the owner himself or one who claims to be so. On
the other hand, one who possesses as a mere holder acknowledges in another a
superior right which he believes to be ownership, whether his belief be right or
wrong. The records show that petitioner Jose Garcia occupied the property not in
the concept of an owner for his stay was merely tolerated by his parents. An
owners act of allowing another to occupy his house, rent-free does not create a
permanent and indefeasible right of possession in the latters favor. Consequently,
it is of no moment that petitioner was in possession of the property at the time of
the sale to the Magpayo spouses. It was not a hindrance to a valid transfer of
ownership. All said, the Magpayo spouses were already the owners when they
mortgaged the property to PBCom.
Rodil Enterprises, Inc. v. Court of Appeals
G.R. No. 129609, November 29, 2001, 371 SCRA 79
Bellosillo, J.
FACTS: Rodil Enterprises Inc. (RODIL) is the lessee of the Ides O'Racca Building
(O'RACCA) since 1959 which is a property owned by the Republic of the Philippines.
In 1980, Rodil entered into a sublease contract with respondents Carmen Bondoc,
Teresita Bondoc-Esto, Divisoria Footwear and Chua Huay Soon, members of the
Ides ORacca Building Tenants Association, Inc. On 12 September 1982 BP 233 was
enacted. It authorized the sale of "former alien properties" classified as commercial
and industrial, and the O'RACCA building was classified as commercial property.
RODIL and Ides ORacca Building Tenants Association, Inc., ofered to purchase the
subject property. Pending action on the ofer of RODIL to purchase the property,
Director Factora of the Building Services and Real Property Management Office
granted RODIL's request for another renewal of the lease contract on 23
September 1987 for another five (5) years from 1 September 1987. The renewal
contract was forwarded to then Secretary Jose de Jesus of Department of General
Services and Real Estate Property Management (DGSREPM) for approval. Upon
recommendation of DGSREPM Rufino Banas, De Jesus disapproved the renewal
contract in favour of Rodil and recalled all papers signed by him regarding the
subject. Secretary De Jesus likewise directed RODIL to pay its realty tax
delinquency and ordered the issuance of a temporary occupancy permit to the
ASSOCIATION.
On 6 October 1987 RODIL filed an action for specific performance, damages and
injunction with prayer for temporary restraining order before the Regional Trial
Court of Manila against the REPUBLIC, De Jesus, Banas, Factora and the
ASSOCIATION. De Jesus, Banas and Factora were later substituted by Secretary
Fulgencio Factoran of the Department of Environment and Natural Resources
(DENR) in the action for specific performance. On 31 May 1988 Factora issued
Order No. 1 designating the Land Management Bureau represented by Director
Abelardo Palad, Jr. as custodian of all "former alien properties" owned by the
REPUBLIC. Pending the action for specific performance, RODIL signed a renewal
contract with Director Palad which was approved by Secretary Factora. The renewal
contract would extend the lease for ten (10) years from 1 September 1987. A
supplement to the renewal contract was subsequently entered into on 25 May
1992 where rentals on the previous lease contract were increased. As a result, the
action was dismissed in favour of Rodil. Rodil then filed an action for unlawful
detainer against Divisoria Footwear, Bondoc, Bondoc-Esto and Chua Huay Soon.

20
Upon appeal, the Court of Appeals declared the contracts null and void and
dismissed the actions for unlawful detainer.
ISSUE: Whether or not Rodil has the right to occupy the building by virtue of its
lease contract with the Republic.
HELD: Yes. The owner has the right to enjoy and dispose of a thing, without other
limitations than those established by law. Every owner has the freedom of
disposition over his property. It is an attribute of ownership, and this rule has no
exception. The REPUBLIC being the owner of the disputed property enjoys the
prerogative to enter into a lease contract with RODIL in the exercise of its jus
disponendi. Hence, as lessor, the REPUBLIC has the right to eject usurpers of the
leased property where the factual elements required for relief in an action for
unlawful detainer are present.
Private respondents claim that the agreements of 23 September 1987, 18 May
1992 and 25 May 1992 did not give rise to valid contracts.This is true only of the
Contract of Lease entered into on 23 September 1987 which the REPUBLIC did not
approve. RODIL neither alleged nor proved that such approval was made known to
it. The so-called approval of the lease contract was merely stated in an internal
memorandum of Secretary De Jesus addressed to Director Factora. This is evident
from the fact that Secretary De Jesus, in his letter, asked Factora to duly execute a
lease contract and forward it to his office for approval. The consequences of this
fact are clear. The Civil Code provides that no contract shall arise unless
acceptance of the contract is communicated to the oferor. Until that moment,
there is no real meeting of the minds, no concurrence of ofer and acceptance,
hence, no contract.
However, the same is not true of the contracts of 18 May 1992 and 25 May 1992.
As argued by RODIL, these contracts are not proscribed by law; neither is there a
law prohibiting the execution of a contract with provisions that are retroactive.
Where there is nothing in a contract that is contrary to law, morals, good customs,
public policy or public order, the validity of the contract must be sustained.
The Court of Appeals invalidated the contracts because they were supposedly
executed in violation of a temporary restraining order issued by the Regional Trial
Court. The appellate court however failed to note that the order restrains the
REPUBLIC from awarding the lease contract only as regards respondent
ASSOCIATION but not petitioner RODIL. While a temporary restraining order was
indeed issued against RODIL, it was issued only on 25 May 1992 or after the
assailed contracts were entered into. As correctly stated by petitioner, one cannot
enjoin an act already fait accompli.
Private respondents argue that the "renewal contract" cannot "renew" a void
contract. However, they could cite no legal basis for this assertion. It would seem
that respondents consider the renewal contract to be a novation of the earlier
lease contract of 23 September 1987. However, novation is never presumed. Also,
the title of a contract does not determine its nature. On the contrary, it is the
specific provisions of the contract which dictate its nature. Furthermore, where a
contract is susceptible of two (2) interpretations, one that would make it valid and
another that would make it invalid, the latter interpretation is to be adopted. The
assailed agreement of 18 May 1992, "Renewal of Contract of Lease," merely states
that the term of the contract would be for ten (10) years starting 1 September
1987. This is hardly conclusive of the existence of an intention by the parties to
novate the contract of 23 September 1987. Nor can it be argued that there is an
implied novation for the requisite incompatibility between the original contract and
the subsequent one is not present. Based on this factual milieu, the presumption of
validity of contract cannot be said to have been overturned.

21
Respondent ASSOCIATION claims that the Decision of the Office of the President
declaring null and void the lease contracts of 18 May 1992 and 25 May 1992
should be counted in its favor. We do not agree. The contention does not hold
water. It is well-settled that a court's judgment in a case shall not adversely afect
persons who were not parties thereto.
Isaguirre v. De Lara
G.R. No. 138053, May 31, 2000, 332 SCRA 803
Gonzaga Reyes, J.
FACTS: Alejandro de Lara was the original applicant-claimant for a Miscellaneous
Sales Application over a parcel of land with an area of 2,342 square meters. Upon
his death, Alejandro de Lara was succeeded by his wife-respondent Felicitas de
Lara as claimant. On this lot stands a two-story residential-commercial apartment
declared for taxation purposes in the name of respondents sons, Apolonio and
Rodolfo de Lara. When Felicitas encountered financial difficulties, she approached
petitioner Cornelio M. Isaguirre. On February 10, 1960, a document denominated
as Deed of Sale and Special Cession of Rights and Interests was executed by
Felicitas and Isaguirre, whereby the former sold a 250 square meter portion of the
subject lot, together with the two-story commercial and residential structure
standing thereon. Sometime in May 1969, Apolonio and Rodolfo de Lara filed a
complaint against petitioner for recovery of ownership and possession of the twostory building. However, petitioner filed a sales application over the subject
property and was issued an OCT. Due to overlapping of title, petitioner filed an
action for quieting of title. Judgment was rendered in favor of the respondents.
When respondent filed a motion for execution, petitioner opposed, and alleged that
he had a right of retention over the property until payment of the value of the
improvements he had introduced on the property.
ISSUE: Whether or not petitioner can be considered a builder in good faith with
respect to the improvements he made on the property.
HELD: No. The petitioner is a possessor in bad faith. Based on the factual findings
from this case, it is evident that petitioner knew from the very beginning that there
was really no sale and that he held respondents property as mere security for the
payment of the loan obligation. Therefore, petitioner may claim reimbursement
only for necessary expenses; however, he is not entitled to reimbursement for any
useful expenses which he may have incurred.
Custodio v. Court of Appeals
G.R. No. 116100, February 9, 1996, 253 SCRA 483
Regalado, J.
FACTS: Pacifico Mabasa owns a parcel of land with a two-door apartment. Said
property may be described to be surrounded by other immovables owned by
petitioner Spouses Custodio, Spouses Santos and Rosalina Morato. From the main
street P. Burgos, there are two possible passageways to Mabasas property. One of
the tenants of the apartment vacated because an adobe fence was constructed
thereby making the first passageway narrower in width. Ma. Cristina Santos
testified that she constructed said fence for security reasons. Morato also
constructed her fence and even extended it in such a way that the entire
passageway was enclosed. It was then that the remaining tenants of the
apartment left. Thereafter, Mabasa filed a case for the grant of an easement of
right of way against petitioners. The RTC granted the easement of right of way

22
sought by private respondent. On appeal, the CA affirmed the decision of the RTC
and furthermore, ordering petitioners to pay private respondent a sum of money
for damages.
ISSUE: Whether the award of damages to private respondent is proper.
HELD: No, the act of petitioners in constructing a fence within their lot is a valid
exercise of their right as owners. Article 430 of the Civil Code provides that every
owner may enclose or fence his land or tenements by means of walls, ditches, live
or dead hedges, or by any other means without detriment to servitudes constituted
thereon. The proper exercise of a lawful right cannot constitute a legal wrong for
which an action will lie, although the act may result in damage to another. The
courts can give no redress for hardship to an individual resulting from action
reasonably calculated to achieve a lawful end by lawful means.
Abejaron v. Nabasa
G.R. No. 84831, June 20, 2001, 359 SCRA 47
Puno, J.
FACTS: Petitioner Abejaron avers that he is the actual and lawful possessor and
claimant of a 118-square meter portion of a 175-square meter residential lot in
Silway, General Santos City. Petitioner Abejaron and his family occupied the 118square meter land. At that time, the land had not yet been surveyed. They fenced
the area and built thereon a family home with nipa roofing and a small store.
Petitioner later improved their abode to become a two-storey house. This house,
which stands to this day, occupies a portion of Lot 1, Block 5 and a portion of the
adjoining Lot 2 of the same Psu. Lot 2 belongs to petitioners' daughter, Conchita.
The small store was eventually destroyed and in its stead, petitioner Abejaron
another store. He later planted five coconut trees on the property. Knowing that
the disputed land was public in character, petitioner declared only his house, and
not the disputed land, for taxation purposes. The last two declarations state that
petitioners' house stands on Lots 1 and 2, Block 5. Petitioner stated that
respondent Nabasa resided on the remaining 57-square meter portion of Lot 1.
Nabasa built his house about 4 meters away from petitioner Abejaron's house.
Employees of the Bureau of Lands surveyed the area. Abejaron did not apply for
title of the land on the belief that he could not secure title over it as it was
government property. Without his knowledge and consent, Nabasa applied for and
caused the titling in his name the entire Lot 1, including petitioner Abejaron's 118square meter portion. Nabasa was issued an Original Certificate of Title pursuant to
a Free Patent covering Lot 1. As the title included petitioner Abejarons portion of
the lot, he filed a protest with the Bureau of Lands against Nabasa's title and
application. The protest was dismissed for failure of the petitioner to attend the
hearings. Petitioner Abejaron then filed an action for reconveyance with damages
against respondent Nabasa before the RTC. The RTC The Regional Trial Court ruled
in favor of petitioner in its reconveyance case declaring the possession and
occupancy of Abejaron over 118 square meters of lot in good faith and thereby
declaring the inclusion of said portion in the OCT issued in the name of Nabasa
erroneous. On appeal, the CA reversed the decision of the RTC stating that the only
basis for reconveyance is actual fraud which in this case was failed to be
substantiated by Abejaron. Without proof of irregularity neither in the issuance of
title nor in the proceedings incident thereto nor a claim that fraud intervened in
the issuance of the title, the title would become indefeasible. The petitioner hence
resorts to the Supreme Court.
ISSUE: Whether or not petitioner has acquired title over the disputed land.

23
HELD: An action for reconveyance of a property is the sole remedy of a landowner
whose property has been wrongfully or erroneously registered in another's name
after one year from the date of the decree so long as the property has not passed
to an innocent purchaser for value. The action does not seek to reopen the
registration proceeding and set aside the decree of registration but only purports
to show that the person who secured the registration of the property in controversy
is not the real owner thereof. Fraud is a ground for reconveyance. For an action for
reconveyance based on fraud to prosper, it is essential for the party seeking
reconveyance to prove by clear and convincing evidence his title to the property
and the fact of fraud.
Reconveyance is a remedy granted only to the owner of the property alleged to be
erroneously titled in another's name. In the case at bench, petitioner does not
claim to be the owner of the disputed portion. Admittedly, what he has is only a
"preferential right" to acquire ownership thereof by virtue of his actual possession
since January 1947. Title to alienable public lands can be established through open,
continuous, and exclusive possession for at least 30 years. Not being the owner,
petitioner cannot maintain the present suit. Persons who have not obtained title to
public lands could not question the titles legally issued by the State.
German Management & Services, Inc. v. Court of Appeals
G.R. No. 76216 and 76217, September 14, 1989, 177 SCRA 495
Fernan, J.
FACTS: Spouses Cynthia Cuyegkeng Jose and Manuel Rene Jose, residents of
Pennsylvania, Philadelphia, USA are the owners of a parcel of land situated in Sitio
Inarawan, San Isidro, Antipolo, Rizal, with an area of 232,942 sq. M. The land was
originally registered on 5 August 1948 in the Office of the Register of Deeds Rizal
as OCT 19, pursuant to a Homestead Patent granted by the President of the
Philippines on 27 July 1948. On 26 February 1982, the spouses Jose executed a
special power of attorney authorizing German Management Services to develop
their property into a residential subdivision. Consequently, the German
Management obtained Development Permit 00424 from the Human Settlements
Regulatory Commission for said development. Finding that part of the property was
occupied by Gernale and Villeza and 20 other persons, German Management
advised the occupants to vacate the premises but the latter refused. Nevertheless,
German Management proceeded with the development of the subject property
which included the portions occupied and cultivated by Gernale, et.al. Gernale,
et.al. filed an action for forcible entry against German Management before the MTC
Antipolo, Rizal, alleging that they are mountainside farmers of Sitio Inarawan who
have occupied and tilled their farmholdings some 12 to 15 years prior to the
promulgation of PD 27, and that they were deprived of their property without due
process of law when German Management forcibly removed and destroyed the
barbed wire fence enclosing their farmholdings without notice and bulldozing the
rice, corn, fruit bearing trees and other crops that they planted by means of force,
violence and intimidation The MTC dismissed Gernale et.al.'s complaint for forcible
entry. On appeal, the RTC sustained the dismissal by the MTC. Gernale then filed a
petition for review with the Court of Appeals. Said court gave due course to their
petition and reversed the decisions of the MTC and the RTC. The Appellate Court
held that since Gernale, et.al. were in actual possession of the property at the time
they were forcibly ejected by German Management, they have a right to
commence an action for forcible entry regardless of the legality or illegality of
possession. German Management moved to reconsider but the same was denied
by the Appellate Court. Hence, here is the present recourse.

24
ISSUE: Whether the doctrine of self-help may be availed of when respondents
refused to vacate the premises.
HELD: No. The justification that the drastic action of bulldozing and destroying the
crops of the prior possessor on the basis of the doctrine of self help (enunciated in
Article 429 NCC) is unavailing because the such doctrine can only be exercised at
the time of actual or threatened dispossession, which is absent in the present case.
When possession has already been lost, the owner must resort to judicial process
for the recovery of property. This is clear from Article 536 New Civil Code which
provides that "in no case may possession be acquired through force or intimidation
as long as there is a possessor who objects thereto. He, who believes that he has
an action or right to deprive another of the holding of a thing, must invoke the aid
of the competent court, if the holder should refuse to deliver the thing."
Caisip v. People of the Philippines
G.R. No. L-28716, November 18, 1970, 36 SCRA 17
Concepcion, C. J.
FACTS: Spouses Marcelino Guevarra and Gloria Cabalag cultivated a parcel of
land known as Lot 105-A of Hacienda Palico situated in Nasugbu, Batangas, the
same land used to be tenanted by Cabalags father when he was still alive.
Hacienda Palico is owned by Roxas y Cia, administered by Antonio Chuidian, and
supervised by the overseer, Felix Caisip. Prior to the incident involved, Guevarra
sought recognition as a lawful tenant of Royas y Cia from the Court of Agrarian
Relations but his action was dismissed. Thereafter, Roxas y Cia filed an action
against Guevarra for forcible entry with prayer that Guevarra be ejected from the
premises of Lot 105-A. The Justice of the Peace of Court of Nasugbu decided in
favor of Roxas y Cia and on June 6, 1959, a trouble between Cabalag and Caisip
occurred regarding the cutting of sugarcane.
A day later, Cabalag entered again the premises of Lot 105-A and refused to be
driven out by Caisip. Due to Cabalags tenacious attitude, Caisip sought the help of
the Chief of Police of Nasugbu. The Deputy Sherif, however, informed Caisip that
his request to eject Cabalag cannot be acted upon without a proper court order.
Nevertheless, the Chief of Police assigned Sergeant Ignacio Rojales and Corporal
Frederico Villadelrey to Haciendo Palico. On June 17, 1959, Cabalag was seen
weeding a portion of Lot 105-A which was a ricefield. Caisip approached her and
bade her to leave, but she refused to do so. So, Caisip went to Sgt. Rojales and Cpl.
Villadelrey and brought them to Cabalag. Rojales told Cabalag to stop weeding but
she insisted on her right to stay in the said lot. While in squatting position, Cabalag
was grabbed by Rojales who twisted her right arm and wrested the trowel she was
holding. Villadelrey held her left hand and together Rojales forcibly dragged her
towards a banana plantation while Caisip stood nearby, with a drawn gun. Cabalag
shouted, Ina ko po! Ina ko po! and was heard by some neighbors. Zoilo Rivera,
head of the tenant organization to which Cabalag was affiliated, went with them on
their way to the municipal building. Upon arrival, Cabalag was turned over by
Rojales and Villadelrey to the policemen on duty, who interrogated her. But upon
representations made by Rivera, she was released and allowed to go home.
Cabagan then filed a complaint charging Caisip, Rojales and Villadelrey of the
crime of grave coercion.
The Court of First Instance of Batangas found them guilty as charged. On appeal,
The Court of Appeals affirmed the trial courts decision.

25
ISSUE: Whether or not the force employed by Caisip and others, in the exercise of
his right granted by Article 429, is reasonably necessary to repel or prevent an
actual or threatened unlawful physical invasion or usurpation of his property.
HELD: No. Caisip was not even entitled to the right granted by Article 429. This is
totally inapplicable to the case, for, having been given 20 days from June 6th
within which to vacate the lot, Cabalag did not, on June 17th and within said
period, invades or usurps the said lot. She had merely remained in possession
thereof, even though the hacienda owner may have become its co-possessor by
reason of the prior order of the Justice of Peace Court of Nasugbu. Caisip and
others did not repel or prevent an actual or threatened physical invasion or
usurpation. They expelled Cabalag from a property which she and her husband
were in possession, despite the fact that the Sherif had explicitly authorized
Guevarra and Cabalag to stay in said property up to June 26th, and had expressed
the view that he could not oust them without a judicial order. It is clear, therefore,
that Caisip, Rojales and Villadelrey, by means of violence, and without legal
authority, had prevented the complainant from doing something not prohibited by
law (weeding and being in Lot 105-A), and compelled her to do something against
her will (stopping the weeding and leaving said lot), whether it be right or wrong,
thereby taking the law into their hands, in violation of Article 286 of the Revised
Penal Code.
People of the Philippines v. Pletcha
G.R. No. 19029-CR, June 27, 1977, 22 CA Rep. 807
Sison, J.
FACTS: Tito Pletcha, Jr., farmer, invoking self-help in defense of the land he
inherited from his father 19 years ago against the workers of Radeco Corporation,
who without court order, were constructing a fence in a hacienda allegedly leased
by the corporation from a certain Lopinco.
Claiming actual possession and ownership and believing that the land sought to be
fenced was an integral part of the land he inherited, Pletcha asked the group to
desist from fenicing pending a resurvey he proposed, but he was totally ignored,
thus he fought of and prevented the workers. As a result of such resistance he was
prosecuted and convicted of grave coercion by the Municipal Trial Court. Pletcha
appealed the decision of the MTC with the Court of Appeals.
ISSUE: Whether the appellants action is a legitimate exercise of a private citizens
self-help.
HELD: Yes. In the instant case,the usurpers possession has not yet become
complete and the complainants were in the act of building a fence. Such an act
constitutes force in contemplation of the law. This act of trespass justified the
appellant to drive them away, even by means of bolo because they refused to
listen to his appeal which is reasonable. The appellant need not rush to the court
to seek redress before reasonably resisting the invasion of his property. The
situation required immediate action and Art. 429 gave him the self executory
mechanics of self-defense and self-reliance. The provision in Art 429 of the New
Civil Code confirms the right of the appellant, an owner and lawful possessor, to
use reasonable force to repel an invasion or usurpation, actual, threatened or
physical of his property. The principle of self-defense and the protective measures
related thereto, covers not only his life, but also his liberty and property.
The principle of self-help authorizes the lawful possessor to use force, not only to
prevent a threatened unlawful invasion or usurpation thereof; it is a sort of self-

26
defense. It is lawful to repel force by force. He who merely uses force to defend his
possession does not possess by force. The use of such necessary force to protect
propriety or possessory rights constitutes a justifying circumstance under the Penal
Code.
Andamo v. Intermediate Appellate Court
G.R. No. 74761, November 6, 1990, 191 SCRA 195
Fernan, C.J.
FACTS: Petitioner spouses Emmanuel and Natividad Andamo are the owners of a
parcel of land situated in Biga (Biluso) Silang, Cavite which is adjacent to that of
private respondent, Missionaries of Our Lady of La Salette, Inc., a religious
corporation. Within the land of respondent corporation, waterpaths and
contrivances, including an artificial lake, were constructed, which allegedly
inundated and eroded petitioners' land, caused a young man to drown, damaged
petitioners' crops and plants, washed away costly fences, endangered the lives of
petitioners and their laborers during rainy and stormy seasons, and exposed plants
and other improvements to destruction.
Petitioners instituted a criminal action against the officers and directors of
respondent corporation, for destruction by means of inundation under Article 324
of the Revised Penal Code. Subsequently, petitioners filed a civil action against
respondent corporation for damages. The trial court dismissed the civil case for
lack of jurisdiction, as the criminal case which was instituted ahead of the civil case
was still unresolved. The appellate court affirmed the order of the trial court. The
motion for reconsideration was also denied.
ISSUE: Whether a corporation, which has built through its agents, waterpaths,
water conductors and contrivances within its land, thereby causing inundation and
damage to an adjacent land, can be held civilly liable for damages.
HELD: Yes. Petitioners' complaint sufficiently alleges that petitioners have
sustained and will continue to sustain damage due to the waterpaths and
contrivances built by respondent corporation. It must be stressed that the use of
one's property is not without limitations. Article 431 of the Civil Code provides that
"the owner of a thing cannot make use thereof in such a manner as to injure the
rights of a third person." SIC UTERE TUO UT ALIENUM NON LAEDAS. Moreover,
adjoining landowners have mutual and reciprocal duties which require that each
must use his own land in a reasonable manner so as not to infringe upon the rights
and interests of others. Although we recognize the right of an owner to build
structures on his land, such structures must be so constructed and maintained
using all reasonable care so that they cannot be dangerous to adjoining
landowners and can withstand the usual and expected forces of nature. If the
structures cause injury or damage to an adjoining landowner or a third person, the
latter can claim indemnification for the injury or damage sufered.
Article 2176 of the Civil Code imposes a civil liability on a person for damage
caused by his act or omission constituting fault or negligence.
However,
responsibility for fault or negligence under the said article is entirely separate and
distinct from the civil liability arising from negligence under the Penal Code. The
plaintif cannot recover damages twice for the same act or omission of the
defendant. The decision is reversed and set aside.
Bachrach Motor Co., Inc. v. Talisay Silay Milling Co.
G.R. No. 35223, September 17, 1931, 56 Phil. 117
Romualdez, J.

27
FACTS: On December 22, 1923, the Talisay-Silay Milling Co., Inc., was indebted to
the Philippine National Bank. To secure the payment of its debt, it succeeded in
inducing its planters, among whom, was Mariano Lacson Ledesma, to mortgage
their land to the creditor bank. And in order to compensate those planters for the
risk they were running with their property under the mortgage, the aforesaid
central, by a resolution passed on that same date, i.e., December 22, 1923,
undertook to credit the owners of the plantation thus mortgaged every year with a
sum equal to two per centum of the debt secured according to yearly balance, the
payment of the bonus being made at once, or in part from time to time, as soon as
the central became free of its obligations to the aforesaid bank, and of those
contracted by virtue of the contract of supervision, and had funds which might be
so used, or as soon as it obtained from said bank authority to make such payment.
Bachrach Motor Co., Inc. filed a complaint against the Talisay-Silay Milling Co., Inc.,
for the delivery of the amount P13,850 or promissory notes or other instruments or
credit for that sum payable on June 30, 1930, as bonus in favor of Mariano Lacson
Ledesma.
The Philippine National Bank filed a third party claim alleging a preferential right to
receive any amount which Mariano Lacson Ledesma might be entitled to from the
Talisay-Silay Milling Co. as bonus, because that would be civil fruits of the land
mortgaged to said bank by said debtor for the benefit of the central referred to,
and by virtue of a deed of assignment, and praying that said central be ordered to
delivered directly to the intervening bank said sum on account of the latter's credit
against the aforesaid Mariano Lacson Ledesma.
ISSUE: Whether or not the bonus in question is civil fruits
HELD: No. The said bonus bears no immediate, but only a remote accidental
relation to the land mentioned, having been granted as compensation for the risk
of having subjected one's land to a lien in favor of the bank, for the benefit of the
entity granting said bonus. If this bonus be income or civil fruits of anything, it is
income arising from said risk, or, if one chooses, from Mariano Lacson Ledesma's
generosity in facing the danger for the protection of the central, but certainly it is
not civil fruits or income from the mortgaged property. Hence, the amount of the
bonus, according to the resolution of the central granting it, is not based upon the
value, importance or any other circumstance of the mortgaged property, but upon
the total value of the debt thereby secured, according to the annual balance, which
is something quite distinct from and independent of the property referred to.
Equatorial Realty Development, Inc. v. Mayfair Theater, Inc.
G.R. No. 136221, May 12, 2000, 370 SCRA 56
Pardo, J.
FACTS: Carmelo and Bauermann, Inc. use to own a parcel of land, together with
two 2-storey buildings constructed thereon. Carmelo entered into a Contract of
Lease with Mayfair Theater Inc. for a period of 20 years. The lease covered a
portion a portion of the second floor and mezzanine of a 2-storey building which
respondent used as a movie house known as Maxim Theater. Two years later,
Mayfair entered into a second Contract of Lease with of Carmelo for the lease of
another portion of the latters property namely, part of the second floor of the 2storey building and two store spaces on the ground floor and the mezzanine, on
which Mayfair put up another movie house known as Miramar Theater. The
contract was likewise for a period of 20 years. Both leases contained a provision
granting Mayfair a right of first refusal to purchase the subject properties.

28
However, the subject properties were sold by Carmelo to Equatorial Realty
Development, Inc. without ofering it first to Mayfair. Mayfair filed a Complaint
before the RTC of Manila for the annulment of the Deed of Absolute Sale between
Carmelo and Equatorial. The RTC rendered its decision in favour of Carmelo and
Equatorial. The Court of Appeals completely reversed and set aside the judgment
of the lower court. The Supreme Court denied the petition for review and rescinded
the contract of sale between Carmelo and Equatorial and ordered Carmelo to allow
Mayfair to buy the lots. However, Carmelo could no longer be located. Thus,
following the order of execution of the trial court, Mayfair deposited with the clerk
of court a quo its payment to Carmelo. The lower court issued a Deed of
Reconveyance in favour of Carmelo and a Deed of Sale in favor of Mayfair. Later,
Equatorial filed with the trial court an action for the collection of the sum of money
against Mayfair, claiming payment of rentals or reasonable compensation for the
defendants use of subject premises after its lease contract had expired.
ISSUE: Whether or not Equatorial should be entitled to back rentals.
HELD: No. Rescission creates the obligation to return the things which were the
object of the contract, together with their fruits, and the price with its interest. It is
clear the Equatorial never took actual control and possession of the property sold,
in view of Mayfairs timely objection to the sale and continued actual possession of
the property. Furthermore, the fact that Mayfair paid rentals to Equatorial during
the litigation should not be interpreted to mean actual delivery or ispo facto
recognition of Equatorials title. They were made merely to avoid imminent eviction
and should not be construed as recognition of Equatorial as new owner.
SPS. MACASAET vs SPS. MACASAET
G.R. 154391-92 Sept. 30, 2004
Facts: Petitioners Ismael and Teresita Macasaet and Respondents Vicente and
Rosario Macasaet are first-degree relatives. Ismael is the son of respondents and
Teresita is his wife. On December 10, 1997, the parents filed with the MTC of Lipa
an ejectment suit against the children.
Respondents alleged that they were the owners of 2 parcels of land, situated at
Banay-banay, Lipa City; that by way of a verbal lease agreement, Ismael and
Teresita occupied these lots in Mar. 1992 and used them as their residence and the
situs of their construction business; and that despite repeated demands,
petitioners failed to pay the agreed rental of P500.
Ismael and Teresita denied the existence of any verbal lease agreement. They
claimed that respondents had invited them to construct their residence and
business on the subject lots in order that they could all live near one another,
employ marivic, the sister of Ismael, and help in resolving the problems of the
family. They added that it was the policy of respondents to allot the land they
owned as an advance grant of inheritance in favor of their children.
The MTCC ruled in favor of respondents and ordered petitioners to vacate the
premises. It opined that Ismael and Teresita had occupied the lots, not by virtue of
a verbal lease agreement, but by tolerance of Vicente and Rosario. As their stay
was merely tolerance, petitioners were necessarily bound by an implied promise to
vacate the lots upon demand.
On appeal, the regional trial court updheld the findings fo the MTCC. However, the
RTC allowed the respondents to appropriate the building and other improvements

29
introduced by petitioners, after payment of the indemnity provided for by Art. 448
in relation to Art. 546 and 548 of the NCC.
The CA sustained the finding of the two lower courts that Ismael and Teresita had
been occupying the subject lots only by the tolerance of Vicente and Rosario.
Citing Calubayan v. pascual, the CA further ruled that petitioners status was
analogous to that of a lessee or a tenant whose term of lease had expired, but
whose occupancy continued by tolerance of the owner.
Consequently , in ascertaining the right of the petitioners to be reimbursed for the
improvements they had introduced on respondents properties, the appellate court
applied the Civil Codes provisions on lease.
Issue: Whether the courts should fix the duration of possession.
Held: That Ismael and Teresita had a right to occupy the lots is therefore clear, the
issue is the duration of possession. In the absence of a stipulation on this point,
Art. 1197 of the civil Code allows the courts to fix the duration or the period.
Article 1197. If the obligation does not fix a period, but from its nature and the
circumstances it can be inferred that a period was intended, the courts may fix the
duration thereof. The courts shall also fix the duration of the period when it
depends upon the will of the debtor. Article 1197, however, applies to a situation in
which the parties intended a period. Such qualification cannot be inferred from the
facts of the present case.
The mere failure to fix the duration of their agreement does not necessarily justify
or authorize the courts to do so. It can be safely concluded that the agreement
subsisted as long as the parents and the children mutually benefited from the
arrangement. Efectively, there is a resolutory condition in such an agreement.
Their possession which was originally lawful became unlawful when the reason
therefore love and solidarity ceased to exist between them.
FELICIANO VS. ZALDIVAR
G.R. 162593
FACTS: Feliciano filed against the spouses Aurelio and Luz Zaldivar a complaint for
declaration of nullity of Transfer Certificate of Title and reconveyance of the subject
property in Cagayan de Oro City. The said title is registered in the name of Aurelio
Zaldivar.
In her complaint,Feliciano alleged that she was the registered owner of a parcel of
land covered by a TCT. Sometime in 1974, Aurelio, allegedly through fraud, was
able to obtain a TCT covering the portion of Felicianos lot as described in her TCT.
According to Feliciano, the subject lot was originally leased from her by Pio
Dalman, Aurelios father-in-law. She further alleged that she was going to mortgage
the subject lot to Ignacio Gil which however, did not push through because Gil took
back the money without returning the receipt she had signed as evidence of the
supposed mortgage contract. Thereafter, in 1974, Aurelio filed with the then CFI of
Misamis Oriental a petition for partial cancellation of theTCT in Felicianos name. It
was allegedly made to appear therein that Aurelio and his spouse Luz acquired the
subject lot from Dalman who, in turn, purchased it from Gil. The petition was
granted and a TCT was issued in Aurelios name.

30
Remegia denied that she sold the subject lot either to Gil or Dalman. She likewise
impugned as falsified the joint affidavit of confirmation of sale that she and her
uncle, Narciso Labuntog, purportedly executed before a notary public, where
Remegia appears to have confirmed the sale of the subject property to Gil. She
alleged that she never parted with the certificate of title and that it was never lost.
As proof that the sale of the subject lot never transpired, Remegia pointed out that
the transaction was not annotated on her TCT.
In their answer, the spouses Zaldivar denied the material allegations in the
complaint and raised the affirmative defense that Aurelio is the absolute owner
and possessor of the subject lot as evidenced by his TCT and Tax Declaration
covering the same. Aurelio claimed that he acquired the subject lot by purchase
from Dalman who, in turn, bought the same from Gil on April 4, 1951. Gil allegedly
purchased the subject lot from Remegia and this sale was allegedly conformed and
ratified by the latter and her uncle, Narciso Labuntog, before a notary public on
December 3, 1965.
After Aurelio obtained a loan from the GSIS, the spouses Zaldivar constructed their
house on the subject lot. They alleged that they and their predecessors-in-interest
had been occupying the said property openly, publicly, adversely and continuously
for over 41 years already. Aurelio filed a petition for the issuance of a new owners
duplicate copy of because when he asked Remegia about it, the latter claimed that
it had been lost.
The RTC rendered judgment in favor of Remegia. On appeal, the CA reversed the
decision of the RTC and ruled in favor of the spouses Zaldivar. When their MR was
denied by the CA, the heirs of Feliciano (the petitioners) sought recourse to the
Court in their petition for review.
ISSUE: Whether the CA erred:
1. in ruling that the court who ordered the issuance of new certificate of title
despite existence of owners duplicate copy that was never lost has jurisdiction
over the case.
2. in concluding that the respondents (defendants-appellants) are the absolute
owners of the subject lot based on the TCT issued to them.
3. in concluding that petitioners claim of ownership over the subject lot was barred
by estoppel or laches.
HELD:
1. YES. As the trial court correctly held, the CFI which granted respondent Aurelios
petition for the issuance of a new owners duplicate copy did not acquire
jurisdiction to issue such order. It has been consistently ruled that when the
owners duplicate certificate of title has not been lost, but is in fact in the
possession of another person, then the reconstituted certificate is void, because
the court that rendered the decision had no jurisdiction. Reconstitution can validly
be made only in case of loss of the original certificate. In such a case, the decision
authorizing the issuance of a new owners duplicate certificate of title may be
attacked any time
2. YES. The court a quo correctly nullified the TCT in Aurelios name, emanating as
it did from the new owners duplicate, which Aurelio procured through fraud.
The appellate courts reliance on the joint affidavit of confirmation of sale
purportedly executed by Remegia and her uncle, Narciso Labuntog, is not proper.
In the first place, respondent Aurelio cannot rely on the joint affidavit of
confirmation of sale to prove that they had validly acquired the subject lot
because, by itself, an affidavit is not a mode of acquiring ownership. Moreover, the

31
affidavit is written entirely in English, the hearing revealing that Feliciano does not
understand English.
On this point, Article 1332 of the Civil Code is relevant:
ART.1332. When one of the parties is unable to read, or if the contract is in a
language not understood by him, and mistake or fraud is alleged, the person
enforcing the contract must show that the terms thereof have been fully explained
to the former.
If the person enforcing the contract fails to discharge this burden, the presumption
of mistake, if not, fraud, stands unrebutted and controlling. The bare statement of
Atty. Velez (testified for the Zaldivar spouses) that he read and interpreted the
document to the affiants and that he asked them as to the correctness of its
contents does not necessarily establish that Remegia actually comprehended or
understood the import of the joint affidavit of confirmation of sale
In a long line of cases, the Court has consistently ruled that lands covered by a title
cannot be acquired by prescription or adverse possession. A claim of acquisitive
prescription is baseless when the land involved is a registered land.
Moreover, respondent Aurelio cannot raise the defense of indefeasibility of [his]
title because the principle of indefeasibility of a Torrens title does not apply where
fraud attended the issuance of the title. The Torrens title does not furnish a shield
for fraud. As such, a title issued based on void documents may be annulled.
3. As registered owners of the lots in question, the private respondents have a
right to eject any person illegally occupying their property. This right is
imprescriptible. Even if it be supposed that they were aware of the petitioners
occupation of the property, and regardless of the length of that possession, the
lawful owners have a right to demand the return of their property at any time as
long as the possession was unauthorized or merely tolerated, if at all. This right is
never barred by laches
Nonetheless, the Court is not unmindful of the fact that respondents had built their
house on the subject lot and, despite knowledge thereof, Remegia did not lift a
finger to prevent it. Article 453 of the
Civil Code is applicable to their case:
ART. 453. If there was bad faith, not only on the part of the person who built,
planted or sowed on the land of another, but also on the part of the owner of such
land, the rights of one and the other shall be the same as though both had acted in
good faith.
It is understood that there is bad faith on the part of the landowner whenever the
act was done with his knowledge and without opposition on his part.
Under the circumstances, respondents and Remegia are in mutual bad faith and,
as such, would entitle the former to the application of Article 448 of the Civil Code
governing builders in good faith:
ART. 448. The owner of the land on which anything has been built, sown or planted
in good faith, shall have the right to appropriate as his own the works, sowing or
planting, after payment of the indemnity provided for in Articles 54629 and
548,30 or to oblige the one who built or planted to pay the price of the land, and
the one who sowed, the proper rent. However, the builder or planter cannot be
obliged to buy the land if its value is considerably more than that of the building or
trees. In such a case, he shall pay reasonable rent, if the owner of the land does
not choose to appropriate the building or trees after the proper indemnity. The
parties shall agree upon the terms of the lease and in case of disagreement, the
court shall fix the terms thereof.

32
Following the above provision, the owner of the land on which anything has been
built, sown or planted in good faith shall have the right to appropriate as his own
the building, planting or sowing, after payment to the builder, planter or sower of
the necessary and useful expenses, and in the proper case, expenses for pure
luxury or mere pleasure.
Consequently, the petitioners are obliged to exercise either of the following
options:
(1) to appropriate the improvements, including the house, built by the respondents
on the subject lot by paying the indemnity required by law, or
(2) sell the subject lot to the respondents.
Petitioners cannot refuse to exercise either option and compel respondents to
remove their house from the land. In case petitioners choose to exercise the
second option, respondents are not obliged to purchase the subject lot if its value
is considerably more than the improvements thereon and in which case,
respondents must pay rent to petitioners. If they are unable to agree on the terms
of the lease, the court shall fix the terms thereof.
SPOUSES NUGUID V. CA
G.R. 151815
FACTS: Pedro P. Pecson owned a commercial lot on which he built a four-door twostorey apartment building. For failure to pay realty taxes, the lot was sold at public
auction by the City Treasurer of Quezon City to Mamerto Nepomuceno,
who in turn sold it for P103,000 to the spouses Juan and Erlinda Nuguid,
Pecson challenged the validity of the auction sale before the RTC of Quezon City.
TheRTC upheld the spouses title but declared that the four-door two-storey
apartment building was not included in the auction sale. This was affirmed in toto
by the Court of Appeals and thereafter by this Court.
The Nuguids became the uncontested owners of the commercial lot. The Nuguid
moved for delivery of possession of the lot and the apartment building.
The trial court, ruled that the Spouses Nuguid were to reimburse
Pecson for his construction cost and also directed Pecson to pay monthly rentals
to the Nuguids as paid by the tenants occupying the apartment units and allowed
the ofset of the due from the Nuguids
against
the
amount
of
rents
collected by Pecson from the tenants of the apartment.
The CA affirmed the order of payment of construction costs but rendered the issue
of possession moot on appeal.
Upon review, the SC handed down the decision remanding to the trial court for it to
determine the current market value of the apartment building on the lot which the
Spouses must pay to Pedro Pecson else petitioner shall be restored to the
possession of the apartment building until payment of the required indemnity.
The RTC ordered the spouses to pay the sum ofP1,344,000 as reimbursement of
the unrealized income of Pecson. The Court of Appeals reduced the rentals from
P1,344,000 to P280,000.
ISSUE: Whether or not the petitioners are liable to pay rent over and above the
current market value of the improvement and that such increased award of rentals
by the RTC was reasonable. and equitable.

33
HELD: Under Article 448, the landowner is given the option, either to appropriate
the improvement as his own upon payment of the proper amount of indemnity or
to sell the land to the possessor in good faith. Relatedly, Article 546 provides that a
builder in good faith is entitled to full reimbursement for all the necessary and
useful expenses incurred; it also gives him right of retention until full
reimbursement is made.
While the law aims to concentrate in one person the ownership of the land and the
improvements thereon in view of the impracticability of creating a state of forced
co-ownership, it guards against unjust enrichment insofar as the goodfaith builders improvements are concerned. The right of retention is considered
as one of the measures devised by the law for the protection of builders in good
faith. Its object is to guarantee full and prompt reimbursement as it permits the
actual possessor to remain in possession while he has not been reimbursed (by the
person who defeated him in the case for possession of the property) for those
necessary expenses and useful improvements made by him on the thing
possessed. Accordingly, a builder in good faith cannot be compelled to pay rentals
during the period of retention nor be disturbed in his possession by ordering
him to vacate. In addition, as in this case, the owner of the land is prohibited from
ofsetting or compensating the necessary and useful expenses with the
fruits received by the builder-possessor in good faith. Otherwise, the security
provided by law would be impaired. This is so because the right to the expenses
and the right to the fruits both pertain to the possessor, making compensation
juridically impossible; and one cannot be used to reduce the other.
Since petitioners opted to appropriate the improvement for themselves as early as
June 1993, when they applied for a writ of execution despite knowledge that the
auction sale did not include the apartment building, they could not
benefit from the lots improvement, until they reimbursed the improver in full,
based on the current market value of the property.
Despite the Courts recognition of Pecsons right of ownership over the
apartment building, the petitioners still insisted on dispossessing Pecson by filing
for a Writ of Possession to cover both the lot and the building. Clearly, this resulted
in a violation of respondents right of retention. Worse, petitioners took advantage
of the situation to benefit from the highly valued, income-yielding, four-unit
apartment building by collecting rentals thereon, before they paid for the cost of
the apartment building. It was only four years later that they finally paid its full
value to the respondent.
Given the circumstances of the instant case where the builder in good faith has
been clearly denied his right of retention for almost half a decade, we find that the
increased award of rentals by the RTC was reasonable and equitable. The
petitioners had reaped all the benefits from the improvement introduced by the
respondent during said period, without paying any amount to the latter as
reimbursement for his construction costs and expenses. They should account and
pay for such benefits.
Ignacio v. Hilario
G.R. No. L-175, August 30, 1946, 76 Phil. 605
Moran, C. J.
FACTS: This case concerns the ownership of a parcel of land, partly rice-land and
partly residential. The lower court rendered judgment holding plaintifs as the legal
owners of the whole property but conceding to defendants the ownership of the

34
houses and granaries built by them on the residential portion with the rights of a
possessor in good faith, in accordance with article 361 of the Civil Code.
Subsequently, the plaintifs prayed for an order of execution alleging that since
they chose neither to pay defendants for the buildings nor to sell to them the
residential lot, said defendants should be ordered to remove the structure at their
own expense and to restore plaintifs in the possession of said lot. Defendants
objected to this motion which, after hearing, was granted by Judge Natividad.
Hence, this petition by defendants praying for (a) a restraint and annulment of the
order of execution issued by Judge Natividad; (b) an order to compel plaintifs to
pay them the sum of P2,000 for the buildings, or sell to them the residential lot for
P45; or (c), a rehearing of the case for a determination of the rights of the parties
upon failure of extra-judicial settlement.
ISSUE: Whether the respondent Court erred in its judgment.
HELD: Yes. The Civil Code provides:
ART. 361. The owner of land on which anything has been built, sown or planted in
good faith, shall have the right to appropriate as his own the work, sowing or
planting, after the payment of the indemnity stated in articles 453 and 454, or to
oblige the one who built or planted to pay the price of the land, and the one who
sowed, the proper rent.
ART. 453. Necessary expenses shall be refunded to every possessor; but only the
possessor in good faith may retain the thing until such expenses are made good to
him.
Useful expenses shall be refunded to the possessor in good faith with the same
right of retention, the person who has defeated him in the possession having the
option of refunding the amount of the expenses or paying the increase in value
which the thing may have acquired in consequence thereof.
The owner of the building erected in good faith on a land owned by another, is
entitled to retain the possession of the land until he is paid the value of his
building, under article 453. The owner of the land, upon the other hand, has the
option, under article 361, either to pay for the building or to sell his land to the
owner of the building. But he cannot, as respondents here did, refuse both to pay
for the building and to sell the land and compel the owner of the building to
remove it from the land where it is erected. He is entitled to another motion only
when, after having chosen to sell his land, the other party fails to pay for the same.
The Court holds, therefore, that the order of Judge Natividad compelling
defendants-petitioners to remove their buildings from the land belonging to
plaintifs-respondents only because the latter chose neither to pay for such
buildings not to sell the land, is null and void, for it amends substantially the
judgment sought to be executed and is, furthermore, ofensive to articles 361 and
453 of the Civil Code.
Filipinas Colleges, Inc. v. Garcia Timbang, et. al.,
G.R. No. L-12812, September 29, 1989, 164 SCRA 287
Barrera, J.
FACTS: After appropriate proceedings, the Court of Appeals held, among other
things, that Filipinas Colleges, Inc. are declared to have acquired the rights of the
spouses Timbang in the questioned lots, they are ordered to pay the spouses

35
Timbang in the amount of P15,807.90 plus such other amount which said spouses
might have paid or had to pay. On the other hand, Maria Gervacio Blas was also
declared to be a builder in good faith of the school building constructed in the lot in
question and was entitled to be paid the amount of P19,000.00 for the same. Also,
in case that Filipinas Colleges, Inc. failed to deposit the value of the land, which
after liquidation was fixed at P32,859.34, within the 90-day period set by the Court,
Filipinas Colleges would lose all its rights to the land and the spouses Timbang
would then become the owners thereof. If that is the case, the Timbangs are
ordered to make known to the court their option under Article 448 of the Civil Code
whether they would appropriate the building in question, in which even they would
have to pay Filipinas Colleges, Inc. the sum of P19,000.00, or would compel the
latter to acquire the land and pay the price thereof. Filipinas Colleges, Inc. failed to
pay the sum of P32,859.34 so the spouses Timbang made known to the court their
decision that they had chosen not to appropriate the building but to compel
Filipinas Colleges, Inc., for the payment of the sum of P32,859,34 which was
granted by the Court. As a consequence of which, a writ of execution was issued.
Meanwhile, Blas filed a motion for execution of her judgment representing the
unpaid portion of the price of the house sold to Filipinas which was granted. Levy
was made on the house in virtue of the writs of execution. Then, the Sherif of
Manila sold the building in public auction in favor of the spouses Timbang, as the
highest bidders. Several motion were the subsequently filed before the lower court
wherein the court held that: a) the Sherif's certificate of sale covering a school
building sold at public auction was null and void unless within 15 days from notice
of said order spouses Timbang shall pay to Blas the sum of P5,750.00 that the
spouses Timbang had bid for the building at the Sherif's sale; b) that Filipinas is
owner of 245.00/32,859.34 undivided interest in Lot No. 2-a on which the building
sold in the auction sale is situated; and c) that the undivided interest of the
Filipinas in the lot should be sold to satisfy the unpaid portion of the judgment in
favor of Blas and against Filipinas in the amount of P8,200.00 minus the sum of
P5,750.00. The spouses Timbang contends that because the builder in good faith
has failed to pay the price of the land after the owners thereof exercised their
option under Article 448 of the Civil Code, the builder lost his right of retention
provided in Article 546 and that by operation of Article 445, the spouses Timbang
as owners of the land automatically became the owners ipso facto of the school
building.
ISSUE: Whether or not the spouses Timbang automatically become the owners of
the building upon failure of Filipinas to pay the value of the land.
HELD: No. Based on Article 448 and 546 of the New Civil Code, the owner of the
land has the right to choose between appropriating the building by reimbursing the
builder of the value thereof or compelling the builder in good faith to pay for his
land. Even this second right cannot be exercised if the value of the land is
considerably more than that of the building. In addition to the right of the builder
to be paid the value of his improvement, Article 546 gives him the corollary right of
retention of the property until he is indemnified by the owner of the land. There is
nothing in the language of these two articles, 448 and 546, which would justify the
conclusion of appellants that, upon the failure of the builder to pay the value of the
land, when such is demanded by the land-owner, the latter becomes automatically
the owner of the improvement under Article 445. The case of Bataclan vs Bernardo
cannot be applied in this case in the sense that although it is true it was declared
therein that in the event of the failure of the builder to pay the land after the
owner thereof has chosen this alternative, the builder's right of retention provided
in Article 546 is lost, nevertheless there was nothing said that as a consequence
thereof, the builder loses entirely all rights over his own building. Also, in the
present case, the Court of Appeals has already adjudged that appellee Blas is
entitled to the payment of the unpaid balance of the purchase price of the school

36
building. Blas is actually a lien on the school building are concerned. The order of
the lower court directing the Timbang spouses, as successful bidders, to pay in
cash the amount of their bid in the sum of P5,750.00 is therefore correct.
Manotok Realty v. Tecson
G.R. No. L-47475 August 19, 1988, 164 SCRA 287
Gutierrez Jr., J.
FACTS: Petitioner Manotok Realty filed a complaint against Nilo Madlangawa for
recovery of possession with damages with the Court of First Instance of Manila.
Said court rendered judgment declaring Madlangawa as a builder-possessor in
good faith; ordering the company to recognize the right of Madlangawa to remain
in Lot 345, Block 1, of the Clara Tambunting Subdivision until after he shall have
been reimbursed by the company the sum of P7,500.00, without pronouncement
as to costs.
Not satisfied with the trial courts decision, petitioner appealed to the Court of
Appeals and upon affirming the trial courts decision, it elevated the case to the
Supreme Court. On July 13, 1977, the Supreme Court issued a resolution denying
Manotoks petition for lack of merit. Petitioner then filed with the trial court (Judge
Jose H. Tecson), a motion for the approval of the companys exercise of option and
for satisfaction of judgment. However, Judge Tecson denied the motion for
approval. Hence, this petition is filed.
ISSUE: Whether or not respondent Judge Tecson can deny petitioners (landowner)
motion to avail of its option.
HELD: No. There is, therefore, no basis for the respondent judge to deny the
petitioners motion to avail of its option to appropriate the improvements made on
its property. Neither can the judge deny the issuance of a writ of execution
because the private respondent was adjudged a builder in good faith or on the
ground of peculiar circumstances which supervened after the institution of this
case, like, for instance, the introduction of certain major repairs of and other
substantial improvements because the option given by law belongs to the
owner of the land. Under Article 448 of the Civil Code, the right to appropriate the
works or improvements or to oblige the one who built or planted to pay the proper
price of the land belongs to the owner of the land. The only right given to the
builder in good faith is the right of reimbursement of necessary expenses for the
preservation of the land; the builder cannot compel the landowner to sell such land
to the former.
Bernardo v. Bataclan
G.R. No. L-44606, November 28, 1938, 66 Phil. 598
Laurel, J.
FACTS: Bernardo bought a parcel of land from Samonte which was located in
Cavite. In order that he may take possession and occupy the said land, he filed a
case in the CFI for such purpose and the court rendered a favorable decision for
Bernardo. However, when he was supposedly set in occupying the said land, he
found Bataclan. He was within the premises because he was authorized by the
previous owners to clear the land and make the necessary improvements he
deems fit, further claiming that such authorization was granted to him ever since
1922. Since Bataclan was not a party in the first case, Bernardo filed against him a
separate case. Bernardo was declared owner but the defendant was held to be a
possessor in good faith for whom the work done and improvements made by him

37
should be reimbursed. An appeal to the decision of the court was filed by both
Bernardo and Bataclan. The decision was modified by lowering the price of the
land from P300 to P200 per hectare. Bernardo was given 30 days to exercise his
option, whether to sell the land to Bataclan or to buy the improvements from him.
Bernardo chose the option which would require Bataclan to pay him the value of
the land at the rate of P200 per hectare. However, Bataclan informed the court
that he will not be able to pay for the price of the land. The court then gave
Bataclan 30 days to pay the price of the property and after the lapse of the period,
the land shall be sold in a public auction. After 30 days, the land was sold to
Teodoro at a public auction, after failure of Bataclan to pay within the period the
purchase price.
ISSUE: Whether or not Bataclan has the right of retention over the parcel of land
in question.
HELD: No. Bataclan no longer has lost the right of retention. The option of the
owner was already exercised where he decided that he will just allow the
defendant to purchase the land such that Bataclan was to comply with the option if
he wants to retain the land. From the moment that he told the courts of his
inability to pay for the price of the land, he already lost his right to retain the land.
Heirs of Ramon Durano, Sr. v. Uy
G.R. No. 136456 October 24, 2000, 344 SCRA 238
Gonzaga Reyes, J.
FACTS: Respondents stated that sometime in August 1970 and months thereafter
they received mimeographed notices dated August 2, 1970 and signed by the late
Ramon Durano, Sr., informing them that the lands which they are tilling and
residing in, formerly owned by the Cebu Portland Cement Company (hereafter,
Cepoc), had been purchased by Durano & Co., Inc. The notices also declared that
the lands were needed by Durano & Co. for planting to sugar and for roads or
residences, and directed respondents to immediately turn over the said lands to
the representatives of the company. Simultaneously, tall bamboo poles with
pennants at the tops thereof were planted in some areas of the lands and metal
sheets bearing the initials RMD were nailed to posts.
As early as the first week of August 1970, and even before many of the
respondents received notices to vacate, men who identified themselves as
employees of Durano & Co. proceeded to bulldoze the lands occupied by various
respondents, destroying in their wake the plantings and improvements made by
the respondents therein. On September 15, 1970, Durano & Co. sold the disputed
property to petitioner Ramon Durano III, who procured the registration of these
lands in his name under TCT No. T-103 and TCT No. T-104.
Respondents contended that the display of force and the known power and
prestige of petitioners and their family restrained them from directly resisting this
wanton depredation upon their property. Respondents urged the Department of
Justice to conduct the preliminary investigation. The RTC found that the case
preponderated in favor of respondents, who all possessed their respective portions
of the property covered by TCT Nos. T-103 and T-104 thinking that they were the
absolute owners thereof. A number of these respondents alleged that they
inherited these properties from their parents, who in turn inherited them from their
own parents. Some others came into the properties by purchase from the former
occupants thereof. They and their predecessors were responsible for the plantings
and improvements on the property. They were the ones who sought for the
properties to be tax-declared in their respective names, and they continually paid

38
the taxes thereto. Respondents maintained that they were unaware of anyone
claiming adverse possession or ownership of these lands until the bulldozing
operations in 1970.
Dissatisfied, petitioners appealed the RTC decision to the Court of Appeals, which,
in turn, affirmed the said decision and ordered the return of the property to all the
respondents-claimants.
ISSUE: Whether or not the Court of Appeals erred in its decision ordering the
petitioners to return the properties to the respondents.
HELD: No. The evidence shows that respondents successfully complied with all the
requirements for acquisitive prescription to set in. The properties were conveyed to
respondents by purchase or inheritance, and in each case the respondents were in
actual, continuous, open and adverse possession of the properties. They exercised
rights of ownership over the lands, including the regular payment of taxes and
introduction of plantings and improvements. They were unaware of anyone
claiming to be the owner of these lands other than themselves until the notices of
demolition in 1970 --- and at the time each of them had already completed the tenyear prescriptive period either by their own possession or by obtaining from the
possession of their predecessors-in-interest.
Furthermore, a purchaser of a parcel of land cannot close his eyes to facts which
should put a reasonable man upon his guard, such as when the property subject of
the purchase is in the possession of persons other than the seller. A buyer who
could not have failed to know or discover that the land sold to him was in the
adverse possession of another is a buyer in bad faith. In the case, respondents
were in open possession and occupancy of the properties when Durano & Co.
supposedly purchased the same from Cepoc. Petitioners made no attempt to
investigate the nature of respondents possession before they ordered demolition
in August 1970.
In the same manner, the purchase of the property by petitioner Ramon Durano III
from Durano & Co. could not be said to have been in good faith. It is not disputed
that Durano III acquired the property with full knowledge of respondents
occupancy thereon. There even appears to be undue haste in the conveyance of
the property to Durano III, as the bulldozing operations by Durano & Co. were still
underway when the deed of sale to Durano III was executed on September 15,
1970. There is not even an indication that Durano & Co. attempted to transfer
registration of the property in its name before it conveyed the same to Durano III.
Since petitioners knew fully well the defect in their titles, they were correctly held
by the Court of Appeals to be builders in bad faith.
The Civil Code provides:
Art. 449. He who builds, plants or sows in bad faith on the land of another, loses
what is built, planted or sown without right of indemnity.
Art. 450. The owner of the land on which anything has been built, planted or sown
in bad faith may demand the demolition of the work, or that the planting or sowing
be removed, in order to replace things in their former condition at the expense of
the person who built, planted or sowed; or he may compel the builder or planter to
pay the price of the land, and the sower the proper rent.
Art. 451. In the cases of the two preceding articles, the landowner is entitled to
damages from the builder, planter or sower.

39
Based on these provisions, the owner of the land has three alternative rights: (1) to
appropriate what has been built without any obligation to pay indemnity therefor,
or (2) to demand that the builder remove what he had built, or (3) to compel the
builder to pay the value of the land. In any case, the landowner is entitled to
damages under Article 451.
The Court sustains the return of the properties to respondents and the payment of
indemnity as being in accord with the reliefs under the Civil Code.
Ballatan v. Court of Appeals
G.R. No. 125683, March 2, 1999, 304 SCRA 34
Puno, J.
FACTS: Ballatan, Martinez and Ling are the owners of adjacent lots in Malabon,
Metro Manila. Lot No. 24, 414 square meters in area, is registered in the name of
petitioners Eden Ballatan and spouses Betty Martinez and Chong Chy Ling. Lots
Nos. 25 and 26 are registered in the name of respondent Gonzalo Go, Sr. On Lot
No. 25, respondent Winston Go, son of Gonzalo Go, Sr., constructed his house.
Adjacent to Lot No. 26 is Lot No. 27, registered in the name of respondent Li Ching
Yao. In 1985, petitioner Ballatan constructed her house on Lot No. 24. During the
construction, she noticed that the concrete fence and side pathway of the
adjoining house of respondent Winston Go encroached on the entire length of the
eastern side of her property. Her building contractor informed her that the area of
her lot was actually less than that described in the title. Forthwith, Ballatan
informed respondent Go of this discrepancy and his encroachment on her property.
Go, however, claimed that his house, including its fence and pathway, were built
within the parameters of his father's lot; and that this lot was surveyed by
Engineer Jose Quedding, the authorized surveyor of the Araneta Institute of
Agriculture (AIA), the owner-developer of the subdivision project. So Ballatan called
the attention of the IAI and after another survey of the land, Engineer Quedding
found that the lot area of petitioner Ballatan was less by few meters and that of
respondent Li Ching Yao, which was three lots away, increased by two meters.
Engineer Quedding declared that he made a verification survey of Lots Nos. 25 and
26 of respondents Go in 1983 and allegedly found the boundaries to have been in
their proper position. He, however, could not explain the reduction in Ballatan's
area since he was not present at the time respondents Go constructed their
boundary walls.
On 10 June 1985, petitioner Ballatan made a written demand on respondents Go to
remove and dismantle their improvements on Lot No. 24 but Go refused. So
Ballatan instituted against Go a civil case for recovery of possession the RTC of
Malabon decided in favor of Ballatan, ordering the Go's to vacate the subject
portion of Lot No. 24, demolish their improvements and pay petitioner Ballatan
actual damages, attorney's fees and the costs of the suit. Go appealed.
ISSUE: Whether or not Ballatan have a right of remotion.
HELD: All the parties have acted in good faith so Article 448 must apply.
Petitioners are ordered to exercise within thirty (30) days from finality of the
decision their option to either buy the portion of respondents Go's improvement on
their Lot No. 24, or sell to said respondents the portion of their land on which the
improvement stands. If petitioners elect to sell the land or buy the improvement,
the purchase price must be at the prevailing market price at the time of payment.
If buying the improvement will render respondents Go's house useless, then
petitioners should sell the encroached portion of their land to respondents Go. If

40
petitioners choose to sell the land but respondents Go are unwilling or unable to
buy, then the latter must vacate the subject portion and pay reasonable rent from
the time petitioners made their choice up to the time they actually vacate the
premises. But if the value of the land is considerably more than the value of the
improvement, then respondents Go may elect to lease the land, in which case the
parties shall agree upon the terms, the lease. Should they fail to agree on said
terms, the court of origin is directed to fix the terms of the lease.
Spouses Del Ocampo v. Abesia
G.R. No. L-49219, April 15, 1998, 160 SCRA 379
Gancayco, J.
FACTS: Plaintifs spouses Concepcion Fernandez and Estanislao Del Campo and
defendant Bernarda Fernandez Abesia are co-owners of parcel of land with an area
of 45 square meters and divided in the proportion of 2/3 and 1/3 share each,
respectively. A commissioner, who is appointed by the court, conducted a survey
and recommended that the property be divided into two lots: Lot 1161 A with an
area of 30 square meters for the plaintifs and Lot 1161 B with an area of 15
square meters for the defendants. However, it was shown in the sketch plan that
the house of the defendant occupied the portion with an area of 5 square meters of
Lot 1161 A of plaintifs. The parties asked the court to finally settle and
adjudicate who among the parties should take possession of the 5 square meters
of land.
ISSUES:
1.) Whether or not Article 448 of the Civil Code, the rights of a builder in good faith,
should be applied to the plaintif-spouses Del Campo.
2.) Whether or not the house of the defendant Abesia should be removed and
demolished at their expense.
HELD: 1.) Yes. Article 448 of the Civil Code cannot apply where a co-owner builds,
plants or sows on the land owned in common for then, he did not build, plant or
sow upon land that exclusively belongs to another but of which he is a co-owner.
The co-owner is not a third person under the circumstances, and the situation is
governed by the rules of co-ownership. However, when, as in this case, the coownership is terminated by the partition and it appears that the house of
defendants overlaps or occupies a portion of 5 square meters of the land
pertaining to plaintifs which the defendants obviously built in good faith, then the
provisions of Article 448 of the new Civil Code should apply. Manresa and Navarro
Amandi agree that the said provision of the Civil Code may apply even when there
was co-ownership if good faith has been established.
2.) It depends. Applying Article 448 of the Civil Code, the plaintifs have the right to
appropriate said portion of the house of defendants upon payment of indemnity to
defendants as provided for in Article 546 of the Civil Code. Otherwise, the plaintifs
may oblige the defendants to pay the price of the land occupied by their house.
However, if the price asked for is considerably much more than the value of the
portion of the house of defendants built thereon, then the latter cannot be obliged
to buy the land. The defendant shall then pay the reasonable rent to the plaintifs
upon such terms and conditions that they may agree. In case of disagreement, the
trial court shall fix the terms thereof. Of course, defendants may demolish or
remove the said portion of their house, at their own expense, if they so decide.
Pacific Farms Inc. v. Esguerra
G.R. No. L-21783, November 29, 1969, 30 SCRA 684

41
Castro, J.
FACTS: On October 1, 1956 to March 2, 1957 the Company sold and delivered
lumber and construction materials to the Insular Farms Inc. which the latter used in
the construction of the si buildings at its compound in Bolinao, Pangasinan, of the
total procurement price of P15,000.00, the sum of P4,710.18 has not been paid.
Consequently, the Company instituted a civil case to recover the unpaid balance
and the court sustained their claim. The defendant sherif levied th six buildings.
The Pacific Farms, Inc. filed a suit against the Company and the sherif asserting
ownership over the levied buildings which it had acquired from the Insular Farms
by virtue of absolute sale executed on March 21, 1958. Pacific prays that the
judicial sale of the six buildings be declared null and void. The trial court rendered
judgment annulling the levy and the certificate of sale. However, it denied the
plaintif's claim for actual and exemplary damages on the ground that it was not
"prepared to find there was gross negligence or bad faith on the part of any
defendants".
ISSUE: Whether or not the application by analogy of the rules of accession would
suffice for a just adjudication.
HELD: Article 447 of the Civil Code contemplates a principal and an accessory; the
land being considered the principal, and the plantings, constructions or works, the
accessory. The owner of the land who in good faith - whether personally or through
another - makes constructions or works thereon, using materials belonging to
somebody else, becomes the owner of the said materials with the obligation
however of paying for their value. On the other hand, the owner of the materials is
entitled to remove them, provided no substantial injury is caused to the landowner.
Otherwise, he has the right to reimbursement for the value of his materials,
Applying article 447 by analogy, the Court consider the buildings as the principal
and the lumber and construction materials that went into their construction as the
accessory. Thus the appellee, if it does own the six buildings, must bear the
obligation to pay for the values of the said materials; the appellant which
apparently has no desire to remove the materials, and, even if it were minded to
do so, cannot remove them without necessarily damaging the buildings has the
corresponding right to recover the value of the unpaid lumber and construction
materials.
Pecson v. Court of Appeals
G.R. No. 115814, May 26, 1995, 244 SCRA 407
Davide, Jr. J.
FACTS: Pedro Pecson owned a commercial lot situated in Kamias street, Quezon
City, on which he built a a four-door, two-storey apartment building. But because of
failure to pay realty taxes amounting to P12,000.00, the commercial lot owned was
sold at a public auction. It was purchased by Nepomuceno, which later sold the
same to the Nuguid spouses for P103,000 on October 12, 1983. Pecson then
challenged the sale, alleging that the apartment building, contrary to the claim of
the Nuguid spouses, was not included in the sale. The lower court judged in favor
of Pecson, declaring that the apartment building was indeed not included in the
subject sale. The Court of Appeals affirmed the same. The Spouses Nuguid then
filed a motion for delivery of possession of the lot and the apartment building. The
lower court ruled in favor of the private respondents, but subject to the
reimbursement to Pecson of the cost of constructing the apartment building minus
the rents due to the spouses (calculated at P21,000 from June 23, 1993 to
September 23, 1993). With the said decision at hand, the spouses then made a

42
move to eject Pecson and as well as the tenants residing therein. However, the
spouses have yet to pay Pecson for the construction costs.
ISSUE: Whether the Nuguid Spouses can eject Pecson even if reimbursement
hasnt been given for the construction costs.
HELD: No. The Court ruled that since the spouses still havent reimbursed Pecson
for the cost of construction of the building, the latter has the right to retain the
property, and along with it, the fruits of which during such possession.
The court ruled that though Article 448 do not apply in the case at bar. By its clear
language, Article 448 refers to a land whose ownership is claimed by two or more
parties, one of whom has built some works, or sown or planted something. The
building, sowing or planting may have been made in good faith or in bad faith. As
in this case, since the owner himself was the one who constructed the
improvement, good faith and bad faith becomes irrelevant. However, by analogy,
the indemnity may be applied, considering that the primary intent of Article 448 is
to avoid a state of forced co-ownership and that the parties agree that Articles 448
and 546 of the Civil Code are applicable and indemnity for the improvements may
be paid, although they difer as to the basis of the indemnity. Since the spouses
have opted to appropriate the apartment building, Pecson is thus entitled to the
possession and enjoyment of the apartment building, until he is paid the proper
indemnity, as well as of the portion of the lot where the building has been
constructed. This is so because the right to retain the improvements while the
corresponding indemnity is not paid implies the tenancy or possession in fact of
the land on which it is built, planted or sown. The petitioner not having been so
paid, he was entitled to retain ownership of the building and, necessarily, the
income therefrom.
Mercy Vda. De Roxas v. Our Lady's Foundation Inc.
G.R. No. 182378
FACTS: On 1 September 1988, Salve Dealca Latosa filed before the RTC a
Complaint for the recovery of ownership of a portion of her residential land located
at Our Ladys Village, Bibincahan, Sorsogon, Sorsogon, docketed as Civil Case No.
5403. According to her, Atty. Henry Amado Roxas (Roxas), represented by
petitioner herein, encroached on a quarter of her property by arbitrarily extending
his concrete fence beyond the correct limits.
In his Answer, Roxas imputed the blame to respondent Our Ladys Village
Foundation, Inc., now Our Ladys Foundation, Inc. (OLFI). He then filed a Third-Party
Complaint against respondent and claimed that he only occupied the adjoining
portion in order to get the equivalent area of what he had lost when OLFI trimmed
his property for the subdivision road. The RTC admitted the Third-Party Complaint
and proceeded to trial on the merits.
ISSUE: The determination of the amount to be reimbursed by respondent
foundation to Roxas.
HELD: To settle the contention of respondent foundation, the Court resorted to Art.
450 of the NCC referring to encroachments in bad faith. The owner of the land
encroached upon petitioner has the option to require respondent builder to pay the
price of land. The court cites Tuatis v. Sps. Escol which illustrates that the present
and current fair value of the land is to be reckoned at the time the land owner
elected the choice and not at the time the property was purchased.

43
Technogas Philippines Manufacturing Corporation v. Court of Appeals
G.R. No. 108894, February, 10, 1997, 268 SCRA 5
Panganiban, J.
FACTS: Technogas purchased a parcel of land from Pariz Industries, Inc. In the
same year, Eduardo Uy purchased the land adjacent to it. The following year, Uy
bought another lot adjoining the lot of Technogas. Portions of the buildings and
wall bought by Technogas together with the land from Pariz Industries are
occupying a portion of Uys adjoining land. The knowledge of some encroachment
was only made known to both parties after their parties of their respective parcels
of land.
ISSUES:
1.) Whether or not petitioner Technogas Philippines is a possessor in bad faith.
2.) Whether or not petitioner Technogas Philippines has stepped into the shoes of
the seller.
HELD: 1.) No. Unless one is versed in the science of surveying, no one can
determine the precise extent or location of his property by merely examining his
paper title. There is no question in that when Technogas purchased the land from
Pariz Industries, the buildings and other structures were already in existence.
Furthermore, it is not clear as to who actually built these structures but it can be
assumed that the predecessor-in-interest of Technogas, Pariz Industries, did so. An
article 527 of the New Civil Code presumes good faith. Since no proof exists to
show that the builder built the encroaching structures in bad faith, the structures
should be presumed to have been built in good faith. Good faith consists in the
belief of the builder that the land he is building on is his, and his ignorance of any
defect or flaw in his title. Furthermore, possession acquired in good faith does not
lose this character except in case and from the moment facts exist which show
that the possessor is not aware that he possesses the thing improperly or
wrongfully. The good faith ceases from the moment the defects in the title are
made known to the possessor, by extraneous evidence or by suit for recovery of
the property of the true owner.
2.) Yes. Has been shown, contrary as to the good faith of Technogas has not been
overthrown. Similarly, upon delivery of the property to Pariz Industries, as seller, to
Technogas, as buyer, the latter acquired ownership of the property. Consequently,
Technogas is deemed to have stepped into the shoes of the seller with regard to all
the rights of ownership of the property over the immovable sold, including the
right to compel Uy to exercise either of the two options under Article 448 of the
New Civil Code. Thus, the landowners exercise of his option can only take place
after the builder shall have to know the intrusion in short, when both parties shall
have become aware of it. Only then will the occasion for exercising the option
arise, for it is only then that both parties will have been aware that a problem
exists with regard to their property rights.
Pleasantville Development Corporation v. Court of Appeals
G.R. No. 79688, February 1, 1996,
Panganiban, J.
FACTS: On March 26, 1974, Wilson Kee on installment Lot 8 from C.T. Torres
Enterprises Inc. the exclusive real estate agent of petitioner. Under the Contract to
Sell on installment. Kee can exercise possession over the parcel of land even
before the completion of installment payments. On January 20, 1975, Kee paid
CTTEI relocation fee of Php 50.00 and another on January 27, 1975 for the

44
preparation of lot plan. These amounts were paid by Kee before he took possession
of Lot 8. After the preparation of the lot plan and a copy was presented to Kee,
Zenaida Octaviano, employee of CTTEI accompanied Donnabelle Kee the wife of
Wilson Kee to inspect Lot 8. Unfortuantely, Octaviano pointed Lot 9. Thereafter,
Kee constructed his residence on the said Lot 9 together a store, repair shop and
other improvements.
Edith Robillo purchased from Pleasantville Development Corporation Lot 9.
Sometime in 1975, she sold the said parcel of land, Lot 9, to Eldred Jardinico which
at that time is vacant. Upon paying completely to Robillo, Jardinico secured from
the Register of Deeds of Bacolod City on December 19, 1978 Transfer Certificate of
Title No. 106367 in his name. It was only that time that he discovered that Wilson
Kee take possession of that lot and that the same have introduced improvements
to the same lot. Jardinico confronted Kee and tried to reach for an amicable
settlement, but failed.
On January 30, 1981, Jardinico, through his lawyer, demanded that Kee vacate Lot
9 and remove all the improvements introduced by the latter. Kee refused which
made Jardinico filed with the Municipal Trial Court in Cities, Branch 3, Bacolod City
a complaint for ejectment with damages against Kee. Kee, in turn filed a third-party
complaint against Pleasantville Development Corporation and CTTEI.
The MTCC held that the erroneous delivery was attributable to CTTEI and the Kee
has no rights to Lot 9 because of the rescission made by CTTEI of their contract
due to Kees failure to pay the installment. MTCC also held that Kee must pay
reasonable rental for the use of Lot 9 and furthermore he cannot claim
reimbursement for the improvements introduced by him. On appeal, the Regional
Trial Court held that Pleasantville and CTTEI were not negligent and that Kee was in
bad faith.
Kee appealed directly to the Supreme Court which referred the matter to the Court
of Appeals. The Appellate Court overturned the ruling of the RTC and held the Kee
was a builder in good faith and the erroneous delivery was attributable to the
negligence of CTTEI. Hence the instant petition filed by Pleasantville.
ISSUES:
1.) Whether or not, Wilson Kee is a builder in good faith.
2.) Whether or not petitioner is liable for the acts of its agent CTTEI.
HELD: 1.) Petitioner fails to persuade the Court to abandon the findings and
conclusions of the Court of Appeals that Kee was a builder in good faith. Good faith
consists in the belief of the builder that the land he is building on is his and his
ignorance of any defect or flaw in his title. And as good faith is presumed,
petitioner has the burden of proving bad faith on the part of Kee. At the time he
built improvements on Lot 8, Kee believed that said lot was what he bought from
petitioner. He was not aware that the lot delivered to him was not Lot 8. Thus, Kee
is in good faith. Petitioner failed to prove otherwise.
To demonstrate Kee's bad faith, petitioner points to Kee's violation of paragraphs
22 and 26 of the Contract of Sale on Installment. It has no merit. Such violations
have no bearing whatsoever on whether Kee was a builder in good faith, that is, on
his state of mind at the time he built the improvements on Lot 9. These alleged
violations may give rise to petitioner's cause of action against Kee under the said
contract (contractual breach), but may not be the basis to negate the presumption
that Kee was a builder in good faith.
2.) Yes. The rule is that the principal is responsible for the acts of the agent done

45
within the scope of his authority, and should bear the damage caused to third
persons. On the other hand, the agent who exceeds his authority is personally
liable for the damage. But CTTEI was acting within its authority as the sole real
estate representative of petitioner when it made the delivery to Kee, only that in so
acting, it was negligent. It is this negligence that is the basis of petitioner's liability,
as principal of CTTEI, per Articles 1909 and 1910 of the Civil Code. For such
negligence, the petitioner should be held liable for damages. The rights of Kee and
Jardinico vis-a-vis each other, as builder in good faith and owner in good faith,
respectively, are regulated by law (i.e., Arts. 448, 546 and 548 of the Civil Code). It
was error for the Court of Appeals to make a "slight modification" in the application
of such law [by holding petitioner and CTTEI solidarily liable], on the ground of
"equity".
Germiniano v. Court of Appeals
G.R. No. 120303, July 24, 1996, 259 SCRA 344
Davide, Jr., J.
FACTS: This is a petition for review on certiorari which has its origins in Civil Case
No. 9214 of Branch 3 of the Municipal Trial Court in Cities (MTCC) in Dagupan City
for unlawful detainer and damages. During the pre-trial conference, the parties
agreed to confine the issues to: (1) whether there was an implied renewal of the
lease which expired in November 1985; (2) whether the lessees were builders in
good faith and entitled to reimbursement of the value of the house and
improvements; and (3) the value of the house.
On the first issue, the court held that since the petitioners' mother was no longer
the owner of the lot in question at the time the lease contract was executed in
1978, in view of its acquisition by Maria Lee as early as 1972, there was no lease to
speak of, much less, a renewal thereof. And even if the lease legally existed, its
implied renewal was not for the period stipulated in the original contract, but only
on a month-to-month basis pursuant to Article 1687 of the Civil Code. The refusal
of the petitioners' mother to accept the rentals starting January 1986 was then a
clear indication of her desire to terminate the monthly lease. As regard the
petitioners' alleged failed promise to sell to the private respondents the lot
occupied by the house, the court held that such should be litigated in a proper
case before the proper forum, not an ejectment case where the only issue was
physical possession of the property.
The court resolved the second issue in the negative, holding that Articles 448 and
546 of the Civil Code, which allow possessors in good faith to recover the value of
improvements and retain the premises until reimbursed, did not apply to lessees
like the private respondents, because the latter knew that their occupation of the
premises would continue only during the life of the lease. Besides, the rights of the
private respondents were specifically governed by Article 1678, which allow
reimbursement of up to one-half of the value of the useful improvements, or
removal of the improvements should the lessor refuse to reimburse.
On the third issue, the court deemed as conclusive the private respondents'
allegation that the value of the house and improvements was P180,000.00, there
being no controverting evidence presented.
On appeal by the private respondents, the RTC of Dagupan City reversed the trial
court's decision.
ISSUE: Whether or not Article 448 or Article 1678 of the Civil Code should apply in
the instant case.

46
HELD: In this case, both parties admit that the land in question was originally
owned by the petitioners' mother. The land was allegedly acquired later by one
Maria Lee by virtue of an extrajudicial foreclosure of mortgage. Lee, however,
never sought a writ of possession in order that she gain possession of the property
in question. The petitioners' mother therefore remained in possession of the lot. It
has been said that while the right to let property is an incident of title and
possession, a person may be lessor and occupy the position of a landlord to the
tenant although he is not the owner of the premises let. There is no need to apply
by analogy the provisions of Article 448 on indemnity as was done in Pecson vs.
Court of Appeals, because the situation sought to be avoided and which would
justify the application of that provision, is not present in this case. Suffice it to say,
"a state of forced co-ownership" would not be created between the petitioners and
the private respondents. For, as correctly pointed out by the petitioners, the right
of the private respondents as lessees is governed by Article 1678 of the Civil Code
which allows reimbursement to the extent of one-half of the value of the useful
improvements.
It must be stressed, however, that the right to indemnity under Article 1678 of the
Civil Code arises only if the lessor opts to appropriate the improvements. Since the
petitioners refused to exercise that option the private respondents cannot compel
them to reimburse the one-half value of the house and improvements. Neither can
they retain the premises until reimbursement is made. The private respondents'
sole right then is to remove the improvements without causing any more
impairment upon the property leased than is necessary.
ROSALES VS. CASTELLFORT
[G.R. No. 157044]
FACTS: Spouses-petitioners Rodolfo V. Rosales andLily Rosqueta-Rosales
(petitioners) are the registeredowners of a parcel of land with an area of
approximately 315 square meters, covered byTransfer Certificate of Title (TCT) No.
36856[4] anddesignated as Lot 17, Block 1 of Subdivision Plan LRCPsd-55244
situated in Los Baos, Laguna.On August 16, 1995, petitioners discoveredthat a
house was being constructed on their lot,without their knowledge and consent, by
respondentMiguel Castelltort (Castelltort).It turned out that respondents
Castelltortand his wife Judith had purchased a lot, Lot 16 of thesame Subdivision
Plan, from respondent Lina Lopez-Villegas (Lina) through her son-attorney-in-fact
ReneVillegas (Villegas) but that after a survey thereof bygeodetic engineer Augusto
Rivera, he pointed to Lot17 as the Lot 16 the Castelltorts purchased.Negotiations
for the settlement of the case thus began,with Villegas ofering a larger lot near
petitioners lot inthe same subdivision as a replacement thereof. In thealternative,
Villegas proposed to pay the purchaseprice of petitioners lot with legal interest.
Bothproposals were, however, rejected by petitionerswhose counsel, by letter of
August 24, 1995, directedCastelltort to stop the construction of and demolish
hishouse and any other structure he may have builtthereon, and desist from
entering the lot.Petitioners subsequently filed on September 1, 1995 a complaint
for recovery of possession anddamages with prayer for the issuance of a
restrainingorder and preliminary injunction against spouses-respondents Miguel
and Judith Castelltort before theRTC of Calamba, Laguna, docketed as Civil Case
No.2229-95ISSUE:

Whether or not Castelltort is a builder in good faith

47
RULING: Yes, he is a buider in good faith. A builder in good faith is one who builds
with the belief that the land he is building on is his, or that by some title one has
the right to build thereon, and is ignorant of any defect or flaw in his title.
Article 527 of the Civil Code provides that good faith is always presumed, and upon
him who alleges bad faith on the part of a possessor rests the burden of proof.
In the case at bar, Lot 16 was sold by Lina, through her attorney-in-fact Villegas, to
Castelltort and a certain Elizabeth Cruz for a consideration of P500,000.00. While
prior to the sale, what Villegas showed Castelltort as evidence of his mother Linas
ownership of the property was only a photocopy of her title TCT No. (T-42171) T18550[30] he explaining that the owners duplicate of the title was lost and that
judicial reconstitution thereof was ongoing, Castelltort acted in the manner of a
prudent man and went to the Registry of Deeds of Laguna to procure a certified
true copy of the TCT. The certified true copy bore no annotation indicating any
prior adverse claim on Lot 16. The records indicate that at the time Castelltort
began constructing his house on petitioners lot, he believed that it was the Lot 16
he bought and delivered to him by Villegas.
Agustin v. Intermediate Appellate Court
G.R. No. 66075-76, July 5, 1990, 187 SCRA 218
Grino Aquino, J.
FACTS: The Cagayan River separates the towns of Solana on the west and
Tuguegarao on the east in the province of Cagayan. In 1919 the lands of the east
of the river were covered by the Tuguegarao Cadastre. In 1925, OCT 5472 was
issued for land east of the Cagayan River owned by Eulogio Agustin. As the years
went by, the Cagayan River moved gradually eastward, depositing silt on the west
bank. The shifting of the river and siltation continued until 1968. In 1950, all lands
west of the river were included in the Solana Cadastre. Among these occupying
lands covered by Solana Cadastre were Pablo Binayug and Maria Melad. Through
the years, the Cagayan River eroded lands of the Tuguegarao Cadastre on its
eastern bank among which was Agustins Lot 8457, depositing the alluvium as
accretion on the land possessed by Binayug on the western bank. However, 1968,
after a big flood, the Cagayan River changed its course, returned to its 1919 bed
and in the process, cut across the lands of Maria Melad, Timoteo Melad, and the
spouses Pablo Binayug and Geronima Ubina whose lands were transferred on the
eastern, or Tuguegarao, side of the river. To cultivate those lots they had to cross
the river. In April 1969, while the Melads, Binayug, Urbina and their tenants were
planting corn on their lots located on the easter side of Cagayan River, Agustin, the
heirs of Baldomero Langcay, Juan Langcay, and Arturo Balisi, accompanied by the
mayor and some policemen of Tuguegarao, claimed the same lands as their own
and drove away the Melads, Binayug and Urbina from the premises.
ISSUE: Whether or not ownership of accretion is lost upon sudden and abrupt
change of the river.
HELD: No. The ownership of the accretion to the lands was not lost upon sudden
and abrupt change of the course of the river (Cagayan River in 1968 or 1969 when
it reverted to its old 1919 bed), and separated or transferred said accretions to the
other side (eastern bank) of the river. Articles 459 and 463 of the New Civil Code
apply to this situation. Article 459 provides that whenever the current of a river,
creek or torrent segregates from an estate on its bank a known portion of land and
transfer it to another estate, the owner of the land to which the segregated portion
belonged retains the ownership of it, provided that he removes the same within
two years. Article 463 provides that, whenever the current of a river divides itself

48
into branches, leaving a piece of land or part thereof isolated, the owner of the
land retains his ownership. He also retains it if a portion of land is separated from
the estate by the current.
Cureg v. Intermediate Appellate Court
G.R. No. 73465, September 7, 1989, 177 SCRA 313
Medialdea, J.
FACTS: On November 5, 1982, private respondents Domingo Apostol et al. filed a
complaint for quieting of title against petitioners Leonida Cureg et al. The
complaint alleged that private respondents, except Apostol, are the legal and/or
the forced heirs of the late Domingo Gerardo, and his predecessors-in-interest
have been in actual, open, peaceful and continuous possession, under a bona fide
claim of ownership of a parcel of land (referred to as their motherland).
Subsequently, the heirs verbally sold the motherland to Apostol. The
motherland showed signs of accretion caused by the movement of the Cagayan
River. When private respondents were about to cultivate their motherland
together with its accretion, they were prevented by the petitioners. Petitioners
alleged that the motherland claimed by the private respondents is non-existent,
that the subject land is an accretion to their registered land, and that petitioners
have been in possession and cultivation of the accretion for many years now.
ISSUE: Whether or not the petitioners have the better right of accretion.
HELD: Yes. The petitioners are entitled to the accretion. The subject land is an
alluvial deposit left by the northward movement of the Cagayan River and
pursuant to Article 457 of the New Civil Code: To the owners of land adjoining the
banks of river belong the accretion which they gradually receive from the efects of
the current of the waters. However, the increase in the area of the petitioners
land, being an accretion left by the change of course or the northward movement
of the Cagayan River does not automatically become registered land just because
the lot which receives such accretion is covered by a Torrens title. As such, it must
also be placed under the operation of the Torrens system.
Viajar v. Court of Appeals
G.R. No. 77294, December 12, 1988, 168 SCRA 405
Medialdea, J.
FACTS: The spouses Ricardo and Leonor Ladrido were the owners of Lot 7511.
Spouses Rosendo and Ana Te were also the registered owners of a parcel of land
described in their title as Lot 7340 of the Cadastral Survey of Pototan. On 6
September 1973, Rosendo Te, with the conformity of his wife, sold this lot to
Angelica F. Viajar and Celso F. Viajar for P5,000. A Torrens title was later issued in
the latters names. Later, Angelica Viajar had Lot 7340 relocated and found out
that the property was in the possession of Ricardo Y. Ladrido. Consequently, she
demanded its return but Ladrido refused. The piece of real property which used to
be Lot 7340 of the Cadastral Survey of Pototan was located in barangay
Guibuanogan, Pototan, Iloilo; that at the time of the cadastral survey in 1926, Lot
7511 and Lot 7340 were separated by the Suague River; that Lot 7340 has been in
the possession of Ladrido; that the area of 14,036 sq.ms., which was formerly the
river bed of the Suague River per cadastral survey of 1926, has also been in the
possession of Ladrido; and that the Viajars have never been in actual physical
possession of Lot 7340. On 15 February 1974, Angelica and Celso Viajar instituted
a civil action for recovery of possession and damages against Ricardo Y. Ladrido.
The trial court rendered its decision in favor of Ladrido, dismissing the complaint of

49
Angelica and Celso Viajar with costs against them, declaring the Ladridos are
entitled to the possession thereof. Not satisfied with the decision, the Viajars
appealed to the Court of Appeals. The Court of Appeals affirmed the decision of the
court. The Viajars filed a petition for review on certiorari.
ISSUE: Whether the respondents are entitled to the land on the ground of
accretion.
HELD: Article 457 of the New Civil Code provides that to the owners of lands
adjoining the banks of rivers belong the accretion which they gradually receive
from the efects of the current of the waters." The presumption is that the change
in the course of the river was gradual and caused by accretion and erosion. In the
present case, the lower court correctly found that the evidence introduced by the
Viajars to show that the change in the course of the Suague River was sudden or
that it occurred through avulsion is not clear and convincing. The Ladridos have
sufficiently established that for many years after 1926 a gradual accretion on the
eastern side of Lot 7511 took place by action of the current of the Suague River so
that in 1979 an alluvial deposit of 29,912 sq.ms. more or less, had been added to
Lot 7511. The established facts indicate that the eastern boundary of Lot 7511 was
the Suague River based on the cadastral plan. For a period of more than 40 years
(before 1940 to 1980) the Suague River overflowed its banks yearly and the
property of the defendant gradually received deposits of soil from the efects of the
current of the river. The consequent increase in the area of Lot 7511 due to
alluvion or accretion was possessed by the defendants whose tenants plowed and
planted the same with corn and tobacco. The quondam river bed had been filled by
accretion through the years. The land is already plain and there is no indication on
the ground of any abandoned river bed. Under the law, accretion which the banks
or rivers may gradually receive from the efects of the current of the waters
becomes the property of the owners of the lands adjoining the banks. Therefore,
the accretion to Lot 7511 which consists of Lots A and B belong to the Ladridos.
Vda. De Nazareno v. Court of Appeals
G.R. No. 98045, June 26, 1996, 257 SCRA 589
Romeo, J.
FACTS: The subject of this controversy is a parcel of land formed as a result of
sawdust dumped into the dried-up Balacanas Creek and along the banks of the
Cagayan river. Private respondents Salasalan and Rabaya leased the subject lots
on which their houses stood from Antonio Nazareno, petitioners predessor-ininterest. Private respondents allegedly stopped paying rentals. As a result,
Nazareno and petitioners filed a case for ejectment with the MTC of Cagayan de
Oro City. The MTC rendered a decision against private respondents which was
affirmed by the RTC. After several petitions for annulmentof judgment by private
respondents which were all dismissed, the decision of the lower court was finally
enforced with the private respondents being ejected from portions of the subject
lots they occupied. Before Nazareno died, he caused the approval by the Bureau of
lands of the survey plan with a view to perfecting his title over the accretion area
being claimed by him. The said petition was protested by private respondents.
After conducting a survey of the subject land, land investigator Avelino labis
recommended that the survey plan be cancelled and that private respondents be
directed to file appropriate public land application covering their respective
portions. Nazareno filed a motion for reconsideration with the Undersecretary of
the Department of Natural Resources and OIC of the Bureau of lands Ignacio who
denied the Motion. Respondent Director of lands Abelardo Palad ordered Nazareno
to vacate the portions adjudicated to private respondents and remove whatever
improvements they have introduced; he also ordered that private respondents be

50
placed in possession thereof. A petitioner filed a case for annulment of the previous
decisions with the RTC but was dismissed. The CA affirmed the RTC decision
contending that the approved of the survey plan belongs exclusively to the
Director of lands and the same shall be conclusive when approved by the Secretary
of Agriculture and Natural Resources.
ISSUE: Whether or not petitioners can claim ownership of the subject land by
virtue of Art 457 of the Civil Code.
HELD: No, accretion as a mode of acquiring property under Art 457 of the NCC
requires the concurrence of the requisites mentioned in the Article. These are
called rules on alluvion, which if present in a case, give to the owners of lands
adjoining the banks of rivers or streams any accretion gradually received from the
efects of the current of waters. The word current indicates the participation of
the body of water in the flow of waters due to high and low tide. Petitioners,
however, admit that the accretion was formed by the dumping of boulders, soil and
other filling materials on portions of the Balacanas creek and the Cagayan River.
The Bureau of lands classified the subject land as an accretion area which was
formed by deposits of sawdust. Petitioners submission not having met the first and
second requirements of the rules of alluvion, they cannot claim the rights of a
riparian owner. The subject being public land is under the jurisdiction of the Bureau
of lands, respondent Palad is authorized to exercise executive control over any
form of concession, disposition and management of the lands of public dominion.
Heirs of Navarro v. Intermediate Appellate Court
G.R. No. 68166, February 12, 1997, 268 SCRA 589
Hermosisima, J:
FACTS: On October 3, 1946, Sinforoso Pascual, filed an application for foreshore
lease covering a tract of foreshore land in Sibocon, Balanga, Bataan, having an
area of approximately seventeen (17) hectares. Subsequently, petitioners'
predecessor-in-interest, Emiliano Navarro, filed a fishpond application with the
Bureau of Fisheries covering twenty five (25) hectares of foreshore land also in
Sibocon, Balanga, Bataan. Initially, such application was denied by the Director of
Fisheries on the ground that the property formed part of the public domain.
Sometime in the early part of 1960, Sinforoso Pascual flied an application to
register and confirm his title to a parcel of land, situated in Sibocon, Balanga,
Bataan, described in Plan Psu-175181 and said to have an area of 146,611 square
meters. Pascual claimed that this land is an accretion to his property, situated in
Barrio Puerto Rivas, Balanga, Bataan, and covered by Original Certificate of Title
No. 6830. It is bounded on the eastern side by the Talisay River, on the western
side by the Bulacan River, and on the northern side by the Manila Bay. The Talisay
River as well as the Bulacan River flow downstream and meet at the Manila Bay
thereby depositing sand and silt on Pascual's property resulting in an accretion
thereon. Sinforoso Pascual claimed the accretion as the riparian owner.
On March 25, 1960, the Director of Lands, represented by the Assistant Solicitor
General, filed an opposition thereto stating that neither Pascual nor his
predecessors-in-interest possessed sufficient title to the subject property, the same
being a portion of the public domain and, therefore, it belongs to the Republic of
the Philippines.
ISSUE: Whether or not the land sought to be registered is accretion or foreshore
land, or, whether or not said land was formed by the action of the two rivers of
Talisay and Bulacan or by the action of the Manila Bay.

51
HELD: Accretion as a mode of acquiring property under said Article 457, requires
the concurrence of the following requisites: (1) that the accumulation of soil or
sediment be gradual and imperceptible; (2) that it be the result of the action of the
waters of the river; and (3) that the land where the accretion takes place is
adjacent to the bank of the river. If the accretion were to be attributed to the action
of either or both of the Talisay and Bulacan Rivers, the alluvium should have been
deposited on either or both of the eastern and western boundaries of petitioners'
own tract of land, not on the northern portion thereof which is adjacent to the
Manila Bay. Clearly lacking, thus, is the third requisite of accretion, which is, that
the alluvium is deposited on the portion of claimant's land which is adjacent to the
river bank.
The disputed land, thus, is an accretion not on a river bank but on a sea bank, or
on what used to be the foreshore of Manila Bay which adjoined petitioners' own
tract of land on the northern side. Applicant Pascual has not presented proofs to
convince the Court that the land he has applied for registration is the result of the
settling down on his registered land of soil, earth or other deposits so as to be
rightfully be considered as an accretion [caused by the action of the two rivers].
Article 457 finds no applicability where the accretion must have been caused by
action of the bay.
The conclusion formed by the trial court on the basis of the aforegoing observation
is that the disputed land is part of the foreshore of Manila Bay and therefore, part
of the public domain. Thus, the disputed property is an accretion on a sea bank,
Manila Bay being an inlet or an arm of the sea; as such, the disputed property is,
under Article 4 of the Spanish Law of Waters of 1866, part of the public domain.
Avila v. Spouses Barabat
G.R. No. 141993
FACTS: The subject of this controversy is a 433 square meter parcel of land
located in Toledo City, Cebu in the name of Anunciacion Bahena de Nemeo. Upon
her death, ownership was transferred to her five children, petitioners Narcisa Avila,
Natividad, Francisca, Leon and Jose. They built their respective houses on the lot.
Respondent Benjamin Barabat and his wife leased a portion of the house owned by
Avila. She then ofered it to respondents who agreed to buy it. Their agreement
was evidenced by a private document.
Respondents were then confronted by petitioner Januario Adlawan who informed
them that they need to leave the place because he was buying the property.
Respondents replied that the property had already been sold to them. Respondents
filed a complaint for quieting of title with the Regional Trial Court (RTC). Avila
denied having ofered to sell her property to respondents. Petitioners claim that
they were the ones who paid taxes. They also claimed that they have the right to
redeem the property.
ISSUE: Whether petitioners may redeem the subject property
RULING: No. The evidence show that Avila intended to sell the property to
respondents. Also, it was only after the perfection of the contract, when her
siblings began protesting the sale, that she wanted to change the agreement.
Petitioners right to redeem would have existed only had there been co-ownership
among petitioners-siblings. none. For this right to be exercised, co-ownership must
exist at the time the conveyance is made by a co-owner and the redemption is
demanded by the other co-owner or co-owners. However, by their own admission,

52
petitioners were no longer co-owners when the property was sold to respondents in
1979. The co-ownership had already been extinguished by partition.
Del Banco v. Intermediate Appellate Court
G.R. No. 72694, December 1, 1987, 156 SCRA 55
Paras, J.
FACTS: In a document executed in the Municipality of San Rafael, Bulacan, on
February 11, 1859, three brothers, Benedicto Pansacola, Jose Pansacola and
Manuel Pansacola (known as Fr. Manuel Pena) entered into an agreement which
provided, among others: (1) That they will purchase from the Spanish Government
the lands comprising the Island of Cagbalite which is located within the boundaries
of the Municipality of Mauban, Province of Tayabas (now Quezon) and has an
approximate area of 1,600 hectares; (2) That the lands shall be considered after
the purchase as their common property; (3) That the co-ownership includes
Domingo Arce and Baldomera Angulo, minors at that time represented by their
father, Manuel Pansacola (Fr. Manuel Pena) who will contribute for them in the
proposed purchase of the Cagbalite Island; (4) That whatever benefits may be
derived from the Island shall be shared equally by the co-owners in the following
proportion: Benedicto Pansacola-1/4 share; Jose Pansacola-1/4 share; and,
Domingo Arce and Baldomera Angulo-2/4 shares which shall be placed under the
care of their father, Manuel Pansacola (Fr. Manuel Pena). On August 14, 1866, coowners entered into the actual possession and enjoyment of the Island purchased
by them from the Spanish Government. On April 11, 1868 they agreed to modify
the terms and conditions of the agreement entered into by them on February 11,
1859.
About one hundred years later, on November 18, 1968, private respondents
brought a special action for partition in the Court of First Instance of Quezon, under
the provisions of Rule 69 of the Rules of Court, including as parties the heirs and
successors-in-interest of the co-owners of the Cagbalite Island in the second
contract of co-ownership dated April 11, 1968. In their answer some of the
defendants, petitioners herein, interposed such defenses as prescription, res
judicata, exclusive ownership, estoppel and laches.
After trial on the merits, the trial court rendered a decision dated November 6,
1981 dismissing the complaint. The motion for reconsideration filed by the
plaintifs, private respondents herein, was denied by the trial court in an order
dated February 25, 1982. On appeal, respondent Court reversed and set aside the
decision of the lower court .It also denied the motion for reconsideration and the
supplement to motion for reconsideration filed by private respondents, in its
resolution dated October 15, 1983.
ISSUES:
1.) Whether or not Cagbalite Island is still undivided property owned in common by
the heirs and successors-in-interest of the brothers, Benedicto, Jose and Manuel
Pansacola.
2.) Whether or not a prescription may run in favor of a co-owner against his coowners or co-heirs.
HELD: 1.) On the first issue, there is nothing in all four agreements that suggests
that actual or physical partition of the Island had really been made by either the
original owners or their heirs or successors-in-interest. The agreement entered into
in 1859 simply provides for the sharing of whatever benefits can be derived from
the island. The agreement, in fact, states that the Island to be purchased shall be
considered as their common property. In the second agreement entered in 1868

53
the co-owners agreed not only on the sharing proportion of the benefits derived
from the Island but also on the distribution of the Island each of the brothers was
allocated a 1/4 portion of the Island with the children of the deceased brother,
Eustaquio Pansacola allocated a 1/4 portion and the children of Manuel Pansacola
(Fr. Manuel Pena) also allocated a 1/4 portion of the Island. With the distribution
agreed upon each of the co-owner is a co-owner of the whole, and in this sense,
over the whole he exercises the right of dominion, but he is at the same time the
sole owner of a portion, in the instant case, a 1/4 portion (for each group of coowners) of the Island which is truly abstract, because until physical division is
efected such portion is merely an Ideal share, not concretely determined (3
Manresa, Codigo Civil, 3rd Ed., page 486, cited in Lopez vs. Cuaycong, 74 Phil. 601;
De la Cruz vs. Cruz, 32 SCRA 307 [1970]; Felices vs. Colegado, 35 SCRA 173
[1970],; Dultra vs. CFl 70 SCRA 465 [1976]; Gatchalian vs. Arlegui, 75 SCRA 234
[1977].)
In the agreement of January 20, 1907, the heirs that were represented agreed on
how the Island was to be partitioned. The agreement of April 18, 1908 which
supplements that of January 20, 1907 reveals that as of the signing of the 1908
agreement no actual partition of the Island had as yet been done. The second and
fourth paragraphs of the agreement speaks of a survey yet to be conducted by a
certain Amadeo and a plan and description yet to be made. Virgilio Pansacola, a
son of the surveyor named Amadeo who is referred to in the contract dated April
18, 1908 as the surveyor to whom the task of surveying Cagbalite Island pursuant
to said agreement was entrusted, however, testified that said contracts were never
implemented because nobody defrayed the expenses for surveying the same.
It is not enough that the co-owners agree to subdivide the property. They must
have a subdivision plan drawn in accordance with which they take actual and
exclusive possession of their respective portions in the plan and titles issued to
each of them accordingly (Caro vs. Court of Appeals, 113 SCRA 10 [1982]). The
mechanics of actual partition should follow the procedure laid down in Rule 69 of
the Rules of Court. Maganon vs. Montejo, 146 SCRA 282 [1986]).
Neither can such actual possession and enjoyment of some portions of the Island
by some of the petitioners herein be considered a repudiation of the co-ownership.
It is undisputed that the Cagbalite Island was purchased by the original co-owners
as a common property and it has not been proven that the Island had been
partitioned among them or among their heirs. While there is co-ownership, a coowner's possession of his share is co-possession which is linked to the possession
of the other co-owners (Gatchalian vs. Arlegui, 75 SCRA 234 [1977]).
2.) On the second issue, no prescription shall run in favor of a co-owner against his
co-owners or co-heirs so long as he expressly or impliedly recognizes the coownership (Valdez vs. Olonga, 51 SCRA 71 [1973], Tero vs. Tero, 131 SCRA 100
[1984]). Co-owners cannot acquire by prescription the share of the other coowners, absent a clear repudiation of the co-ownership clearly communicated to
the other co-owners. An action for partition does not prescribe. Article 403 of the
Old Civil Code, now Article 497, provides that the assignees of the co-owners may
take part in the partition of the common property, and Article 400 of the Old Code,
now Article 494 provides that each co-owner may demand at any time the partition
of the common property, a provision which implies that the action to demand
partition is imprescriptible or cannot be barred by laches (Budlong vs. Pondoc, 79
SCRA 24 [1977]). An action for partition does not lie except when the co-ownership
is properly repudiated by the co- owner.
Adlawan vs. Adlawan

54
G.R. No. 161916, January 20, 2006
Ynares Santiago, J.
FACTS: Petitioner Arnelito Adlawan, the acknowledged illegitimate child of
Dominador Adlawan filed an ejejctment suit against the siblings of his father,
respondents Narcisa and Emeterio Adlawan. Being the sole heir of Dominador, he
executed an affidavit adjudicating the house and lot owned by his father. However,
he alleged that out of respect and generosity to respondents, he granted their plea
to occupy the subject property provided they would vacate the same should his
need for the property arise. Later, when he verbally requested respondents to
vacate the house and lot, they refused and filed instead an action for quieting of
title. He then also filed a complaint for ejectment. In answer, the respondents, 70
and 59 years of age respectively denied that they begged petitioner to allow them
to say on the property since they have been staying there since birth. They
claimed that the said lot was originally registered in the name of their deceased
parents, Ramon and Oligia Adlawan. Spouses Ramon and Oligia needed money to
finance the renovation of their house. Since they were not qualified to obtain a
loan, they transferred ownership of the lot to Dominador who was the only one in
the family who had a college education. Dominador and his wife, Graciana did not
disturb respondents possession of the property until they died. They also argued
that even if petitioner is indeed Dominadors acknowledged illegitimate son, his
right to succeed is doubtful because Dominador was survived by his wife,
Graciana.
ISSUE: Whether or not the petitioner can validly maintain the instant case of
ejectment.
HELD: No. Petitioner averred that he is an acknowledged illegitimate son and the
sole heir of Dominador. However, the RTC lost sight of the fact that the theory of
succession invoked by petitioner would end up proving that he is not the sole
owner of the subject lot. This so because Dominador was survived not only by
petitioner but also by his legal wife, Graciana, who died 10 years after the death of
Dominador. By intestate succession, Graciana and petitioner became co-owners of
the subject lot and house. Petitioner then contended that even granting that he is
a co-owner, he can file the instant case pursuant to Article 487 of the Civil Code.
This article covers all kinds of actions for the recovery of possession. It includes
forcible entry and unlawful detainer (accion interdictal), recovery of possession
(accion publiciana) and recovery of ownership (accion de reinvindicacion). A coowner may bring such action without the necessity of joining all the other coowners as co-plaintifs because the suit is presumed to have been filed to benefit
his co-owners. It should be stressed, however, that where the suit is for the benefit
of the petitioner alone who claims to be the sole owner and entitled to the
possession of the litigated property, the action should be dismissed.
According to the renowned civilest, Professor Arturo M. Tolentino, he explained that
a co-owner may bring such an action, without the necessity of joining all the other
co-owners as co-plaintifs, because the suit is deemed to be instituted for the
benefit of all. If the action is for the benefit of the plaintif alone, such that he
claims possession for himself and not for the co-ownership, the action will not
prosper. In this case, it is not disputed that petitioner brought the suit for unlawful
detainer in his name alone and for his own benefit to the exclusion of the heirs of
Graciana as he even executed an affidavit of self-adjudication over the disputed
property. It is clear therefore that petitioner cannot validly maintain the instant
action considering that he does not recognize the co-ownership that necessarily
flows from his theory of succession to the property of his father, Dominador.

55
Oesmer vs. Paraiso Development Corporation
G.R. No. 157493, February 5, 2007)
FACTS: Petitioners Rizalino, Ernesto, Leonora, Bibiano, Jr., Librado, and Enriquita,
Adolfo and Jesus, Oesmer are brothers and sisters, and the co-owners of undivided
shares of two parcel of land. Respondent Paraiso Development Corporation drafted
a Contract to Sell5 signed by the petitioners except Adolfo and Jesus. A check in the
amount of P100,000.00, payable to Ernesto, was given as option money. Later, the
petitioners informed the respondent of their intention to rescind the Contract to
Sell and to return the amount of P100,000.00.
Petitioners filed a Complaint7 for Declaration of Nullity or for Annulment of
Option Agreement or Contract to Sell with Damages before the RTC. The RTC ruled
that the contract to sell was valid and binding. Ernesto, who was alleged to have
acted as an agent, was ordered to execute the Contract of Absolute Sale
concerning his 1/8 share over the subject two parcels of land in favor of herein
respondent.
Petitioners appealed to CA which ordered petitioners who signed the contract to
execute the Deed of Absolute Sale concerning their 6/8 share over the subject two
parcels of land and in favor of herein respondent.
ISSUE :whether or not the rights of Adolfo and Jesus as co-owners of the subject
land is afected by the contract to sell
RULING: No, The Supreme Court explained that being owners of their respective
undivided shares in the subject properties, they can dispose of their shares even
without the consent of all the co-heirs. Article 493 of the Civil Code expressly
provides: Each co-owner shall have the full ownership of his part and of the fruits
and benefits pertaining thereto, and he may therefore alienate, assign or
mortgage it, and even substitute another person in its enjoyment, except when
personal rights are involved.
The Supreme Court affirmed the ruling of the CA
AGUIRRE, ET AL. vs. COURT OF APPEALS, ET AL.
G.R. No. 122249 January 29, 2004
FACTS: Leocadio Medrano and his first wife Emilia owned a piece of land. When
Emilia died, he married Miguela. When Leocadio died, all his heirs agreed that Sixto
Medrano, a child of the first marriage, should manage and administer the said
property. After Sixto died, his siblings learned that he sold a portion of the subject
land to Tiburcio Balitaan and another portion to Maria Bacong, Maria Bacong later
sold the said portion to Rosendo Bacong. Petitioners, all heirs of Leocadio who were
afected by the sale demanded reconveyance of the portions sold by Sixto but the
3 vendees refused. Resultantly, petitioners filed a suit against them seeking the
nullity of the documents and partition thereof. The vendees contended that they
acquired the property under the valid deed of sale and petitioners cause of action
was barred by laches and prescription. Tiburcio also contended that he is an
innocent purchaser for value.
ISSUE: Whether or not there was a valid sale considering the fact that it was
made without the consent of the co-owners
HELD: The Supreme Court held that the sale was a valid conveyance only insofar
as the share of Sixto Medrano in the co-ownership is concerned. Under Article 493
of the New Civil Code, a sale by a co-owner of the whole property as his will afect
only his own share but not those of the other co-owners who did not consent to the
sale). The provision clearly provides that the sale or other disposition afects only
the sellers share, and the transferee gets only what corresponds to his grantors
share in the partition of the property owned in common. Since a co-owner is
entitled to sell his undivided share, a sale of the entire property by one co-owner

56
without the consent of the other co-owner is not null and void; only the rights of
the co-owner-seller are transferred, thereby making the buyer a co-owner of the
property.
BALO vs. Court of Appeals
G.R. No. 129704, September 30, 2005
FACTS: Josefina Garrido filed a complaint for Judicial Partition of Real Properties
and Accounting with Damages against Ulpiano Balo and his children alleging that
she and petitioners are the co-owners of undivided parcels of land. The subject
lands were originally owned by the spouses Eugenio Balo, Sr. and Ma. Pasagui-Balo,
who had two children, Ulpiano, Sr. and Maximino, the father of Garrido who already
died. Immediately upon the death of her grandfather, the petitioners took
possession of the land without her knowledge and consent.
ISSUE: WHETHER OR NOT THE ACTION FOR JUDICIAL PARTITION sgould be
dismissed when the the defendant asserts exclusive title over the property
HELD: The Supreme court agreed to the ruling of the CA that An action for
partition is at once an action for declaration of co-ownership and for segregation
and conveyance of a determine portion of the properties involved. If the defendant
asserts exclusive title over the property, the action for partition should not be
dismissed. Rather, the court should resolve the case and if the plaintif is unable to
sustain his claimed status as a co-owner, the court should dismiss the action, not
because the wrong remedy was availed of, but because no basis exists for
requiring the defendant to submit to partition. If, on the other hand, the court after
trial should find the existence of co-ownership among the parties, the court may
and should order the partition of the properties in the same action.
Galvez vs. Court of Appeals
G.R. No. 157954, March 24, 2006
Chico Nazario, J.
FACTS: Timotea F. Galvez died intestate and left a parcel of land in La Union. She
left behind her children Ulpiano and petitioner Paz Galvez. Ulpiano who died before
Timotea was survived by his son, private respondent, Porfirio Galvez. With regards
to the property of Timotea, it is supposed to pass to Paz and Porfirio. However,
Porifirio was surprised to discover that Paz executed an affidavit of adjudication
stating that she is the true and lawful owner of the said property. Moreover,
without the knowledge and consent of Porfirio, Paz sold the property to petitioner
Carlos Tam for P10,000.00. Tam thereafter filed an application for registration for
said parcel of land. Subsequently, Tam sold the property to Tycoon Properties, Inc.
Having knowledge of such sale, Porfirio filed a complaint for Legal Redemption with
Damages and Cancellation of documents against petitioner which was affirmed by
the lower court and the Court of Appeals.
ISSUES:
1.) Whether or not the claim of Porfirio Galvez which is based on an implied trust
has already prescribed because the action was filed 24 years after Paz Galvez
repudiated the said trust?
2.) Whether or not the claim of Porfirio Galvez which is based on an implied trust is
already banned by laches because he failed to assert his alleged right for
almost 24 years?
3.) Whether or not Carlos Tam and Tycoon Properties are buyers in good faith and
for value and has the right to rely on the face of the title?

57
HELD: 1.) No. Article 494 of the Civil Code provides that "a prescription shall not
run in favor of a co-owner or co-heir against his co-owners or co-heirs as long as he
expressly or impliedly recognizes the co-ownership." It is a fundamental principle
that a co-owner cannot acquire by prescription the share of the other co-owners,
absent any clear repudiation of the co-ownership. Prescription, as a mode of
terminating a relation of co-ownership, must have been preceded by repudiation
(of the co-ownership). The act of repudiation, in turn, is subject to certain
conditions: (1) a co-owner repudiates the co-ownership; (2) such an act of
repudiation is clearly made known to the other co-owners; (3) the evidence
thereon is clear and conclusive; and (4) he has been in possession through open,
continuous, exclusive, and notorious possession of the property for the period
required by law. In this case, we find that Paz Galvez efected no clear and evident
repudiation of the co-ownership. The execution of the affidavit of self-adjudication
does not constitute such sufficient act of repudiation as contemplated under the
law as to efectively exclude Porfirio Galvez from the property. This Court has
repeatedly expressed its disapproval over the obvious bad faith of a co-heir
feigning sole ownership of the property to the exclusion of the other heirs
essentially stating that one who acts in bad faith should not be permitted to profit
from it to the detriment of others.
2.) No. On the matter of laches, it is hornbook doctrine that laches is a creation of
equity and its application is controlled by equitable considerations. Laches cannot
be used to defeat justice or perpetrate fraud and injustice. Neither should its
application be used to prevent the rightful owners of a property from recovering
what has been fraudulently registered in the name of another. The equitable
remedy of laches is, therefore, unavailing in this case.
3.) No. As to petitioners Carlos Tam and Tycoon Properties, Inc.s claim that they
are buyers in good faith, same fails to persuade. A purchaser in good faith and for
value is one who buys the property without notice that some other person has a
right to or interest in such property and pays its fair price before he has notice of
the adverse claims and interest of another person in the same property. So it is
that the "honesty of intention" which constitutes good faith implies a freedom from
knowledge of circumstances which ought to put a person on inquiry. "Tam did not
exert eforts to determine the previous ownership of the property in question" and
relied only on the tax declarations in the name of Paz Galvez. It must be noted that
Carlos Tam received a copy of the summons and the complaint on 22 September
1994. This notwithstanding, he sold the property to Tycoon Properties, Inc. on 27
September 1994. Significantly, Carlos Tam is also an owner of Tycoon Properties,
Inc. to the extent of 45%. A notice of lis pendens dated 8 July 1997 filed with the
Registry of Deeds of the Province of La Union was inscribed on TCT No. T- 40390.
Despite the inscription, Tycoon Properties, Inc. mortgaged the land to Far East Bank
and Trust Company for the sum of P11,172,600. All these attendant circumstances
negate petitioners claim of good faith.
Baloloy vs Hular
G.R. No. 157767
FACTS: Spouses Lino and Victoriana Estopin were the original owners of a parcel
of land located in Barangay Biriran, Juban, Sorsogon ( Lot No. 3347 ) of the Juban
Cadastre. A major portion of the property was agricultural, while the rest was
residential. November 11 and 25, 1961: When Lino Estopin died intestate, his
widow, Victoriana Lagata, executed a Deed of Absolute Sale on over the
agricultural portion of Lot No. 3347, ( 15,906 sqm) and the residential portion of
the property (287 sqm) to Astrologo Hular. In 1961 or thereabouts: Iluminado

58
asked Hulars permission to construct a house on a portion of Lot No. 3347 near
the road, and the latter agreed.
Iluminado Baloloy in 1945 acquired a coconut land (north of the residential portion
of Lot 3347 Lot No. 3353 (9302 sqm) and registered the same. Iluminado
constructed his house on a portion of Lot No. 3353. He and his family, including
his children, forthwith resided in said house.
In 1979, respondent Hular had his house constructed near the trail (road) on Lot
No. 3347, which, however, occupied a big portion of Lot No. 3353.
Iluminado died intestate on November 29, 1985. His widow and their children
continued residing in the property, while petitioner Reynaldo Baloloy, one of
Iluminados children, later constructed his house near that of his deceased father.
When Astrologo died, he was survived by his children, Jose, Romeo, Anacleto,
Elena, Leo, Teresita, and the respondent, among others, who continued to reside in
their house.
Sometime in l991 the respondent had Lot No. 3353 surveyed and discovered that
the residential area deeded by Lagata to Astrologo Hular had an area of 1,405
square meters, instead of 287 square meters only.
Respondent Alfredo Hular filed a complaint for quieting of title of real property
against the children and heirs of Iluminado Baloloy, namely, Anacorita, Antonio,
and petitioners Reynaldo and Adelina, all surnamed Baloloy. He prayed among
others that he be declared the absolute owner of the property in question.
ISSUE: Whether all the indispensable parties had been impleaded by the
respondent in the trial court.
HELD: Respondent adduced evidence that when his parents died intestate, they
were survived by their children, the respondent and his siblings Elena, Jose,
Romeo, Anacleto, Leo, and Teresita.
Article 1078 of the Civil Code provides that where there are two or more heirs, the
whole estate of the decedent is, before partition, owned in common by such heirs,
subject to the payment of the debts of the deceased. Under Article 487 of the New
Civil Code, any of the co-owners may bring an action in ejectment. This article
covers all kinds of actions for the recovery of possession, including an accion
publiciana and a reinvidicatory action.
If the action is for the benefit of the
plaintif alone who claims to be the sole owner and entitled to the possession
thereof, the action will not prosper unless he impleads the other co-owners who
are indispensable parties.
In this case, the respondent alone filed the complaint, claiming sole ownership over
the subject property and praying that he be declared the sole owner thereof.
There is no proof that the other co- owners had waived their rights over the
subject property or conveyed the same to the respondent or such co-owners were
aware of the case in the trial court.
Pardell v. Bartolome
G.R. No. L-4656, November 18, 1912, 23 Phil. 450
Torres, J.
FACTS: Plaintif Vicenta Ortiz and defendant Matilde Ortiz are the duly recognized
natural daughters of the spouses Miguel and Calixta who died in Vigan, Ilocos Sur.

59
Prior to the death of their mother, she executed a will whereby Matilde and Vicenta
became the heirs of all her property. Subsequently, defendants, without judicial
authorization or extrajudicial agreement took over the administration and
enjoyment of the properties as well as collection of the rents, fruits and products
thereof. Moreover, Matilde and her husband occupied the upper storey of the
house and the room of the lower floor as an office. With this, Vicenta demanded
that she be given rental payments by Matilde in occupying the house since she is a
co-owner of the property not occupying the same and as such is entitled to its
enjoyment and/or fruits.
ISSUE: Whether or not Vicenta can collect rentals from Matilde who occupies and
enjoy the property alone as a co-owner.
HELD: No. The law grants each co-owner the right to use the property for the
purpose intended provided that the interest of the co-ownership must not be
injured or prejudiced and the other co-owners must not be prevented from using it
according to their rights.
Matilde occupied the property owned in common in accordance with the purpose
for which it is intended. Records show no proof that she neither occasioned any
detriment to the interest of the community property nor prevented her sister from
utilizing the said property in accordance to her right as a co-owner thereof. Matilde
was excercising her right as a co-owner without being prejudicial to Vicenta who
could have also occupied her property had she wanted to.
Each co-owner of a property has the right pro-indiviso over the whole property and
may use and enjoy the same with no other limitation than that he shall not injure
the interests of his co-owners, for the reason that until a division is made, the
respective part of each holder of a right as a co-owner cannot be determined and
every co-owner exercises joint ownership over the pro-indiviso property in addition
to his use and enjoyment of the same.
Caro v. Court of Appeals
G.R. No. L-46001, March 25, 1982, 113 SCRA 10
Guerrero, J.
FACTS: Alfredo Benito, Mario Benito and Benjamin Benito were the original coowners of two parcels of land somewhere in Sorsogon. Sometime in 1957, Mario
died. His wife, Basilia Lahorra and his father, Saturnino Benito, were subsequently
appointed as joint administrators of Marios estate by the CFI of Sorsogon.
On August 26, 1959, Benjamin executed a deed of absolute sale of his one-third
undivided portion over said parcels of land in favor of herein petitioner, Luz Caro
for the sum of 10,000.
Subsequently, with the consent of Saturnino Benito and Alfredo Benito as shown in
their affidavits, a subdivision title was issued to petitioner Luz Caro over the lot.
Sometime in May 1966, when private respondent Basilia Lahorra learned from a
pleading sent to her that petitioner Luz Caro acquired from Benjamin Benito the
aforesaid one-third of the undivided share of the subject lands. She sent to
petitioner thru counsel, a written ofer to redeem the said one-third share.
However, this ofer was ignored by the petitioner. Hence, private respondent
Basilia Lahorra filed a case for legal redemption and sought to prove that as joint
administrator of the estate of Mario Benito, she had not been notified of the sale as
required by articles 1620 and 1623 of the Civil Code.

60
During the hearing of the case, petitioner presented the following secondary
evidence to prove the service of notice of the intended sale to possible
redemptioners: (1) affidavit of Benjamin Benito attesting to the fact that the
possible redemptioners were formally notified in writing of his intention to sell his
undivided share; (2) deposition of Saturninos widow that she received and showed
the notice to husband but the latter was not interested to buy the property.
The trial court ruled in favor of the petitioner. However, the decision was reversed
by the CA. Hence, the case was brought to the SC.
ISSUE: Whether or not co-ownership on the lots in question still exist thereby
allowing private respondent Basilia Lahorra to exercise the right of legal
redemption.
HELD: The court held that as early as 1960, co-ownership of the parcels of land
covered by TCT Nos. T-609 and T-610 was terminated when Alfredo Benito, Luz
Caro and the intestate estate of Mario Benito, represented by administrators
Saturnino Benito, as trustee and representative of the heirs of Mario Benito, agreed
to subdivide the property. It added that an agreement of partition, though oral, is
valid and consequently binding upon the parties.
A partition for subdivision was then filed for the purpose. This was accompanied by
the affidavits of Alfredo Benito and Saturnino Benito to the efect that they agree
to the segregation of the land owned in common by the three amigos. A
subdivision plan was made and by common agreement Lot 1-C, with an area of 163
hectares, was ceded to petitioner, to wit, TCT no. T-4978.
In addition, notwithstanding the ruling in the Caram case wherein the sale of the
property took place after the partition agreement, the court therein saw no
diference with respect to a conveyance which took place before the partition
agreement.
Regarding the contention of private respondent that she was not notified of the
sale, the court ruled that since the right of legal redemption does not exist nor
apply in this case because admittedly a subdivision title has already been issued in
the name of the petitioner on Lot 1-C sold to her, it becomes moot and academic.
It becomes unnecessary to decide whether private respondent complied with the
requirements for the exercise of legal redemption under Article 1623 of the New
Civil Code.
Bailon Casilao v. Court of Appeals
G.R. No. 78178, April 15, 1988, 160 SCRA 738
Cortes, J.
FACTS: The Roman Catholic Archbishop [sic] of Manila was the owner of a parcel
of land (Lot No. 1272, Balanga Cadastre) situated in the Barrio of Puerto Rivas,
Municipality of Balanga, Bataan, having an area of 3,368 sq. m., more or less
covered by OCT No. 14379 of de Registry of Deeds for the province of Bataan. With
respect to its rights over its properties in Bataan (inclusive of Lot No. 1272), the
said church was succeeded by the Roman Catholic Bishop of San Fernando,
Pampanga which was, likewise, succeeded by Catholic Bishop of Balanga
registered as a corporation on 15 December 1975.Prior thereto, or on 23 August
1936, by virtue of the authority given him by the Roman Catholic Archbishop of
Manila to donate a portion of Lot No. 1272, the then parish priest and administrator
of all the properties of the said church in the Municipality of Balanga Bataan, Rev.

61
Fr. Mariano Sarili, executed an Escritura De Donacion donating an area of 12.40
meters by 21.40 meters or 265.36 sq. m (the subject property) of Lot No. 1272 to
Ana de los Reyes and her heirs, as a reward for her long and satisfactory service to
the church. Her acceptance of the donation, as well as her possession of the
subject property, is indicated in the deed of donation, which deed, for unknown
reasons, was refused registration by the Register of Deeds. Six (6) years later, or in
1939, Ana de los Reyes died without issue. Nevertheless, before her death, she
had given the subject property to her nephew who had been living with her, the
herein defendant-appellant [private respondent]. The latter immediately took
possession of the property in the concept of owner, built his house thereon and,
through the years, declared the land for taxation purposes as well as paid the
taxes due thereon. His possession of the subject property was never disturbed by
anybody until plaintif-appellee [petitioner] filed the instant complaint against him
on 5 November 1985, or more than 49 years after the deed of donation was
executed.
ISSUE: Whether or not petitioner is barred to recover the property by the doctrine
of laches.
HELD: Yes. Laches means the failure or neglect for an unreasonable and
unexplained length of time, to do that which, by exercising due diligence, could or
should have been done earlier; it is negligence or omission to assert a right within
a reasonable time, warranting the presumption that the party entitled to assert it
either has abandoned or declined to assert it. It has also been defined as such
neglect or omission to assert a right taken in conjunction with the lapse of time
and other circumstances causing prejudice to an adverse party, as will operate as
a bar in equity. The following are the essential elements of laches: (1) Conduct on
the part of the defendant, or of one under whom he claims, giving rise to the
situation complained of; (2) Delay in asserting complainant's right after he had
knowledge of the defendant's conduct and after he has an opportunity to sue; (3)
Lack of knowledge or notice on the part of the defendant that the complainant
would assert the right on which he bases his suit; and (4) Injury or prejudice to the
defendant in the event relief is accorded to the complainant. 32 Under the present
circumstances, all of the aforegoing elements are attendant in this case.
Finally, we agree with the respondent Court of Appeals that, while petitioner is
admittedly still the registered owner of the donated property, and jurisprudence is
settled as to the imprescriptibility and indefeasibility of a Torrens Title, there is
equally an abundance of cases in the annals of our jurisprudence where we
categorically ruled that a registered landowner may lose his right to recover the
possession of his registered property by reason of laches.
Roque v. Intermediate Appellate Court
G.R. No. L-75886, August 30, 1988, 165 SCRA 118
Feliciano, J.
FACTS: Petitioner Concepcion Roque, on 6 December 1977, filed a Complaint for
"Partition with Specific Performance" (docketed as Civil Case No. 5236-M) with
Branch 2 of the then Court of First Instance of Malolos against respondents Emesto
Roque and the heirs of Victor Roque. In her complaint, petitioner (plaintif below)
claimed legal ownership of an undivided three-fourths (3/4) portion of Lot No.
1549, by virtue of the 27 November 1961 "Bilihan Lubos at Patuluyan" executed in
her favor by Emesto Roque and Victor Roque.
In support of this claim, petitioner also presented an undated and unnotarized
"Kasulatang Pagkilala sa Bilihan Patuluyan ng Bahagui at Pagmamana sa Labas ng

62
Hukuman at Paghahati-hati at Abuyan ng Bahagui" said to have been signed by the
respondents in acknowledgment of the existence and validity of the Bilihan in favor
of petitioner. Finally, petitioner alleged that, as a co-owner of Lot No. 1549, she had
a right to seek partition of the property, that she could not be compelled to remain
in the co-ownership of the same. Respondents Ernesto Roque and the legal heirs of
Victor Roque, however, refused to acknowledge petitioner's claim of ownership of
any portion of Lot No. 1549 and rejected the plan to divide the land.
ISSUE: Whether or not petitioner can be compelled to remain in the co-ownership.
HELD: No. Article 494 of the Civil Code provides that "no co-owner shall be obliged
to remain in the co-ownership" and that "each co-owner may demand at any time
the partition of the thing owned in common, insofar as his share is concerned." The
facts on record clearly show that petitioner Concepcion Roque had been in actual,
open and continuous possession of a three-fourths (3/4) portion of Lot No. 1549
ever since execution of the "Bilihan Lubos at Patuluyan" in November of 1961. The
Court notes that it was only in their Answer with Compulsory Counterclaim filed
with the trial court in December of 1977 more than sixteen (16) years later
that respondents first questioned the genuineness and authenticity of the "Bilihan
Lubos at Patuluyan." Not once during those sixteen (16) years did respondents
contest petitioner's occupation of a three-fourths (3/4) portion of Lot No. 1549.
Furthermore, if indeed it is true that respondents, as they claim, are the absolute
owners of the whole of Lot No. 1549, it is most unusual that respondents would
have allowed or tolerated such prolonged occupation by petitioner of a major
portion (3/4) of the land while they, upon the other hand, contented themselves
with occupation of only a fourth thereof. This latter circumstance, coupled with the
passage of a very substantial length of time during which petitioner all the while
remained undisturbed and uninterrupted in her occupation and possession, places
respondents here in laches: respondents may no longer dispute the existence of
the co-ownership between petitioner and themselves nor the validity of petitioner's
claim of a threefourths (3/4) interest in Lot No. 1549, as they are deemed, by their
unreasonably long inaction, to have acquiesced in the co-ownership.
Delima v. Court of Appeals
G. R. No. L-46296, September 24, 1991, 201 SCRA 641
Medialdea J.
FACTS: Lino Delima acquired a lot from the friar lands. Later, he died, leaving as
his only heirs three brothers and sisters namely: Eulalio Delima, Juanita Delima,
Galileo Delima and Vicente Delima. Galileo was the caretaker of the property. He
was able to execute an affidavit adjusting to himself the parcel of land and was
able to secure the issuance of a Transfer Certificate of Title in his name. This
prompted the heirs of his siblings to file a action for reconveyance.
ISSUE: Whether or not the property is subject to prescription.
HELD: Yes. From the moment one of the co-owners claims that he is the absolute
and exclusive owner of the properties and denies the others any share therein, the
question involved is no longer one of partition but of ownership. In such case, the
imprescriptibility of the action for partition can no longer be invoked or applied
when one of the co-owners has adversely possessed the property as exclusive
owner for a period sufficient to vest ownership by prescription. It is settled that
possession by the co-owner or co-heir is that of a trutee. In order that such
possession is considered adverse to the cestui que trust amounting to a
repudiation of the co-ownership, the following elements must concur: 1) that the

63
trustee has performed unequivocal acts amounting to an ouster of cestui que trust;
2) that such positive acts of repudiation had been made known to the cestui que
trust; and 3) that the evidence thereon should be clear and conclusive.
When the co-owner of the property executed a deed of partition and on the
strength thereof, obtained a cancellation of the title in the name of their
predecessor and the issuance of a new title in his name as the owner, the statute
of limitations started to run for the purposes of the action instituted by the latter
seeking a declaration of the existence of the co-ownership and their rights
thereafter. The issuance of a new title constituted a clear act of repudiation of the
trust and co-ownership.

Aguilar v. Court of Appeals


G.R. No. 76351, October 29, 1993, 227 SCRA 472
Bellosillo, J.
FACTS: Petitioner Virgilio and respondent Senen are brothers, and were among the
seven (7) children of the late Maximiano Aguilar. In 1969, the two brothers
purchased a house and lot in Paraaque where their father could spend and enjoy
his remaining years in a peaceful neighborhood. Initially, the brothers agreed that
Virgilio's share in the co-ownership was two-thirds while that of Senen was onethird. By virtue of a written memorandum, Virgilio and Senen agreed that
henceforth their interests in the house and lot should be equal, with Senen
assuming the remaining mortgage obligation of the original owners with the SSS in
exchange for his possession and enjoyment of the house together with their father.
Since Virgilio was then disqualified from obtaining a loan from SSS, the brothers
agreed that the deed of sale would be executed and the title registered in the
meantime in the name of Senen. It was further agreed that Senen would take care
of their father and his needs since Virgilio and his family were staying in Cebu.
After Maximiano Aguilar died in 1974, petitioner demanded from private
respondent that the latter vacate the house and that the property be sold and
proceeds thereof divided among them. Because of the refusal of respondent to
give in to petitioner's demands, the latter filed an action to compel the sale of the
house and lot so that the they could divide the proceeds between them. In his
complaint, petitioner prayed that the proceeds of the sale, be divided on the basis
of two-thirds (2/3) in his favor and one-third (1/3) to respondent. Petitioner also
prayed for monthly rentals for the use of the house by respondent after their father
died. In his answer with counterclaim, respondent alleged that he had no objection
to the sale as long as the best selling price could be obtained; that if the sale
would be efected, the proceeds thereof should be divided equally; and, that being
a co-owner, he was entitled to the use and enjoyment of the property. Rendering
judgment by default against defendant, for failure to appear at pre- trial, the trial
court found him and plaintif to be co-owners of the house and lot, in equal shares
on the basis of their written agreement. However, it ruled that plaintif has been
deprived of his participation in the property by defendant's continued enjoyment of
the house and lot, free of rent, despite demands for rentals and continued
maneuvers of defendants, to delay partition. The trial court also upheld the right of
plaintif as co-owner to demand partition. Since plaintif could not agree to the
amount ofered by defendant for the former's share, the trial court held that this
property should be sold to a third person and the proceeds divided equally
between the parties. The CA set aside the order of the trial court.
ISSUE: Whether or not petitioner may demand partition of the property.

64
HELD: Yes. We uphold the trial court in ruling in favor of petitioner, except as to
the efectivity of the payment of monthly rentals by respondent as co-owner which
we here declare to commence only after the trial court ordered respondent to
vacate in accordance with its order. Article 494 of the Civil Code provides that no
co-owner shall be obliged to remain in the co-ownership, and that each co-owner
may demand at any time partition of the thing owned in common insofar as his
share is concerned. Corollary to this rule, Art. 498 of the Code states that
whenever the thing is essentially, indivisible and the co-owners cannot agree that
it be, allotted to one of them who shall indemnify the others, it shall be sold and its
proceeds accordingly distributed. This is resorted to (1) when the right to partition
the property is invoked by any of the co-owners but because of the nature of the
property it cannot be subdivided or its subdivision would prejudice the interests of
the co-owners, and (b) the co-owners are not in agreement as to who among them
shall be allotted or assigned the entire property upon proper reimbursement of the
co-owners. However, being a co-owner respondent has the right to use the house
and lot without paying any compensation to petitioner, as he may use the property
owned in common long as it is in accordance with the purpose for which it is
intended and in a manner not injurious to the interest of the other co-owners. 9
Each co-owner of property held pro indiviso exercises his rights over the whole
property and may use and enjoy the same with no other limitation than that he
shall not injure the interests of his co-owners, the reason being that until a division
is made, the respective share of each cannot be determined and every co-owner
exercises, together with his co-participants joint ownership over the pro indiviso
property, in addition to his use and enjoyment of the same.
Since petitioner has decided to enforce his right in court to end the co-ownership
of the house and lot and respondent has not refuted the allegation that he has
been preventing the sale of the property by his continued occupancy of the
premises, justice and equity demand that respondent and his family vacate the
property so that the sale can be efected immediately. In fairness to petitioner,
respondent should pay a rental of P1,200.00 per month, with legal interest; from
the time the trial court ordered him to vacate, for the use and enjoyment of the
other half of the property appertaining to petitioner. When petitioner filed an action
to compel the sale of the property and the trial court granted the petition and
ordered the ejectment of respondent, the co-ownership was deemed terminated
and the right to enjoy the possession jointly also ceased. Thereafter, the continued
stay of respondent and his family in the house prejudiced the interest of petitioner
as the property should have been sold and the proceeds divided equally between
them. To this extent and from then on, respondent should be held liable for
monthly rentals until he and his family vacate.
Tomas Claudio Memorial College v. Court of Appeals
G.R. No. 124262, October 12, 1999, 316 SCRA 502
Quisimbing, J.
FACTS: Juan De Castro died intestate in 1993 leaving a parcel of land located in
Morong, Rizal to his heirs. Mariano De Castro one of the heirs sold the said lot to
petitioner Tomas Claudio Memorial College by representing that he is the sole
owner of the property. The other heirs filed an action for partition before the
Regional Trial Court of Rizal alleging that the sale made by Mariano afected only
his undivided share of the lot but not the shares of the other co-owners. Petitioner
filed a motion to dismiss the partition for the reason that it has already been
barred by prescription.
The Regional Trial Court of Rizal dismissed the petitioners motion. The Court of
Appeals affirmed the decision.

65
ISSUES:
1.) Whether or not the sale afected only the undivided share of Mariano
2.) Whether or not the action to file for partition has already prescribed.
HELD: 1.) Yes. The Court has consistently ruled that even if a co-owner sells the
whole property as his, the sale will afect only his own share but not those of the
other co-owners who did not consent to the sale. The sale of the whole property by
a co-owner does not make the sale null and void but it only transfers the rights to
the undivided share of the co-owner who made the sale. The proper action in a
case like this is not nullification nor recovery but a division or partition of the entire
property.
2.) No. As to the issue on prescription, the Civil Code provides that no prescription
shall lie in favor of a co-owner or co-heirs as long as he expressly or impliedly
recognizes the co-ownership.
Robles v. Court of Appeals
GR. No. 123509, March 14, 2000, 328 SCRA 97
Panganiban, J.
FACTS: Leon Robles originally owned the land which was inherited by his son
Silvino Robles. The latter then took possession of the land and declared it in his
name for taxation purposes. Upon his death, the same was inherited by his widow
Maria dela Cruz and his children. The plaintifs entrusted the payment of the land
taxes to their co-heir and half-brother, Hilario Tobles. For unknown reasons, the tax
declaration of the parcel of land in the name of Silvino Robles was cancelled and
transferred to one Exequiel Ballena, father of Andres Robles who is the wife of the
defendant Hilario Robles. He secured a loan from the Cardona Rural Bank, Inc.
which was foreclosed for failure to pay the mortgage debt wherein the defendant
bank emerged as the highest bidder during the auction sale. Defendant Rural Bank
sold the same to the Spouses Santos. A n action for quieting of title was filed by
respondent Santos. The plaintifs alleged that they had been in possession of the
land since 1942 and it was only in 1987 that they knew about the foreclosure of
the mortgage. The Court of Appeals ruled that because of the plaintifs inaction for
more than 20 years, prescription had already set in.
ISSUE: Whether or not the action has prescribed in favour of Hilario Robles.
HELD: Yes. Hilario efected no clear and evident repudiation of the co-ownership. It
is a fundamental principle that a co-owner cannot acquire by prescription the share
of the other co-owners, absent any clear repudiation of the co-ownership. In order
that the title may prescribe in favor of a co-owner, the following requisites must
concur: (1) the co-owner has performed unequivocal acts of repudiation amounting
to an ouster of the other co-owners; (2) such positive acts of repudiation have
been made known to the other co-owner; and (3) the evidence thereof is clear and
convincing. In the present case, Hilario did not have possession of the subject
property; neither did he exclude the petitioners from the use and the enjoyment
thereof, as they had indisputably shared in its fruits. Likewise, his act of entering
into a mortgage contract with the bank cannot be construed to be a repudiation of
the co-ownership. As absolute owner of his undivided interest in the land, he had
the right to alienate his share, as he in fact did. Neither should his payment of land
taxes in his name, as agreed upon by the co-owners, be construed as a repudiation
of the co-ownership. The assertion that the declaration of ownership was
tantamount to repudiation was belied by the continued occupation and possession
of the disputed property by the petitioners as owners.

66

Rizal Cement Co., Inc. v. Villareal


G.R. No. L-30272, February 28, 1985, 135 SCRA 15
Cuevas, J.
FACTS: Respondents are applicants for the registration of two agricultural lands
located in Rizal. They presented testimonial and documentary evidence appearing
that the property applied for, designated as Lot Nos. 1 and 2 of Plan Psu-147662,
have a total area of 26,015 sq. m.; that these lots originally belong to one Maria
Certeza; that upon her death, the property was involved in a litigation between her
grandchildren and Gonzalo Certeza, and that the lots were given by the latter to
Justice de Joya as the latters attorneys fees; that the lots were then sold by de
Joya to Filomeno Sta. Ana, who in turn sold the same to spouses Victoriano Cervo
and Ignacia Guillermo in 1939; that sometime in November 1955, the said spouses
sold the lots to herein applicants as shown by a duly notarized deed of sale. The
spouses Cervo declared the property for taxation purposes in the name of the wife,
Ignacia Guillermo, and paid for the realty taxes thereon; that prior to the sale, the
spouses Cervo had the two lots surveyed first in 1950 and then in 1955. On the
other hand, oppositor (Rizal Cement Company) claims to be the owner of the
subject lots, having bought the same from Maria Certeza, and to have been in
continuous and adverse possession of the property since 1911. To substantiate this
claim, petitioner submitted documentary evidence, one of which is a tax
declaration of the said lots. The Court of First Instance denied the application for
registration of respondents and ordered the issuance of a decree of registration in
the name of Rizal Cement Co., after finality of said decision. On appeal, the Court
of Appeals reversed and set aside the decision of the CFI. The CA denied
petitioners motion for reconsideration. Hence, this petition was filed.
ISSUE: Whether or not respondents had been in actual possession of the land in
question.
HELD: Yes. The CA gave credence to the testimony of the witnesses for
respondents. As a general rule, it is provided in the Civil Code that possession is
acquired by the material occupation of a thing or the exercise of a right or by the
fact that it is subject to the action of our will, or by the proper acts or legal
formalities established for acquiring such right. Petitioners evidence, consisting of
tax receipts, tax declaration and survey plan are not conclusive and indisputable
basis of ones ownership of the property in question. Assessment alone is of little
value as proof of title. Mere tax declaration does not vest ownership of the
property upon defendant.
Wong v. Carpio
G.R. No. 50264, October 21, 1991, 203 SCRA 118
Bidin, J.
FACTS: William Giger sold a parcel of land through a pacto de recto sale to Manuel
Mercado. Mercado only began to harvest the coconut fruits but he never placed
anyone over the land to watch it. Neither did he reside in the land nor was there
any hut constructed thereon to show possession. Thereafter, Ignacio Wong
inspected the land to see if whether there was anyone claiming the land. After
finding there was none, he bought the land from Giger. He placed workers on the
land, constructed a farmhouse, and fenced the boundaries. He couldn't register the
sale due to some technicalities.

67
ISSUE: Whether or not the possession of the disputed land belongs to Ignacio
Wong.
HELD: It should be stressed that "possession is acquired by the material
occupation of a thing or the exercise of a right, or by the fact that it is subject to
the action of our will, or by the proper acts and legal formalities for acquiring such
right." And that the execution of a sale thru a public instrument shall be equivalent
to the delivery of the thing, unless there is stipulation to the contrary. If, however,
notwithstanding the execution of the instrument, the purchaser cannot have the
enjoyment and material tenancy of the thing and make use of it herself, because
such tenancy and enjoyment are opposed by another, then delivery has not been
efected. Applying the above pronouncements on the instant case, it is clear that
possession passed from vendor William Giger to private respondent Manuel
Mercado by virtue of the first sale a retro, and accordingly, the later sale a retro in
favor of petitioner failed to pass the possession of the property because there is an
impediment the possession exercised by private respondent. Possession as a
fact cannot be recognized at the same time in two diferent personalities except in
the cases of co-possession. Should a question arise regarding the fact of
possession, the present possessor shall be preferred; if there are two possessions,
the one longer in possession, if the dates of possession are the same, the one who
presents a title; and if these conditions are equal, the thing shall be placed in
judicial deposit pending determination of its possession or ownership through
proper proceedings.
Somodio v. Court of Appeals
G.R. No. 82680, August 15, 1994, 235 SCRA 307
Quiason, J.
FACTS: Wilfredo Mabugat and Nicanor Somodio bought a residential lot situated at
Rajah Muda, Bula, General Santos. Petitioner and Mabugat partitioned the property
into two portions, with petitioner taking the western part. Immediately after the
partition, petitioner took possession of his portion and planted thereon ipil-ipil
trees, coconut trees and other fruit-bearing trees. In 1976, petitioner began
construction of a structure with a dimension of 22-by-18 feet on his lot. His
employment, however, took him to Kidapawan, North Cotabato, and he left the
unfinished structure to the case of his uncle. He would visit the property every
three months or on weekened when he had time. Sometime in October 1977,
petitioner allowed respondent Felomino Ayco, to transfer his hut to petitioner's lot.
About six years later, petitioner demanded that Ayco vacate the premises but such
demand proved futile. Hence, on August 23, 1983, petitioner filed an action for
unlawful detainer with damages against respondent Ayco. Meanwhile, on June 26,
1983, respondent Ebenecer Purisima entered the land and constructed a house
thereon. Four days later, petitioner filed against respondent Purisima a complaint
for forcible entry before the same court docketed as Civil Case No. 2013-I. Said
case was later consolidated with Civil Case No. 2032-II.
ISSUE: Whether or not Somodio has actual possession of the property.
HELD: Yes. Article 531 of the Civil Code of the Philippines provides that possession
is acquired by the material occupation of a thing or the exercise of a right, or by
the fact that it is subject to the action of our will, or by the proper acts and legal
formalities established for acquiring such right. Petitioner took possession of the
property sometime in 1974 when he planted the property to coconut trees, ipil- ipil
trees and fruit trees. In 1976, he started the construction of a building on the
property. It is immaterial that the building was unfinished and that he left for
Kidapawan for employment reasons and visited the property only intermittently.

68
Possession in the eyes of the law does not mean that a man has to have his feet on
every square meter of ground before it can be said that he is in possession (Ramos
v. Director of Lands, 39 Phil. 175 [1918]). It is sufficient that petitioner was able to
subject the property to the action of his will.
Maglucot Aw v. Maglucot
G.R. No. 132518, March 28, 2000, 329 SCRA 78
Kapunan, J.
FACTS: Sometime in 1946 there was a prior oral agreement to tentatively partition
Lot No. 1639. By virtue of this agreement, the original co-owners occupied specific
portions of Lot No. 1639. It was only in 1952 when the petition to subdivide Lot No.
1639 was filed because two of the co-owners, namely Hermogenes Olis and heirs
of Pascual Olis, refused to have said lot subdivided and have separate certificates
of title. Significantly, after the 1952 proceedings, the parties in this case by
themselves and/or through their predecessors-in-interest occupied specific portions
of Lot No. 1639 in accordance with the sketch plan. Sometime in 1963, Guillermo
Maglucot rented a portion of the subject lot. Subsequently, Leopoldo and Severo,
both surnamed Maglucot, rented portions of subject lot in 1964 and 1969,
respectively, and each paying rentals therefor. Said respondents built houses on
their corresponding leased lots. They paid the rental amount of P100.00 per annum
to Mrs. Ruperta Salma, who represented the heirs of Roberto Maglucot, petitioners
predecessor-in-interest. In December 1992, however, said respondents stopped
paying rentals claiming ownership over the subject lot alleging they had a right
over the land because such was not partitioned and they were co-owners.
Manglucot-Aw thus filed a complaint for recovery of possession and damages
against Manglucot.
ISSUE: Whether or not Manglucot-Aw may recover possession by virtue of a valid
partition.
HELD: Yes. An order for partition is final and not interlocutory and, hence,
appealable because it decides the rights of the parties upon the issue submitted. In
this case, both the order of partition and the unconfirmed sketch plan are, thus,
interlocutory. Nevertheless, where parties do not object to the interlocutory decree,
but show by their conduct that they have assented thereto, they cannot thereafter
question the decree, especially, where, by reason of their conduct, considerable
expense has been incurred in the execution of the commission. Respondents in this
case have occupied their respective lots in accordance with the sketch/subdivision
plan. They cannot after acquiescing to the order for more than forty (40) years be
allowed to question the binding efect thereof. Under the present rule, the
proceedings of the commissioners without being confirmed by the court are not
binding upon the parties. However, this rule does not apply in case where the
parties themselves actualized the supposedly unconfirmed sketch/subdivision plan.
The purpose of court approval is to give efect to the sketch/subdivision plan. In
this case, the parties themselves or through their predecessors-in-interest
implemented the sketch plan made pursuant to a court order for partition by
actually occupying specific portions of Lot No. 1639 in 1952 and continue to do so
until the present until this case was filed, clearly, the purpose of the court approval
has been met. This statement is not to be taken to mean that confirmation of the
commissioners may be dispensed with but only that the parties herein are
estopped from raising this question by their own acts of ratification of the
supposedly non-binding sketch/subdivision plan.
Cequea v. Bolante

69
G.R. No. 137944, April 6, 2000, 330 SCRA 216
Panganiban, J.
FACTS: The petitioners Fernanda Mendoza Cequea and Eduarda Apiado sought
for the ownership and possession of the land occupied by the respondent Honorata
Bolante. Prior to 1954, the land in Binangonan, Rizal was declared for taxation
purposes in the name of Sinforoso Mendoza, the father of respondent. Sinforoso
died in 1930. On the basis of an affidavit, the tax declaration in the name of
Sinforoso Mendoza of the contested lot was cancelled and subsequently declared
in the name of Margarito Mendoza, the father of the petitioners. Margarito and
Sinforoso are brothers. During the cadastral survey, respondent Honorata is the
present occupant of the land together with Miguel Mendoza, another brother of the
petitioners. The trial court rendered the petitioners as the lawful owner and
possessors of the land. However, the Court of Appeals reversed the decision
because the genuineness and the due execution of the affidavit. It was said to be
insufficient to overcome the denial of respondent and her mother. Moreover, the
probative value of petitioners tax receipts and declarations paled in comparison
with respondents proof of ownership of the disputed parcel. The actual, physical,
exclusive and continuous possession by respondent since 1985 gave her a better
title under Article 538 of the Civil Code. The petitioners contended otherwise that
she came into possession through force and violence, contrary to Article 536 of the
Civil Code.
ISSUES:
1.) Whether or not the respondent has the actual, physical, exclusive and
continuous possession of the land.
2.) Whether or not tax declarations and receipts are conclusive evidence of
ownership or possession.
HELD: 1.) Yes. Possession by the petitioner before 1985 was not exclusive, as the
respondent also acquired it before 1985. The records show that the petitioners
father and brother, as well as the respondent and her mother were simultaneously
in adverse possession of the land. Based on Article 538 of the Civil Code, the
respondent is the preferred possessor because, benefitting from her fathers tax
declaration of the subject lot since 1926, she has been in possession thereof for a
longer period. On the other hand, petitioners father acquired joint possession only
in 1952.
2.) No. Tax declarations and receipts are not conclusive evidence of ownership. At
most, they constitute mere prima facie proof of ownership or possession of the
property for which taxes have been paid. In the absence of actual public and
adverse possession, the declaration of the land for tax purposes does not prove
ownership. The petitioners claim of ownership of the whole parcel has no legal
basis.
Aragon v. Insular Government
G.R. No. L-6019, March 25, 1911, 19 Phil. 223
Carson, J.
FACTS: The Government of the Philippine Islands, through its proper
representatives, objected to the application for registration pursuant to the Land
Registration Act of a small lot of parcel of land being instituted by herein petitioner,
Juan Aragon on the ground that said land forms part of the public domain applying
the provisions of subsection 1 of Article 339 of the old Civil Code, now Article 420,
paragraph 1 of the New Civil Code which provides that the following things are
property of public dominion: (1) Those intended for public use, such as roads,

70
canals, rivers, torrents, ports and bridges constructed by the State, banks, shores,
roadstead, and others of similar character. It appears, however, that possessory
title over the land in question was duly registered in favor of petitioner, and that
the applicant and their predecessors in interest have been in possession of the
parcel of land in question, under an undisputed claim of ownership. That there are
strong reasons to believe that the land in question was originally well above the
ebb and flow of the tide and only in later years have the waters risen to such a
height along the shores of the Bay of Manila at this point as to cover the land in
question completely at high tide, though, it cannot be ascertained definitely
whether it is due to changes in the current and flow of the waters in the bay, or to
the gradual sinking of the land along the coast.
ISSUE: Whether or not petitioner is entitled ownership over the land in question.
HELD: The Court affirmed the decree entered by the lower court in favor of
petitioner applying the provisions of Article 446 of the old Civil Code, Article 539 of
the New Civil Code which provides that every possessor has a right to be protected
in his possession; and should he be disturbed therein, he shall be protected in or
restored to said possession by the means established by the laws and the Rules of
Court. Corollary, a possessor may lose his possession under the circumstances
provided under Article 555 of the New Civil Code, to wit: (1) By the abandonment
of the thing; (2) By an assignment made to another either by onerous or gratuitous
title; (3) By the destruction or total loss of the thing, or because it goes out of
commerce; and (4) By the possession of another, subject to the provisions of
Article 537, if the new possession has lasted longer than one year. But the real
right of possession is not lost till after the lapse of ten years. The Court held that
since the foregoing enumerations with respect to the loss of possession was not
conclusively established by the representatives of the government, and the fact
that the owners of the land in question have never intended to abandon the same,
then it is just and proper to register said land in their name.
Catholic Vicar Apostolic of the Mountain Province v. Court of Appeals
G.R. No. 80294, March 23, 1990, 183 SCRA 639
Gancayco, J.
FACTS: CA-G.R. No. 38830-R was a land registration case where petitioner and
private respondents were asking for confirmation of their alleged imperfect titles to
the lots in question under Section 49 (b) of the Public Land Act. In the said
decision, the appellate court found that the petitioner was not entitled to
confirmation of its imperfect title to Lots 2 and 3. In separate motions for
reconsideration filed by private respondents Heirs of Octaviano and Heirs of Juan
Valdez relating to the same decision, they also asked that said two lots be
registered in their names. On August 12, 1977, the Court of Appeals denied both
motions. Efectively, therefore, in the said decision the appellate court ruled that
neither the petitioner nor the private respondents are entitled to the confirmation
of imperfect title over said two lots. Pursuant to the said decision in CA-G.R. No.
38830-R, the two lots in question remained part of the public lands. This is the only
logical conclusion when the appellate court found that neither the petitioner nor
private respondents are entitled to confirmation of imperfect title over said lots.
The present actions that were instituted in the Regional Trial Court by private
respondents are actions for recovery of possession (accion publiciana) and not for
recovery of ownership (accion reivindicatoria).
ISSUE: Whether or not petitioner is entitled to the possession of the subject lots.

71
HELD: Yes. Under Article 555 (4) of the Civil Code, it is provided that a possessor
may lose his possession by the possession of another, subject to the provisions of
Article 537, if the new possession has lasted longer than one year. But the real
right of possession is not lost till after the lapse of ten years. In the case at bar, it
is clear that the petitioner was in possession of the said property as borrower in
commodatum from private respondents since 1906. However, in 1951 petitioner
repudiated the trust when it declared the property for tax purposes under its
name. Thus, when petitioner filed its application for registration of the said
property in 1962, it had been in adverse possession of the same for at least 11
years. Hence, the action for recover of possession of said property filed by private
respondents against petitioner must fail. The Court, therefore, finds that the trial
court and the Court of Appeals erred in declaring the private respondents to be
entitled to the possession thereof. Much less can they pretend to be owners
thereof. Said lots are part of the public domain.
EDCA Publishing & Distributing Corp. v. Santos,
G.R. No. 80298, April 26, 1990, 134 SCRA 614
Cruz, J.
FACTS: Jose Cruz ordered by telephone 406 books from EDCA Publishing and
Distributing Corp. (EDCA), payable on delivery. EDCA prepared the corresponding
invoice and delivered the books as ordered, for which Cruz issued a check.
Subsequently, Cruz sold 120 of the books to Leonor Santos who paid him after
verifying the seller's ownership from the invoice he showed her. Meanwhile, EDCA
having become suspicious over a second order placed by Cruz even before clearing
of his first check, made inquiries with the De la Salle College where he had claimed
to be a dean and was informed that there was no such person in its employ.
Further, Cruz had no account with the Philippine Amanah Bank, against which he
had drawn the check. EDCA went to the police, which arrested Cruz whose real
name was Tomas de la Pea. EDCA sought the assistance of the police, and forced
their way into the store of the Santos and threatened her with prosecution for
buying stolen property. They seized the 120 books. Santos sued for recovery of the
books after demand for their return was rejected by EDCA.
ISSUES:
1.) Whether or not EDCA was unlawfully deprived of the books because the check
issued by the impostor in payment therefor was dishonored.
2.) Whether or not EDCA had the right to cease the books that were sold to Santos.
HELD: 1.) No. EDCA was not unlawfully deprived of the books. Article 559 of the
Civil Code provides that the possession of movable property acquired in good faith
is equivalent to a title. Nevertheless, one who has lost any movable or has been
unlawfully deprived thereof, may recover it from the person in possession of the
same. If the possessor of a movable lost or of which the owner has been unlawfully
deprived has acquired it in good faith at a public sale, the owner cannot obtain its
return without reimbursing the price paid therefor. A contract of sale is perfected
once agreement is reached between the parties on the subject matter and the
consideration. Ownership in the thing sold shall not pass to the buyer until full
payment of the purchase only if there is a stipulation to that efect. Otherwise, the
rule is that such ownership shall pass from the vendor to the vendee upon the
actual or constructive delivery of the thing sold even if the purchase price has not
yet been paid. Non-payment only creates a right to demand payment or to rescind
the contract, or to criminal prosecution in the case of bouncing checks. But absent
the stipulation above noted, delivery of the thing sold will efectively transfer
ownership to the buyer who can in turn transfer it to another.

72
2.) No. Actual delivery of the books having been made, Cruz acquired ownership
over the books which he could then validly transfer to the private respondents. The
fact that he had not yet paid for them to EDCA was a matter between him and
EDCA and did not impair the title to the books acquired by the Santos spouses.
Therefore, EDCA was not unlawfully deprived of the books and Santos had rights
over the books.
De Garcia v. Hon. Court of Appeals
G.R. No. L-20264, January 30, 1971, 37 SCRA 129
Fernando, J.
FACTS: On October 11, 1953, Angelina Guevarra, while talking to Consuelo de
Garcia, recognized her ring in the finger of the latter which she lost sometime in
February 1952. Guevarra asked where de Garcia bought the ring to which de
Garcia answered that she bought it from her comadre. Guevarra explained to de
Garcia that that ring was the very same ring stolen from her. De Garcia handed the
ring to Guevarra and the ring fitted her finger. Two or three days later, at the
request of Guevarra, she, her husband Lt. Col. Juan Guevara, Lt. Cementina of
Pasay PD, de Garcia and her attorney proceeded to the store of Mr. Rebullida to
whom they showed the ring in question. Mr. Rebullida examined the ring with the
aid of high power lens and after consulting the stock card thereon, concluded that
it was the very ring that plaintif bought from him in 1947. The ring was returned to
defendant who despite a written request therefor failed to deliver the ring to
plaintif. In trial, de Garcia said that she bought the ring from Mrs. Miranda who got
it from Mrs. Angelita Hinahon who in turn got it from, Aling Petring who was
boarding in her house.
ISSUE: Whether or not de Garcias possession of the ring in good faith confers her
title to the said ring.
HELD: No. The controlling provision is Article 559 of the Civil Code which provides
that possession of movable property acquired in good faith is equivalent to a title.
Nevertheless, one who has lost any movable or has been unlawfully deprived
thereof may recover it from the person in possession of the same. If the possessor
of a movable lost of which the owner has been unlawfully deprived, has acquired it
in good faith at a public sale, the owner cannot obtain its return without
reimbursing the price paid therefor. Respondent Angelina D. Guevara, having been
unlawfully deprived of the diamond ring in question, was entitled to recover it from
petitioner Consuelo S. de Garcia who was found in possession of the same. The
only exception the law allows is when there is acquisition in good faith of the
possessor at a public sale, in which case the owner cannot obtain its return without
reimbursing the price. The common law principle that where one of two innocent
persons must sufer by a fraud perpetrated by the another, the law imposes the
loss upon the party who, by his misplaced confidence, has enabled the fraud to be
committed, cannot be applied in a case which is covered by an express provision of
the new Civil Code, specifically Article 559. Between a common law principle and
statutory provision, the latter must prevail in this jurisdiction. It is thus
immediately apparent that there is no merit to the contention raised in the first
assigned error that her possession in good faith, equivalent to title, sufficed to
defeat respondent Guevara's claim. As the above cases demonstrate, even on that
assumption the owner can recover the same once she can show illegal deprivation.
Respondent Court of Appeals was so convinced from the evidence submitted that
the owner of the ring in litigation is such respondent.
Dizon v. Suntay

73
G.R. No. L-30817, September 29, 1972, 47 SCRA 160
Fernando, J.
FACTS: Lourdes Suntay is the owner of a 3 carat diamond ring. She entered into a
transaction with Clarita Sison, wherein said ring was delivered to the latter for sale
on commission. Upon receiving the ring, the receipt was delivered to Suntay. After
a lapse of a considerable amount of time, the ring was not yet returned and so
Suntay demanded for its return from Sison but the latter could not comply as she
had already pledged it with Dizons pawnshop for P 2,600.00. After insistent
demands, Sison delivered the pawnshop ticket to Suntay. Suntay through her
counsel, wrote to Dizon asking for the delivery of the ring pledged but, the latter
refused. She filed an action for recovery with P 500 as attorneys fees and costs.
She asked for the remedy of replevin upon filing the requisite bond pending final
determination of the action. The CFI of Manila issued the writ and Suntay was able
to regain possession during the pendency of the action. The lower court rendered a
decision in favor of Suntay. On appeal, Dizon sought the reversal of the lower
courts decision and invoking estoppel. CA affirmed the lower courts decision. SC
affirmed CA decision.
ISSUE: Whether or not the owner of the ring may recover its possession from the
pawnshop owner.
HELD: Yes. Owner of a diamond ring may recover the possession of the same from
a pawnshop where another person had pledged it without authority to do so. Art.
559 of the civil code applies and the defense that the pawnshop acquired
possession of the without notice of any defect in the title of the pledgor is
unavailing. Neither the promptings of equity nor the mandates of moral right and
natural justice come to his rescue. Dizon is engaged in a business where
presumably ordinary prudence would manifest itself to ascertain whether or not an
individual who is ofering a jewelry by way of a pledge is entitled to do so. If no
such care be taken he should be the last to complain if thereafter the right of the
true owner of such jewelry should be recognized.
Ledesma v. Court of Appeals
G.R. No. 86051, September 1, 1992, 213 SCRA 195
Davide, J.
FACTS: Two motor vehiclesHonda Gemini and Holden Premiere Modelwere
purchased from Citiwide Motors by a person who identified himself as
Jojo Consunji. He bought the vehicles purportedly for his father. Upon delivery to
him of the vehicles, he paid a managers check drawn against PCIB. The check
though was dishonored by the bank on the ground that the checks value has been
materially altered. This was reported to the police authorities and it was found out
that the person misrepresenting himself was actually Suarez who had a long line of
criminal cases against him for his modus operandi. The Holden car was recovered
after being abandoned somewhere in Quezon City. The Honda on the other hand,
was discovered to be sold to Ledesma. Ledesma averred he purchased the vehicle
in good faith from one Neyra, as evidenced by his certificate of registration.
Citiwide Motors was able to recover.
ISSUE: Whether or not CITIWIDE MOTORS has been unlawfully deprived.
HELD: No. There was a perfected unconditional contract of sale between Citiwide
Motors and Suarez. The subsequent dishonor of the check merely amounted to
failure of consideration which doesn't render a contract of sale void, but merely
allows the prejudiced party to sue for specific performance or rescission of the

74
sale. This being the case, Citiwide motors wasn't unlawfully deprived of the
property. It is thus not entitled to the return of the vehicle from Ledesma who
bought the property in good faith and for consideration.
Azarcon and Abobo v. Eusebio
G.R. No. L-11977, April 29, 1959, 105 SCRA 569
Labrador, J.
FACTS: Victor Eusebio had a dispute over a parcel of land with Leonardo Azarcon,
Manuel Azarcon and Esteban Abobo. Eusebio filed a lease application for a parcel
of land, a portion thereof was occupied by Azarcon et al. under a homestead
application. Before the dispute could be settled, Eusebio filed a complaint in the
CFI of Nueva Ecija, alleging that he had acquired a big parcel of land by lease from
the Bureau of Lands, and that while he was in possession thereof, Azarcon et al.
occupied a portion. The trial court ruled in favor of Eusebio, and a writ of execution
ordering Azarcon et al. to restore possession of the land to Eusebio was issued on
October 3, 1955. However, in spite of the receipt of the notice of writ of execution,
Azarcon et al. nevertheless entered the land to gather palay which was then
pending harvest.
ISSUE: Whether or not Azarcon and Abobo are entitled to the pending fruits of the
land.
HELD: Yes. While the court order of October 3, 1955 ordered them to move out of
the premises, it did not prohibit them from gathering the crop then existing
thereon. Under the law, a person, who is in possession and who is being ordered to
leave a parcel of land while products thereon are in pending harvests, has the right
to a part of the net harvest, as expressly provided by Article 545 of the Civil Code.
Hence, as the order of execution did not expressly prohibit Azarcon et al. from
gathering the pending fruits, which fruits were the result of their possession and
cultivation of the land, it cannot be said that they committed an act which is clear
violation of the courts order.
Cordero v. Cabral
G.R. No. L-36789, July 25, 1983, 123 SCRA 532
Abad Santos, J.
FACTS: Mr. Gregorio Z. Ocampo of Meycauayan, Bulacan, husband of the plaintif
Felipa Cordero and father of the other plaintifs surnamed Ocampo, died on May
17, 1958. The said deceased left several properties, which were inherited by the
plaintifs including the land in question which parcel of land was originally
registered in accordance with the Land Registration Act on December 14, 1933,
and was registered and/or transferred in the name of Mr. Gregorio Z. Ocampo on
July 31, 1934. After the death of the said Mr. Gregorio Z. Ocampo, the plaintifs
herein took possession of the said parcel of land which is a riceland, but they found
out that the southern portion of the same with an area 4,303 square meters, more
or less, upon verification, was possessed by the defendants herein, Victoria P.
Cabral, Alejandro Berboso and Dalmacio Montaos. Victoria P. Cabral claimed to be
the owner of said portion while her co-defendants co-possessed the same as her
tenants. The plaintifs demanded of the defendants to surrender to the former
possession of the portion of land and/or vacate it but they refused and failed to do
so, and the defendant Victoria P. Cabral continued claiming to be the owner of the
same while her co-defendants continued recognizing her as the owner thereof
instead of the plaintifs. Plaintifs alleged that because of the defendants'
occupancy of the aforementioned plaintifs' portion of land with the area of 4,303

75
square meters, more or less, to the exclusion of the latter, the said plaintifs failed
to realize a yearly harvest of at least ten (10) cavanes of palay at the rate of
P10.00 per cavan, from the harvest-time of 1958 up to the present.
ISSUE: Whether or not the defendants must reimburse the fruits receive.
HELD: Yes. The disputed land is included in T.C.T. No. 14513 issued to Gregorio Z.
Ocampo, the predecessor of the plaintifs. The original registration which includes
the disputed land was not vitiated by error or fraud. The defendants, by their own
admission, are in possession of the disputed land. There is no evidence that they
were possessors in bad faith. However, their good faith ceased when they were
served with summons to answer the complaint. As possessors in bad faith from the
service of the summons they "shall reimburse the fruits received and those which
the legitimate possessor could have received.
Solid Homes Inc v Spouses Tan
G.R. No. 145156-57
FACTS:
1. In 1980, Petitioner Solid Homes sold to Sps. Uy a 1069sqm lot in their QC
subdivision project. Said lot was registered in the name of Uys and TCT was
in their name
2. Afterwards, the Uys sold it to herein Respondent Sps. Tan in 1985, TCT
transferred to them subsequently.
3. Afterwards, after several visits to their property, Respondents found out the
sad state of the development of the property. There was no infrastructure or
utility systems as announced in the approved plans and advertisement of the
subdivision project, and squatters occupy the property and its surrounding
areas.
4. Respondents then demanded Petitioners in a letter dated 1995, to provide
the promised developments for the project and rid their property of the
squatters conformably with PD 957
5. Petitioner did not reply, hence, in 1996, they complained before the HLURB
Arbiter with the same prayers.
a. HLURB Arbiter ruled IFO Respondents
b. Petitioner appealed to HLURB Board, but the Arbiter was affirmed.
c. Petitioner appealed to OP, but affirmed HLURB with modification that if
they cannot deliver the prayers, they should pay back the Respondents
with the purchase price plus interests
6. Both parties appealed to CA. Respondents appeal because the payment
should at least be based on fair market value and not purchase price.
Petitioners appeal is for obvious reasons, kasi natalo parin sila.
a. CA modified OP decision, payment should be based on fair market
value.
7. Hence, this petition
ISSUE: Whether respondents action has prescribed
HELD: No. Petitioners claim that the action has prescribed because more than 10
years has lapsed since the sale in 1980 or the subsequent sale in 1985 up to the
filing of the case in 1996.
SC disagrees. While it is true that Art 1144 of NCC provides that 10 years is the
prescriptive period to which an action should be brought upon, it must be counted
when the cause of action has arisen. Art 1144 says that an action must be brought
within ten years upon a written contract, upon an obligation created by law, or

76
upon a judgment. If not on a written contract, it must be upon an obligation
created by law. According to PD 957, the developer has the obligation to provide
adequate utilities.
Citing Banco Filipino Savings v CA, a cause of action arises when that which should
not have been done is done; or that which should have been done is not done.
The elements of cause of action are (1) right of the applicant, (2) obligation of
defendant to respect such right, (3) act or omission of defendant that violates the
applicants right. It is only upon the happening of the last element that a cause of
action arises.
In this case, it was only when the Respondents demanded in 1995 did the cause of
action arise. Also in SSS v Moonwalk, an obligor violates his obligation from the
time the obligee demands. Absent any demand thereto, the obligor does not incur
delay. As long as he is not in delay, he cannot be guilty of some violation of the
obligees rights. As a result, the prescriptive period does not run until demand is
made.
Mendoza and Enriquez v. De Guzman
G.R. No. L-28721, October 5, 1928, 52 Phil. 164
Malcolm, J.
FACTS: In the cadastral proceedings of the municipality of Sariaya, Tayabas, a
piece of land identified as lot No. 687 was adjudicated in favor of Martin Mendoza
and Natalio Enriquez in equal parts pro indiviso subject to the right of retention on
the part of Manuel de Guzman until he shall have been indemnified for the
improvements existing on the land. Mendoza has possessed it since 1916. By
virtue of this judgment, De Guzman presented a motion requesting the issuance of
a writ of possession for lot No. 687 in his favor which was granted on June 25,
1924. Since then De Guzman has had dominion over the land. Being unable to
come to an agreement as to the amount which should be allowed for the
improvements made on the land, Martin Mendoza and Natalio Enriquez began an
action requesting the court to (a) fix the value of the necessary and useful
expenses incurred by Manuel de Guzman in introducing the improvements; (b)
require the defendant to render an accounting of the fruits received by him and
order that the value of the fruits be applied to the payment of the necessary and
useful expenses; and (c) decree the restitution of the possession to the plaintifs.
Max. B. Solis, one of the persons who were ejected from the land, asked leave to
intervene, alleging, among other things, that De Guzman had transferred all his
rights in the improvements and in the lot to him with the exception of two hundred
coconut trees. This petition was granted. At the trial which followed and at the
instance of the parties, two commissioners were appinted with instructions to
inspect the land and to count the number of coconut trees planted thereon,
determining the number of fruit-bearing trees and those that are not fruit-bearing
as well as the condition of the same. After trial, Judge of First Instance Gloria
rendered judgment declaring (a) that the defendant Manuel de Guzman and the
intervenor Bernardo Solis have the right to collect from the plaintifs Martin
Mendoza and Natalio Enriquez the sum of P2,046 as compensation for the
necessary and useful expenditures in the proportion of 20 per cent for Manuel de
Guzman and 80 per cent for Bernardo Solis; and (b) that Manuel de Guzman and
Bernardo Solis are obliged to pay to the plaintifs the sum of P666.93 per annum
from June 25, 1924, one-fifth of this amount to be paid by Manuel de Guzman and
the other four-fifths by Bernardo Solis. As on the date when this judgment was
rendered, that is on September 23, 1927, the amount that the plaintifs were
required to pay to the defendant and intervenor exceeded the amount that the

77
latter were to pay the former, the defendant and intervenor were ordered to
deliver the land and its improvement as soon as the plaintifs have paid the
diference, without special pronouncement as to costs.
ISSUE: Whether or not the trial court correctly declared the amount to be paid as
"indemnizacion" in the form of necessary and useful expenditures incurred by the
defendant.
HELD: Yes. Article 361 of the Civil Code in the original Spanish text uses the word
"indemnizacion." However one may speculate as to the true meaning of the term
"indemnizacion" whether correctly translated as "compensation" or "indemnity,"
the amount of the "indemnizacion" is the amount of the expenditures mentioned in
articles 453 and 454 of the Civil Code, which in the present case is the amount of
the necessary and useful expenditures incurred by the defendant. Necessary
expenses have been variously described by the Spanish commentators as those
made for the preservation of the thing; as those without which the thing would
deteriorate or be lost; as those that augment the income of the things upon which
they are expanded. Among the necessary expenditures are those incurred for
cultivation, production, upkeep, etc. Here the plaintifs have chosen to take the
improvements introduced on the land and are disposed to pay the amount of the
necessary and useful expenses incurred by the defendant. Inasmuch as the
retentionist, who is not exactly a posessor in good faith with in the meaning of the
law, seeks to be reimbursed for the necessary and useful expenditures, it is only
just that he should account to the owners of the estate for any rents, fruits, or
crops he has gathered from it.
Robles and Martin v. Lizarraga Hermanos
G.R. No. L-16736, December 22, 1921, 42 Phil. 584
Romualdez, J.
FACTS: Anastasia de la Rama died on the 17th of October, 1916, leaving six
children, to wit, Magdalena, Jose, Evarista, Zacarias, Felix, and Purificacion,
surnamed Robles, and some properties, among which is house No. 4 on Iznart
Street in the city of Iloilo. The children and heirs of Anastasia de la Rama entered
into partnership with Lizarraga Hermanos in liquidation and settlement of their
accounts, by virtue of which the competent court awarded to said partnership the
properties left by the deceased, including the aforesaid house No. 4 on Iznart
Street. Evarista Robles, one of the heirs, since before the death of her mother
Anastasia de la Rama, has been with her husband occupying the aforesaid house
No. 4 on Iznart Street, at the beginning, by permission of her mother, later on by
the consent of her coheirs, and lastly by agreement with the partnership, Lizarraga
Hermanos, to whom it had been awarded, having made some improvements on
the house, the value of which is fixed at four thousand five hundred pesos
(P4,500), and paying to said partnership forty pesos (P40) monthly as rent of the
upper story. On March 18, 1918, Lizarraga Hermanos notified Evarista Robles
(Exhibit J) that beginning April next the rent of the upper story of the house would
be raised to sixty pesos (P60) a month, and that, if she did not agree to the new
rate of rent, she might vacate the house. Evarista Robles refused to pay such a
new rate of rent and to vacate the house, and Lizarraga Hermanos brought suit
against her for ejectment. Evarista Robles sued Lizarraga Hermanos afterwards to
recover the value of the improvements.
ISSUES:
1.) Whether or not Evarista Robles is the owner of the aforesaid improvements and
has the right to demand payment of their value.

78
2.) Whether or not she has any right to retain the building until the said value is
paid to her.
HELD: 1.) Yes. Robles is the owner of the improvements. The expenditures
incurred in these improvements were not necessary inasmuch as without them the
house would have continued to stand just as before, but were useful, inasmuch as
with them the house better serves the purpose for which it was intended, being
used as a residence, and the improvements consisting of the addition of a dining
room, kitchen, closet, and bathroom in the lower and upper stories of the house,
and a stable, suitable as a coach house and dwelling, it is beyond doubt that such
improvements are useful to the building. Since the improvements are useful and
Robles possession is in good faith, applying Article 453, it is beyond question that
Evarista Robles is the owner of such improvements, and entitled to reimbursement
therefor.
2.) Yes. It is a fact that the value of the improvements in question has not as yet
been paid by Lizarraga Hermanos. Wherefore, if Evarista Robles and her husband
are entitled to retain the building until the value of such improvements is paid
them, Lizarraga Hermanos have not yet any right to oust them from the building,
nor, therefore, to be indemnified for any damages caused by the refusal of the
plaintifs found on their legitimate rights. Hence, due to the non-reimbursement of
the aforesaid useful expenditures, the possessor in good faith has the right of
retention until she has been fully reimbursed with the same.
Metropolitan Waterworks and Sewerage System v. Court of Appeals
G.R. No. L-54526, August 25, 1986, 143 SCRA 623
Martinez, J.
FACTS: Sometime in 1965, petitioner MWSS (then known as NAWASA) leased
around one hundred twenty eight (128) hectares of its land (hereafter, subject
property) to respondent CHGCCI (formerly the International Sports Development
Corporation) for twenty five (25) years and renewable for another fifteen (15) years
or until the year 2005, with the stipulation allowing the latter to exercise a right of
first refusal should the subject property be made open for sale. The terms and
conditions of respondent CHGCCI's purchase thereof shall nonetheless be subject
to presidential approval. Pursuant to Letter of instruction (LOI) No. 440 issued on
July 29,1976 by then President Ferdinand E. Marcos directing petitioner MWSS to
negotiate the cancellation of the MWSS-CHGCCI lease agreement for the
disposition of the subject property, Oscar Ilustre, then General Manager of
petitioner MWSS, sometime in November of 1980 informed respondent CHGCCI,
through its president herein respondent Pablo Roman, Jr., of its preferential right to
buy the subject property which was up for sale. Valuation thereof was to be made
by an appraisal company of petitioner MWSS' choice, the Asian Appraisal Co., Inc.
which, on January 30, 1981, pegged a fair market value of P40.00 per square meter
or a total of P53,800,000.00 for the subject property. Upon being informed that
petitioner MWSS and respondent CHGCCI had already agreed in principle on the
purchase of the subject property, President Marcos expressed his approval of the
sale as shown in his marginal note on the letter sent by respondents Jose Roxas
and Pablo Roman, Jr. dated December 20, 1982.The Board of Trustees of petitioner
MWSS thereafter passed Resolution 36-83, approving the sale of the subject
property in favor of respondent SILHOUETTE, as assignee of respondent CHGCCI.
The MWSS-SILHOUETTE sales agreement eventually pushed through. Per the
Agreement dated May 11, 1983 covering said purchase, the total price for the
subject property is P50,925,200, P25 Million of which was to be paid upon
President Marcos' approval of the contract and the balance to be paid within one
(1) year from the transfer of the title to respondent SILHOUETTE as vendee with

79
interest at 12% per annum. The balance was also secured by an irrevocable letter
of credit. A Supplemental Agreement was forged between petitioner MWSS and
respondent SILHOUETTE on August 11, 1983 to accurately identify the subject
property. Subsequently, respondent SILHOUETTE, under a deed of sale dated July
26, 1984, sold to respondent AYALA about sixty-seven (67) hectares of the subject
property at P110.00 per square meter. Of the total price of around P74 Million, P25
Million was to be paid by respondent AYALA directly to petitioner MWSS for
respondent SILHOUETTE's account and P2 Million directly to respondent
SILHOUETTE. P11,600,000 was to be paid upon the issuance of title in favor of
respondent AYALA, and the remaining balance to be payable within one (1) year
with 12% per annum interest. Respondent AYALA developed the land it purchased
into a prime residential area now known as the Ayala Heights Subdivision. Almost a
decade later, petitioner MWSS on March 26, 1993 filed an action against all herein
named respondents before the Regional Trial Court of Quezon City seeking for the
declaration of nullity of the MWSS-SILHOUETTE sales agreement and all
subsequent conveyances involving the subject property, and for the recovery
thereof with damages.
ISSUE: Whether or not MWSS failed to provide appropriate security measures over
its own records; Circumstances led NBI to believe that the fraudulent encashment
as an inside job.
HELD: Yes. The records likewise show that MWSS failed to provide appropriate
security measures over its own records thereby laying confidential records open to
unauthorized persons. MWSS's own Fact Finding Committee, in its report submitted
to their General Manager underscored this laxity of records control. It observed
that the "office of Mr. Ongtengco (Cashier VI of the Treasury Department at the
NAWASA) is quite open to any person known to him or his staf members and that
the check writer is merely on top of his table. Relying on the foregoing statement
of Mr. Ongtengco, the NBI concluded in its Report dated 2 November 1970 that the
fraudulent encashment of the 23 checks in question was an "inside job". Thus the
NBI believe that the fraudulent act was an inside job or one pulled with inside
connivance at NAWASA. The serial numbers of the checks in question conform with
the numbers in current use of NAWASA, aside from the fact that these fraudulent
checks were found to be of the same kind and design as that of NAWASA's own
checks. While knowledge as to such facts may be obtained through the possession
of a NAWASA check of current issue, an outsider without information from the
inside can not possibly pinpoint which of NAWASA's various accounts has sufficient
balance to cover all these fraudulent checks. None of these checks, it should be
noted, was dishonored for insufficiency of funds.
Shipside incorporated v. Court of Appeals
G.R. No. 143377 February 20, 2001
FACTS: On October 29, 1958, Original Certificate of Title No. 0-381 was issued in
favor of Rafael Galvez, over fourparcels of land - Lot 1 with 6,571 square meters;
Lot 2, with 16,777 square meters; Lot 3 with 1,583 square meters; and Lot 4, with
508 square meters. On April 11, 1960, Lots No. 1 and 4 were conveyed by Rafael
Galvez in favor of Filipina Mamaril, Cleopatra Llana, Regina Bustos, and Erlinda
Balatbat in a deed of sale which was inscribed asEntry No. 9115 OCT No.0-381 on
August 10, 1960. August 16, 1960, Mamaril, et al. sold Lots No. 1 and 4 to
LepantoConsolidated Mining Company.On February 1, 1963, unknown to Lepanto
Consolidated Mining Company, the Court of First Instance of LaUnion, Second
Judicial District, issued an order declaring OCT No. 0-381 of the Registry of Deeds
for the Province of La Union issued in the name of Rafael Galvez, null and void, and
ordered the cancellation thereof.On October 28, 1963, Lepanto Consolidated

80
Mining Company sold to herein petitioner Lots No. 1 and 4. Inthe meantime, Rafael
Galvez filed his motion for reconsideration against the order issued by the trial
courtdeclaring OCT No. 0-381 null and void. The motion was denied. The Court of
Appeals ruled in favor of the Republicof the Philippines.Thereafter, the Court of
Appeals issued an Entry of Judgment, certifying that its decision dated August
14,1973 became final and executory on October 23, 1973. Twenty four long years,
thereafter, on January 14, 1999,the Office of the Solicitor General received a letter
dated January 11, 1999 from Mr. Victor G. Floresca, Vice-President, John Hay Poro
Point Development Corporation, stating that the aforementioned orders and
decision of the trial court in L.R.C. No. N-361 have not been executed by the
Register of Deeds, San Fernando, La Uniondespite receipt of the writ of execution.
On April 21, 1999, the Office of the Solicitor General filed a complaint forrevival of
judgment and cancellation of titles before the Regional Trial Court of the First
Judicial Region (Branch 26,San Fernando, La Union)
ISSUE: whether or not the Republic of the Philippines can maintain the action for
revival of judgment herein
HELD: No. While it is true that prescription does not run against the State, the
same may not be invoked by thegovernment in this case since it is no longer
interested in the subject matter. While Camp Wallace may have belonged to the
government at the time Rafael Galvezs title was ordered cancelled, the same
no longer holds true today.
With the transfer of Camp Wallace to the BCDA, the government no longer has a
right or interest toprotect. Consequently, the Republic is not a real party in interest
and it may not institute the instant action. Normay it raise the defense of
imprescriptibility, the same being applicable only in cases where the government is
a party in interest. Under Section 2 of Rule 3 of the 1997 Rules of Civil Procedure,
every action must be prosecutedor defended in the name of the real party in
interest. To qualify a person to be a real party in interest in whosename an action
must be prosecuted, he must appear to be the present real owner of the right
sought to enforced.A real party in interest is the party who stands to be benefited
or injured by the judgment in the suit, or the partyentitled to the avails of the suit.
And by real interest is meant a present substantial interest, as distinguished froma
mere expectancy, or a future, contingent, subordinate or consequential interest.
Being the owner of the areascovered by Camp Wallace, it is the Bases Conversion
and Development Authority, not the Government, whichstands to be benefited if
the land issued in the name of petitioner is cancelled.
Bachrach v. Seifert and Elianoff
G.R. No. L-2659, October 12, 1950, 87 Phil. 483
Ozaeta, J.
FACTS: The deceased E. M. Bachrach, who left no forced heir except his widow
Mary McDonald Bachrach, in his last will and testament made various legacies in
cash and willed the remainder of his estate. The estate of E. M. Bachrach, as owner
of 108,000 shares of stock of the Atok-Big Wedge Mining Co., Inc., received from
the latter 54,000 shares representing 50 per cent stock dividend on the said
108,000 shares. On June 10, 1948, Mary McDonald Bachrach, as usufructuary or
life tenant of the estate, petitioned the lower court to authorize the Peoples Bank
and Trust Company, as administrator of the estate of E. M. Bachrach, to transfer to
her the said 54,000 shares of stock dividend by indorsing and delivering to her the
corresponding certificate of stock, claiming that said dividend, although paid out in
the form of stock, is fruit or income and therefore belonged to her as usufructuary
or life tenant. Sophie Seifert and Elisa Elianof, legal heirs of the deceased,

81
opposed said petition on the ground that the stock dividend in question was not
income but formed part of the capital and therefore belonged not to the
usufructuary but to the remainderman. While appellants admit that a cash
dividend is an income, they contend that a stock dividend is not, but merely
represents an addition to the invested capital.
ISSUE: Whether or not a dividend is an income and whether it should go to the
usufructuary.
HELD: Yes. The usufructuary shall be entitled to receive all the natural, industrial,
and civil fruits of the property in usufruct. The 108,000 shares of stock are part of
the property in usufruct. The 54,000 shares of stock dividend are civil fruits of the
original investment. They represent profits, and the delivery of the certificate of
stock covering said dividend is equivalent to the payment of said profits. Said
shares may be sold independently of the original shares, just as the ofspring of a
domestic animal may be sold independently of its mother. If the dividend be in fact
a profit, although declared in stock, it should be held to be income. A dividend,
whether in the form of cash or stock, is income and, consequently, should go to the
usufructuary, taking into consideration that a stock dividend as well as a cash
dividend can be declared only out of profits of the corporation, for if it were
declared out of the capital it would be a serious violation of the law.
Under the Massachusetts rule, a stock dividend is considered part of the capital
and belongs to the remainderman; while under the Pennsylvania rule, all earnings
of a corporation, when declared as dividends in whatever form, made during the
lifetime of the usufructuary, belong to the latter. The Pennsylvania rule is more in
accord with our statutory laws than the Massachusetts rule.
Hemedes v. Court of Appeals,
G.R. No. 107132, October 8, 1999, 316 SCRA 347
Gonzaga Reyes, J.
FACTS: Jose Hemedes, father of Maxima Hemedes and Enrique D. Hemedes. Jose
Hemedes executed a document entitled "Donation Inter Vivos with Resolutory
Conditions" whereby he conveyed ownership over the subject land, together with
all its improvements, in favor of his third wife, Justa Kausapin, subject to the
following resolutory conditions that upon her death or marriage, the donee shall
revert the said property to anyone of Jose Hemedes children. On September 27,
1960 a "Deed of Conveyance of Unregistered Real Property by Reversion" was
made conveying to Maxima Hemedes. She had it titled and mortgage it to R & B
Insurance with an annotation of Usufruct in favor of her stepmother, Justa
Kausapin. Unable to pay the mortgage, R & B Insurance extra-judicially foreclosed
the property. However, Justa Kausapin executed another agreement or Kasunduan
on May 27, 1971 to his stepson, Enrique D. Hemedes. He obtained tax
declarations and pay realty taxes from thereon. The Ministry of Agrarian Reform
Office conducted a cadastral survey and indicated Enrique Hemedes as the owner.
Enrique Hemedes sold the property to Dominium Realty Const. Corp. (Dominium),
a sister company of Asia Brewery. Asia Brewery started to introduce some
improvements already when R & B insurance informed them that they are the
owners of the property where these improvements are being built.
ISSUE: Whether or not the kasunduan executed by Justa Kausapin in favor of
Enrique D. Hemedes was valid.
HELD: No. The court dismissed the petition and affirmed the decision of the CA. It
held that Maxima failed to comply with the requirements of Art. 1332 of the civil

82
code and also failed to repudiate Justa Kausapins allegation that she did not
execute such a deed and she never allowed to use the land as security for the
loan. It was found that the deed of conveyance to Maxima was spurious and it
follows that the original title she had for the property was also null and void so as
the mortgage to R & B Insurance. On the other hand, Kausapin executed an
affidavit to affirm the authenticity of the the kasundudan in favor of his stepson,
Enrique Hemedes whom she is dependent from for her financial support.
Fabie v. Gutierrez David
G.R. No. L-123, December 12, 1945, 75 Phil. 536
Ozaeta, J.
FACTS: The petitioner Josefa Fabie is the usufructuary of the income of certain
houses located at 372-376 Santo Cristo, Binondo, and 950-956 Ongpin, Santa Cruz,
Manila, under the ninth clause of the will of the deceased Rosario Fabie y Grey. The
owner of Santo Cristo property abovementioned is the respondent Juan Grey, while
those of the Ongpin property are other person not concern herein. Previous to
September 1944 litigation arose between Josefa Fabie as plaintif and Juan Grey as
defendant and the owner of the Ongpin property as intervenors, involving the
administration of the houses mentioned.
ISSUE: Whether or not the action instituted by the petitioner Josefa Fabie is a
purely possessory action and as such within the jurisdiction of said court, or an
action founded on property right and therefore beyond the jurisdiction of the
municipal court.
HELD: Yes. It is admitted by the parties that the petitioner Josefa Fabie is the
usufructuary of the income of the property in question and that the respondent
Juan Grey is the owner thereof. It is likewise admitted that by virtue of a final
judgment entered in Civil Case No. 1659 of the Court of First Instance of Manila
between the usufructuary and the owner, the former has the right to collect all the
rents of said property for herself with the obligation on her part to pay all the real
estate taxes, special assessments, and insurance premiums, and make all
necessary repairs thereon, and in case default on her part the owner shall have the
right to do all those things, in which event he shall be entitled to collect all
subsequent rents of the property concerned until the amount paid by him and the
expenses of collection are fully satisfied, after which the usufructuary shall again
collect the rents. There is therefore no dispute as to the title to or the respective
interests of the parties in the property in question. The naked title to the property
is to admittedly in the respondent Juan Grey, but the right to all the rents thereof,
with the obligation to pay the taxes and insurance premiums and make the
necessary repairs, is, also admittedly, vested in the usufructuary, the petitioner
Josefa Fabie, during her lifetime.
Construing said judgment in the light of the ninth clause of the will of the deceased
Rosario Fabie y Grey, which was quoted in the decision and by which Josefa Fabie
was made by the usufructuary during her lifetime of the income of the property in
question, we find that the said usufructuary has the right to administer the
property in question. All the acts of administration to collect the rents for
herself, and to conserve the property by making all necessary repairs and paying
all the taxes, special assessments, and insurance premiums thereon were by
said judgment vested in the usufructuary
Felix gochan and sons realty v. Heirs of raymundo baba
GR. No. 138945

83
FACTS: In 1966, Dorotea Inot and 2 of her children, Victoriano Baba and Gregorio
Baba sold a parcel of land to Felix Gochan and Sons Realty Corporation (Gochan
Realty). Consequently, the title over the land was transferred to Gochan Realty.
In 1995, the other 5 children of Inot (Bestra, Maricel, Crecencia, Antonio and
Petronila all surnamed Baba) discovered the sale executed in 1966. They filed a
complaint for quieting of title and reconveyance with damages against Gochan
Realty. They alleged that Gochan Realty and their mother and two siblings
connived in executing the extrajudicial settlement and deed of sale which
fraudulently deprived them of their hereditary share in the said parcel of land. And
that said transactions are void insofar as their respective shares are concerned
because they never consented to the said sale and extrajudicial settlement.
The trial court dismissed the complaint filed by Baba et al as it ruled that their
action has prescribed by reason of prescription and laches. It applied the rule that
the fraudulent conveyance of the property creates an implied trust, an obligation
created by law, which prescribes in ten years from the date of the issuance of the
certificate of title.
The Court of Appeals reversed the decision of the trial court. It found that the heirs
action is a suit to enforce an implied or constructive trust based on fraud, but it
ruled that since the heirs are in possession of the disputed property, their action
cannot be barred by prescription and laches, being in the nature of a suit for
quieting of title.
ISSUE: Whether or not the decision of the Court of Appeals is correct.
HELD: Yes. But the ground relied upon by the CA is erroneous. The contract in
question executed in 1966 is void ab initio by reason of the lack of consent from
the other heirs in executing said contract. Without said consent, there can be no
valid contract of sale between Inot et al and Gochan Realty. The rule is: Actions for
the declaration of inexistence of contracts on the ground of absence of any of the
essential requisites thereof do not prescribe.
The case is however remanded to the trial court to determine whether or not the
heirs are guilty of laches (which is diferent from prescription). Though laches
applies even to imprescriptible actions, its elements must be proved positively.
Laches is evidentiary in nature which could not be established by mere allegations
in the pleadings and cannot be resolved in a motion to dismiss, hence, it must be
settled in the trial court. Gochan Realty is not precluded from presenting evidence
that it is a purchaser in good faith or that the heirs have no personality to sue for
reconveyance or, even assuming that they are lawful heirs of Dorotea Inot and
Raymundo Baba, that they are guilty of laches or are estopped from questioning
the validity of the extrajudicial partition and deed of sale.
Vda. De Aranas v. Aranas
G.R. No. L-56249, May 29, 1987, 150 SCRA 415
Paras, J.
FACTS: Fr. Teodoro Aranas, a priest of the Roman Catholic Church, died on January
19, 1953. He had executed on June 6, 1946 his Last Will and Testament which was
admitted to probate on August 31, 1956. In said Last Will and Testament, Fr.
Teodoro Aranas stipulated the special administration of the remainder of his estate
(after returning to his brothers Aniceto and Carmelo or their heirs all properties
acquired by him including 10 parcels of land inherited by him from his parents) by

84
Vicente Aranas, a faithful and serviceable nephew and designating him also as
recipient of 1/2 of the produce of said properties after deducting the expenses for
the administration and the other 1/2 of the produce to be given to the Catholic
Church for the eternal repose of the testator's soul. Said pertinent provision reads
as follows: It is my will that the lands I had bought from other persons should be
converged and placed under a special administrator. The special administrator of
these lands, for his office, should receive one half of all the produce from which
shall be deducted the expenses for the administration, and the other half of the
produce should be received by the Roman Catholic Church and should be spent for
my soul, Vicente B. Aranas (Tingting), because he is a faithful and serviceable
nephew, should be the first special administrator of said properties, without bond,
until his death or until he should not want to hold the said office anymore. Anyone
of the sons of my brother Carmelo Aranas can hold the said office of special
administrator, and none other than they. Their father, my brother Carmelo Aranas
shall be the one to decide who among them shall hold the said office, but upon the
death of my said brother Carmelo Aranas, his said sons will have power to select
the one among them ourselves. The special administration is perpetual.
ISSUE: Whether or not perpetual inalienability and administration of the estate of
the late Fr. Teodoro Aranas is null and void for being violative of Article 870 of the
NCC.
HELD: No. Vicente Aranas as a usufructuary has the right to enjoy the property of
his uncle with all the benefits which result from the normal enjoyment (or
exploitation) of another's property, with the obligation to return, at the designated
time, either the same thing, or in special cases its equivalent. This right of Vicente
to enjoy the fruits of the properties is temporary and therefore not perpetual as
there is a limitation namely his death or his refusal. Likewise his designation as
administrator of these properties is limited by his refusal and/or death and
therefore it does not run counter to Art. 870 of the Civil Code relied upon by the
petitioners. Be it noted that Vicente Aranas is not prohibited to dispose of the fruits
and other benefits arising from the usufruct. Neither are the naked owners (the
other heirs) of the properties, the usufruct of which has been given to Vicente
Aranas prohibited from disposing of said naked ownership without prejudice of
course to Vicente's continuing usufruct. To void the designation of Vicente Aranas
as usufructuary and/or administrator is to defeat the desire and the dying wish of
the testator to reward him for his faithful and unselfish services rendered during
the time when said testator was seriously ill or bed-ridden.
Locsin v. Valenzuela
G.R. No. L-51333, May 18, 1989, 173 SCRA 454
Feliciano, J.
FACTS: Petitioners were co-owners of a large tract of agricultural land known as
Hacienda Villa Regalado. A portion of this land known as Lot No. 2-C-A-3 was
subject to lifetime usufructuary rights of respondent Helen Schon. The bulk of this
lot was cultivated by the lessees who customarily delivered the rentals to
respondent. In 1972, PD 27 was enacted, decreasing the Emancipation of
Tenants. The tract of land owned in common by the petitioners, including the
portion thereof subject to petitioners usufructuary rights, fell within the scope of
the Operation Land Transfer. Petitioners sought the opinion of the Department of
Agrarian Reform(DAR) as to who should be entitled to receive the rental payments
which continued to be made by the tenants to respondent. The DAR District Officer
rendered the opinion that the rental payments were properly considered as
amortization payments for the land and as such should pertain to the landowners
and not the usufructuary.

85
ISSUE: Whether or not the usufructuary was extinguished by PD 27 and who,
between the naked owner and the usufructuary, should be entitled to the amounts
paid by the tenants beginning October 21, 1972.
HELD: Yes. The usufruct which had therefore existed as a jus in re aliena in favour
of Helen Schon was efectively extinguished by PD 27. To hold, as private
respondent apparently urges would obviously defeat the purpose of the land
reform statute. PD 27 was enacted to emancipate the tenants from bondage of
the soil by giving to the tenant-farmers ownership of the land which they were
cultivating. Ownership over the lands subjected to the Operation Land Transfer
moved from the registered owner to the tenants. The Court holds that Lot No. 2-CA-3 having been declared part of the land reform area and subjected to the
Operation Land Transfer, the payments made on October 21, 1972 by the tenantfarmers constituted amortization payments on the cost of the land that they were
required to pay under PD 27. These payments, therefore, legally pertain to the
petitioners as part of the compensation for the dominion over the land of which
they were deprived of by operation of PD 27.
Valisno v. Adriano
G.R. No. L-37409, May 23, 1988, 161 SCRA 398
Grino Aquino, J.
FACTS: Plaintif appellant Nicolas Valisno alleges that he is the owner of a parcel
of land in Nueva Ecija which he bought from his sister, Honorata Adriano Francisco.
Said land is planted with watermelon, peanuts, corn, tobacco and other vegetables
and adjoins the land of Felipe Adriano, on the bank of the Pampanga River. At the
time of the sale of the land to Valisno, the land was irrigated by water from the
Pampanga River through a canal about 70 meters long, traversing Adrianos land.
Later, Adriano levelled a portion of the irrigation canal so that Valisno was deprived
of the irrigation water and prevented from cultivating his 57 hectare land. Thus,
Valisno filed a complaint for deprivation of waters rights in the Bureau of Public
Works and Communications (Bureau PWC). Bureau PWC ruled in favour of
Valisno. Instead of restoring the irrigation canal, Adriano asked for a reinvestigation
of the case which was granted. In the meantime, Valisno rebuilt the irrigation canal
at his own expense due to his urgent need to irrigate his watermelon fields. Valisno
then filed a complaint for damages. However, the Secretary of Bureau PWC
reversed its decision and dismissed Valisnos complaint. It held that Eladio
Adrianos water rights which had been granted in1923 ceased to be enjoyed by
him in 1936 or 1937, when his irrigation canal collapsed. His non-use of the water
rights since then for a period of more than five years extinguished the grant by
operation of law. Hence, the water rights did not form part of his hereditary estate
which his heirs partitioned among themselves. Likewise, Valisno, as vendee of the
land which Honorata received from her fathers estate did not acquire any water
rights with the land purchased. The trial court held that Valisno had no right to
pass through the defendant's land to draw water from the Pampanga River. It
pointed out that under Section 4 of the Irrigation Law, controversies between
persons claiming a right to water from a stream are within the jurisdiction of the
Secretary of Bureau-PWC and his decision on the matter is final, unless an appeal
is taken to the proper court within thirty days. The court may not pass upon the
validity of the decision of the Public Works Secretary collaterally. Furthermore,
there was nothing in Valisnos evidence to show that the resolution was not valid. It
dismissed the complaint and counterclaim. Valisnos motion for reconsideration
was denied, and he appealed to the Court of the Appeals who certified the case to
the Supreme Court.

86
ISSUE: Whether the provisions of the Irrigation Act (Act No. 2152) or those of the
Civil Code should apply to this case.
HELD: The provisions of the Civil Code shall apply. The existence of the irrigation
canal on Adrianos land for the passage of water from the Pampanga River to
Honorata's land prior to and at the time of the sale of Honorata's land to Valisno
was equivalent to a title for the vendee of the land to continue using it as provided
in Article 624 of the Civil Code: The existence of an apparent sign of easement
between two estates, established or maintained by the owner of both shall be
considered, should either of them be alienated, as a title in order that he
easement may continue actively and passively, unless at the time, theownership of
the two estates is divided, the contrary should be provided in the title of
conveyance of either of them, or the sign aforesaid should be removed before the
execution of the deed. This provision shall also apply in case of the division of a
thing owned in common on by two or more persons (Civil Code).
This provision was lifted from Article 122 of the Spanish Law of Waters which
provided: Whenever a tract of irrigated land which previously received its waters
from a single point is divided through inheritance, sale or by virtue of some other
title, between two or more owners, the owners of the higher estates are under
obligation to give free passage to the water as an easement of conduit for the
irrigation of the lower estates, and without right to any compensation therefore
unless otherwise stipulated in the deed of conveyance.
The deed of sale in favor of Valisno included the "conveyance and transfer of the
water rights and improvements" appurtenant to Honorata Adriano's property. By
the terms of the Deed of Absolute Sale, the vendor Honorata Adriano Francisco
sold, ceded, conveyed and transferred to Dr. Nicolas Valisno all "rights, title,
interest and participations over the parcel of land above- described, together with
one Berkely Model 6 YRF Centrifugal Pump G" suction, 6" discharge 500-1500 GPM,
with Serial No. 5415812 and one (1) set of suction pipe and discharge of pipe with
elbow, nipples, flanges and footvalves," and the water rights and such other
improvements appertaining to the property subject of this sale. According to
Valisno, the water right was the primary consideration for his purchase of
Honorata's property, for without it the property would be unproductive.
Water rights, such as the right to use a drainage ditch for irrigation purposes,
which are appurtenant to a parcel of land, pass with the conveyance of the land,
although not specifically mentioned in the conveyance. The purchaser's easement
of necessity in a water ditch running across the grantor's land cannot be defeated
even if the water is supplied by a third person. The fact that an easement by grant
may also have qualified as an easement of necessity does detract from its
permanency as property right, which survives the determination of the necessity.
As an easement of waters in favor of Valisno has been established, he is entitled to
enjoy it free from obstruction, disturbance or wrongful interference (19 CJ 984),
such as Adrianos act of levelling the irrigation canal to deprive him of the use of
water from the Pampanga River.
CONSTANTE AMOR DE CASTRO and CORAZON AMOR DE CASTRO v. COURT
OF APPEALS andFRANCISCO ARTIGO,
G.R. No. 115838, July 18, 2002
Facts: De castro were co-owners of four (4) lots. In a letter, Artigo was authorized
by appellants to act as real estatebroker in the sale of these properties and five
percent(5%) of which will be given to the agent as commission. It wasappellee who
first found Times Transit Corporation, who bought 2 lots. Artigo felt short of his

87
commission. Hence, he suedbelow to collect the balance. De castros then moved
for the dismissal for failure to implead other co-owners asindispensable parties.
The De Castros claim that Artigo always knew that the two lots were co-owned with
their othersiblings and failure to implead such indispensable parties is fatal to the
complaint since Artigo, as agent of all the four co-owners, would be paid with funds
co-owned by the four co-owners.
ISSUE: Whether the complaint merits dismissal for failure to implead other coowners as indispensable parties.
HELD: Devoid of merit. Art. 1915. If two or more persons have appointed an agent
for a common transaction or undertaking, they shallbe solidarily liable to the agent
for all the consequences of the agency.The rule in this article applies even when
the appointments were made by the principals in separate acts, provided thatthey
are for the same transaction.
The solidarity arises from the common interest of the principals, and not fromthe
act of constituting the agency. By virtue of this solidarity, the agent can recover
from any principal the wholecompensation and indemnity owing to him by the
others.
The parties, however, may, by express agreement, negatethis solidary
responsibility. The solidarity does not disappear by the mere partition efected
by the principals after theaccomplishment of the agency.When the law expressly
provides for solidarity of the obligation, as in the liability of co-principals in a
contract ofagency, each obligor may be compelled to pay the entire obligation.The
agent may recover the whole compensation fromany one of the co-principals, as in
this case.
Ronquillo, et. al. v. Roco, et. al.
G.R. No. L-10619, February 28, 1958, 103 Phil. 84
Montemayor, J.
FACTS: Plaintif Leogario Ronquillo have been in the continuous and uninterrupted
use of a road which traversed the land of the defendants, Rocos, in going to
Igualdad Street and the market place of Naga City for more than 20 years and that
the Rocos have long recognized and respected the private legal easement of a
right of way of said plaintifs.
On May 12, 1953, the defendants along with a number of men maliciously
obstructed plaintifs right of way by constructing a chapel in the middle of the said
road and then later, by means of force, intimidation, and threats, illegally and
violently planted wooden posts, fenced with barbed wire and closed hermitically
the road passage way thereby preventing the plaintif from using it.
The plaintif claims that he has already acquired the easement of right of way over
the land thru prescription by his continuous and uninterrupted use of the narrow
strip of land as passage way. However, plaintifs complaint was dismissed by the
CFI.
ISSUE: Whether or not an easement of right of way can be acquired by
prescription.
HELD: No. The Court held than an easement of right of way may not be acquired
thru prescription because though it may be apparent, it is nevertheless
discontinuous or intermittent, and therefore, under Article 622 of the New Civil

88
Code, can be acquired only by a virtue of a title. Furthermore, a right of way cannot
be acquired by prescription because prescription requires that the possession be
continuous and uninterrupted.
Taedo v. Bernad
G.R. No. L-66520 August 30, 1988, 165 SCRA 86
Padilla, J.
FACTS: Private respondent Antonio Cardenas owned Lot 7501-A and Lot 7501-B.
On the said two lots, a septic tank was constructed for the common use of the
occupants of both lots. Cardenas sold Lot 7501-A to herein petitioner Taedo and
the other Lot 7501-B was also mortgaged to Taedo as a security for the payment
of loan with an agreement that Cardenas would only sell Lot 7501-B to him.
However, said Lot 7501-B was sold to herein respondent Spouses Romeo and
Pacita Sim. Upon learning of the said sale, Taedo ofered to redeem the property
from Sim but the latter refused. Instead, Sim blocked the sewage pipe connecting
the building of Eduardo Taedo built on Lot 7501-A, to the septic tank in Lot 7501B. He also asked Taedo to remove that portion of his building enroaching on Lot
7501-B. Taedo was then constrained to file an action for legal redemption and
damages invoking Article 1622 of the Civil Code. On the other hand, respondent
Spouses claimed they are the absolute owners of Lot 7501-B and that Eduardo
Taedo has no right to redeem the land under Art. 1622 of the Civil Code as the
land sought to be redeemed is much bigger than the land owned by Taedo.
ISSUE: Whether or not the petitioners right to continue to use the septic tank,
erected on Lot 7501-B, ceased upon the subdivision of the land and its subsequent
sale to diferent owners who do not have the same interest.
HELD: No. Applying Article 631 and 624 of the Civil Code, no statement abolishing
or extinguishing the easement of drainage was mentioned in the deed of sale of
Lot 7501-A to Eduardo Taedo. Nor did Antonio Cardenas stop the use of the drain
pipe and septic tank by the occupants of Lot 7501-A before he sold said lot to
Eduardo Tafiedo. Hence, the use of the septic tank is continued by operation of law.
Accordingly, the spouses Romeo and Pacita Sim the new owners of the servient
estate (Lot 7501- B), cannot impair, in any manner whatsoever, the use of the
servitude.
Costabella Corporation v. Court of Appeals
G.R. No. 80511 January 25, 1991, 193 SCRA 333
Sarmiento, J.
FACTS: Petitioner owns the real estate properties situated at Sitio Buyong,
Maribago, Lapu-Lapu City, on which it had constructed a resort and hotel. The
private respondents, on the other hand, are the owners of adjoining properties.
Before the petitioner began the construction of its beach hotel, the private
respondents, in going to and from their respective properties and the provincial
road, passed through a passageway which traversed the petitioner's property. In
1981, the petitioner closed the aforementioned passageway when it began the
construction of its hotel, but nonetheless opened another route across its property
through which the private respondents, as in the past, were allowed to pass. Later,
or sometime in August, 1982, when it undertook the construction of the second
phase of its beach hotel, the petitioner fenced its property thus closing even the
alternative passageway and preventing the private respondents from traversing
any part of it. Therefore, an action for injunction with damages was filed against

89
the petitioner by the private respondents before the then Court of First Instance of
Cebu.
The CFI rendered a decision on March 15, 1984 finding that the private
respondents had acquired a vested right over the passageway in controversy
based on its long existence and its continued use and enjoyment by the private
respondents and also by the community at large. On appeal, Appellate Court held
as without basis the trial court's finding that the private respondents had acquired
a vested right over the passageway in question by virtue of prescription. The
appellate court pointed out that an easement of right of way is a discontinuous one
which, under Article 622 of the New Civil Code, may only be acquired by virtue of a
title and not by prescription. That notwithstanding, the appellate court went on to
rule that ". . . in the interest of justice and in the exercise by this Court of its equity
jurisdiction, there is no reason for Us in not treating the easement here sought by
appellees Katipunan Lumber Co., Inc. and Perfecta Guangco as one that is not
dependent upon the claims of the parties but a compulsory one that is legally
demandable by the owner of the dominant estate from the owner of the servient
estate."
ISSUE: Whether or not the easement may be granted to private respondent over
the land of Costabella.
HELD: No. It is already well-established that an easement of right of way, as is
involved here, is discontinuous and as such can not be acquired by prescription.
Insofar therefore as the appellate court adhered to the foregoing precepts, it stood
correct. Unfortunately, after making the correct pronouncement, the respondent
Appellate Court did not order the reversal of the trial court's decision and the
dismissal of the complaint after holding that no easement had been validly
constituted over the petitioner's property. Instead, the Appellate Court went on to
commit a reversible error by considering the passageway in issue as a compulsory
easement which the private respondents, as owners of the "dominant" estate, may
demand from the petitioner the latter being the owner of the "servient" estate.
Based on Articles 649 and 650 of the Civil Code, the owner of the dominant estate
may validly claim a compulsory right of way only after he has established the
existence of four requisites, to wit: (1) the (dominant) estate is surrounded by
other immovables and is without adequate outlet to a public highway; (2) after
payment of the proper indemnity; (3) the isolation was not due to the proprietor's
own acts; and (4) the right of way claimed is at a point least prejudicial to the
servient estate. In the case at bar, there is absent any showing that the private
respondents had established the existence of the four requisites mandated by law.
Encarnacion v. Court of Appeals
G.R. No. 77628, March 11, 1991, 195 SCRA 74
Fernan, C.J.
FACTS: Petitioner owns the dominant estate bounded on north by the servient
estate owned by respondents and an estate owned by Magsino, all of which are
located in Talisay, Batangas. The servient estate is bound on the north by the
national highway. To provide access to the highway, a one meter road path was
paved through in which half of its width was taken from the estate of Magsino and
the other half from the estate of the respondent. Petitioner started a nursery plant
type of business in which pushcarts were used to haul the plants from his estate to
and from his nursery and the highway, using the one meter road path. As his
business grew, he bought a jeepney to enable him to transport more plants and
soil catering to the now bigger demand. The problem however was that the
jeepney cannot pass through the road path since its width would not be

90
accommodated by a one meter width. Petitioner made a request upon the
respondent to sell to him 1 meters of their property so that the pathway may be
widened to enable his jeepney to pass through. The respondents refused.
Petitioner went to court praying that he would be granted the additional land to the
right of way already constituted but the trial court rendered a decision adverse to
the petitioner because there was no such necessity as it was shown that there was
the presence of dried river bed only 80 meters away from the property of the
petitioner which he may use as an alternative route. The CA affirmed said decision
of the trial court.
ISSUE: Whether or not petitioner is entitled to be granted his prayer to buy the
additional land to increase the existing one meter road path.
HELD: Yes. Even with the presence of the dried river bed, upon thorough
investigation, it was found to be an inadequate right of way because a concrete
bridge traverses it thereby the jeep would have to jump over said bridge which has
a height of 5 meters in order to reach the highway. It was also found that during
the rainy season, the same was impassable as it became flooded. This right of way
could not provide adequate access to the highway thereby when an estate has no
access to a public road, it may demand for a right of way. Furthermore, under
Article 651 of the Civil Code, it is the needs of the dominant property which
ultimately determine the width of the right of way. In this case, since the business
of the petitioner grew larger and pushcarts became tedious to transport his
nursery plants, it became necessary for him to do so with a jeepney. And in order
to efficiently make such transportation of his plants, the right of way had to be
widened to accommodate the width of the jeepney of the petitioner. The petitioner
thus shall be granted the additional land to the existing right of way.
Case v. Heirs of Tuason
G.R. No. L-5044, December 1, 1909, 14 Phil. 521
Torres, J.
FACTS: The counsel for the heirs of Pablo Tuason and Leocadia Santibaez alleged
that the parties whom he represents are owners in common of the property
adjoining that of the petitioner Edwin Case on the southwest. The latter, extended
his southwest boundary line to a portion of the lot of the said heirs of Tuason and
Santibaez. They alleged that the true dividing line between the property of the
petitioner and that of the said heirs is a belonging to the respondents, and that
about two years ago, when Case made alterations in the buildings erected on his
land, he improperly caused a portion of them to rest on the wall owned by the
respondents.
ISSUE: Whether or not the wall is the property of the heirs of the late Tuason and
Santibaez.
HELD: The wall in controversy belongs to the heirs of the late Tuason and
Santibaez for the reason, among others, that in the public document by which
one of their original ancestors acquired on the 19th of April, 1796, the property
now possessed by them, it appears that property was then already inclosed by a
stone wall.
The wall supports only the property of the respondents and not that of the
petitioner, can not be a party wall, one-half of which along its entire length would
belong to the adjoining building owned by Mr. Case. There is not sufficient proof to
sustain such claim, and besides, the building erected thereon disproves the
pretension of the petitioner.

91
Under article 572 of the Civil Code the easement of party walls is presumed, unless
there is a title or exterior sign, or proof to the contrary, among others, in dividing
walls adjoining buildings up to the common point of elevation.
The legal presumption as to party walls is limited to the three cases dealt with in
the said article of the code, and is that of juris tantum unless the contrary appear
from the title of ownership of the adjoining properties, that is to say, that the entire
wall in controversy belongs to one of the property owners, or where there is no
exterior sign to destroy such presumption and support a presumption against the
party wall.
It can not be presumed that the aforesaid portion was a party wall, and that it was
not exclusively owned by the respondents, inasmuch as the latter have proven by
means of a good title that has not been impugned by the petitioner, that when one
of their ancestors and principals acquired the property the lot was already inclosed
by the wall on which the building was erected; it must therefore be understood
that in the purchase of the property the wall by which the land was inclosed was
necessarily included.
Choco v. Santamaria
G.R. No. 6076, December 29, 1911, 21 Phil. 132
Mapa, J.
FACTS: The defendant in the building of his house, has made several openings
and windows in the walls of the house on both sides overlooking then property of
the plaintif; that at the time the defendant was building his house, and the
windows and the openings were being made, the plaintifs protested, and later on
and in the year 1905 made written protest and demand on the defendant, and the
defendant received the written protest and referred it to his counsel, who, from the
evidence, appears to have suggested an amicable and adjustment of the matter,
but the adjustment was not made, and this action was brought. The Trial Court
rendered judgment in favor of the plaintifs, Severina and Flora Choco, and against
the defendant, Isidro Santamaria, forever prohibiting the opening of the window
stated, which must be closed, and forever prohibiting the opening of the windows
and openings marked, which must be closed or made to conform to the
requirements of law with regard to dimensions and an iron grate embedded in the
wall, with the costs of the action.
ISSUE: Whether or not the lower court erred by not ordering in his judgment the
final and perpetual closing of the large window opened in the balcony of the back
part of the appellee's house and that, though the appellant's lot can be seen
through the window, it is not contiguous to the latter's property.
HELD: To judge from the photographic views, it opens on the boundary line
between the said lot and that the appellee and is situated perpendicularly above a
part of the wall that belongs to the appellants. This opinion is corroborated by the
testimony of the defendant's witness who took the said photographs, in so far as
he said that "a part of the window in question is in front of the plaintifs' property,
since between it and the plaintifs' property there does not intervene the distance
required by law that of two meters in the first case, and 60 centimeters in the
second, therefore, its opening is a manifest violation of the provisions of article 582
of the Civil Code which reads as follows: Windows with direct views, or balconies
or any similar openings projecting over the estate of the neighbor, cannot be made
if there is not a distance of, at least, 2 meters between the wall in which they are
built and said estate. Neither can side nor oblique views be opened over said

92
property, unless there is a distance of 60 centimeters. Because of the lack of the
distance required by law, the window in question must be closed, and
consequently the judgment appealed from should be modified in this sense, as
regards this window.
Secretary of Education vs. Heirs of Rufino Dulay
G.R. No. 164748 January 27, 2006
FACTS: On August 3, 1981, the spouses Rufino Dulay, Sr. and Ignacia Vicente
Dulay executed a deed of donation over a 10,000-square-meter portion of their
property in favor of the Ministry of Education and Culture.
The property was subdivided. On April 13, 1983, a Transfer Certificate of Title was
issued in the name of the Ministry of Education and Culture, represented by
Laurencio C. Ramel, the Superintendent of Schools of Isabela. However, the
property was not used for school purposes and remained idle.
Sometime in 1988, the DECS, through its Secretary, started construction of the
Rizal National High School building on a parcel of land it acquired from Alejandro
Feliciano. The school site was about 2 kilometers away from the land donated by
the spouses Dulay.
In a letter to the DECS Secretary dated August 19, 1994, the spouses Dulay
requested that the property be returned to them considering that the land was
never used since 1981, or a period of more than 13 years. On August 28, 1994, the
Barangay Council of Rizal, Santiago City issued Resolution No. 397 recognizing the
right of the donors to redeem the subject parcel of land because of the DECS
failure to utilize it for the intended purpose. It further resolved that the Rizal
National High School no longer needed the donated land "considering its distance
from the main campus and [the] failure to utilize the property for a long period of
time."
On August 31, 1997, the heirs of Dulay, Sr., herein respondents, filed a complaint
for the revocation of the deed of donation and cancellation of the title, alleging
that (1) there was a condition in the deed of donation: that the DECS, as donee,
utilize the subject property for school purposes, that is, the construction of a
building to house the Rizal National High School, (2) the DECS did not fulfill the
condition and that the land remained idle up to the present, and (3) the donation
inter vivos was inofficious, since the late Rufino Dulay, Sr. donated more than what
he could give by will.
Petitioners, through the Office of the Solicitor General (OSG), interposed the
following defenses: (a) the DECS complied with said condition because the land
was being used by the school as its technology and home economics laboratory;
(b) the donation was not inofficious for the donors were the owners of five other
parcels of land, all located at Rizal, Santiago City; (c) the DECS acquired the
disputed property by virtue of purchase made on December 8, 1997 by the
barangay of Rizal, Santiago City in the amount of P18,000.00 as certified by its
former Barangay Captain, Jesus San Juan;11 and (d) the action of the respondents
had prescribed. The OSG also claimed that students planted a portion of the land
with rice, mahogany seedlings, and fruit-bearing trees; the produce would then be
sold and the proceeds used for the construction of a school building on the subject
property.
ISSUE: Whether or nor the DECS had complied with the condition imposed on the
the deed of donation.

93
HELD: The contention of petitioners has no merit.
As gleaned from the CA decision, petitioners failed to prove that the donated
property was used for school purposes as indicated in the deed of donation.
We find it difficult to sustain that the defendant-appellants have complied with the
condition of donation. It is not amiss to state that other than the bare allegation of
the defendant-appellants, there is nothing in the records that could concretely
prove that the condition of donation has been complied with by the defendantappellants. In the same breadth, the planting of palay on the land donated can
hardly be considered and could not have been the "school purposes" referred to
and intended by the donors when they had donated the land in question. Also, the
posture of the defendant-appellants that the land donated is being used as
technology and home economics laboratory of the Rizal National High School is far
from being the truth considering that not only is the said school located two
kilometers away from the land donated but also there was not even a single
classroom built on the land donated that would reasonably indicate that, indeed,
classes have been conducted therein. These observations, together with the
unrebutted ocular inspection report made by the trial court which revealed that the
land donated remains idle and without any improvement thereon for more than a
decade since the time of the donation, give Us no other alternative but to conclude
that the defendant-appellants have, indeed, failed to comply with what is
incumbent upon them in the deed of donation.
Solid Manila Corporation v. Bio Hong Trading Co., Inc.
G.R. No. 90596, April 8, 1991, 195 SCRA 748
Sarmiento, J.
FACTS: Petitioner Solid Manila Corporation is the owner of the land in Ermita,
Manila. The same lies in the vicinity of another parcel, registered in the name of
the private respondent Bio Hong Trading Co., Inc. The private respondents title
came from a prior owner, and in their deed of sale, the parties thereto reserved as
easement of way. As a consequence, there is an annotation which was entered
wherein a construction of private alley has been undertaken. However, the
petitioner averred that they and their neighbors have been using the private alley
and maintained and contributed to its upkeep until sometime in 1983. Due to this,
the private respondent constructed steel gates that precluded unhampered used.
The petitioner commenced suit for injunction against the private respondent to
have the gates removed and to allow full access to the easement. The court a quo
issued ex parte an order directing the private respondent to open the gates.
However, the Court of Appeals ordered the restoration of the annotation. They
ruled that an easement is a mere limitation on ownership and that it does not
impair the private respondents title, and that since the private respondent had
acquired title to the property, merger brought about an extinguishment of the
easement. The petitioner then averred that the very deed of sale executed
between the private respondent and the previous owner of the property
excluded the alley in question, and that in any event, the intent of the parties
was to retain the alley as an easement, notwithstanding the sale.
ISSUE: Whether or not an easement had been extinguished by merger.
HELD: No. The Court held that no genuine merger took place as a consequence of
the sale in favor of the private respondent corporation. According to the Civil Code,
a merger exists when ownership of the dominant and servient estates is
consolidated in the same person. Merger then, as can be seen, requires full
ownership of both estates. One thing ought to be noted here, however. The

94
servitude in question is a personal servitude, that is to say, one constituted not in
favor of a particular tenement but rather, for the benefit of the general public as
stated in Article 614 of the Civil Code. In personal servitude, there is therefore no
owner of a dominant tenement to speak of, and the easement pertains to
persons without a dominant estate, in this case, the public at large. Merger, as we
said, presupposes the existence of a prior servient-dominant owner relationship,
and the termination of that relation leaves the easement of no use. Unless the
owner conveys the property in favor of the public, if that is possible, no genuine
merger can take place that would terminate a personal easement.
Floro v. Llenado
G.R. No. 75723, June 2, 1995, 244 SCRA 713
Romeo, J.
FACTS: Petitioner Simeon Floro is the owner of Floro Park Subdivision who has its
own egress and ingress to and from the Mac Arthur Highway by means of its Road
Lot 4 and the PNR level crossing. On the other hand, Respondent Orlando Llenado,
is the registered owner of Llenado Homes Subdivision, adjacent to Floro Park
Subdivision. Prior to its purchase by Llenado, the land was known as the Emmanuel
Homes Subdivision, a duly licensed and registered housing subdivision in the name
of Soledad Ortega. Bounded on the South by the 5 to 6 meter-wide Palanas Creek,
which separates it from the Floro Park Subdivision, and on the west by ricelands
belonging to Marcial Ipapo. the Llenado Homes does not have any existing road or
passage to the Mac Arthur Highway. However, a proposed access road traversing
the idle riceland of Marcial Ipapo has been specifically provided in the subdivision
plan of the Emmanuel Homes Subdivision which was duly approved by the defunct
Human Settlement Regulatory Commission. Meanwhile, the Llenados sought, and
were granted permission by the Floros to use Road Lots 4 and 5 of the Floro Park
Subdivision as passageway to and from MacArthur Highway. However no contract
of easement of right of way was ever perfected by both parties. Later, Floro
barricaded Road Lot 5 with a pile of rocks, wooden posts and adobe stones,
thereby preventing its use by the Llenados. Llenado instituted a complaint before
the RTC of Malolos, Bulacan against Floro for easement of right of way. The RTC
granted the prayer for the issuance of a writ of preliminary mandatory injunction
and ordered Floro to open the road and pay damages. Thereafter, the trial court
rendered another judgment dismissing the case and lifting the writ of preliminary
mandatory injunction previously issued and ordered the plaintif to pay defendant
damages and costs. On appeal by Llenado on the CA, the judgment of the RTC was
reversed ordering Floro to open roads 4 and 5 and remove all the objects that
prevent passage on road 5 and to pay the plaintif damages with costs and
payment of indemnity for the easement of right of way.
ISSUE: Whether or not Llenado is entitled to a compulsory easement of right of
way.
HELD: No. For the Llenados to be entitled to a compulsory servitude of right of
way under the Civil Code, the preconditions provided under Articles 649 and 650
thereof must be established. These preconditions are: (1) that the dominant estate
is surrounded by other immovables and has no adequate outlet to a public
highway (Art. 649, par. 1); (2) after payment of proper indemnity (Art. 649, par. 1);
(3) that the isolation was not due to acts of the proprietor of the dominant estate
(Art. 649, last par.); and, (4) that the right of way claimed is at the point least
prejudicial to the servient estate; and insofar as consistent with this rule, where
the distance from the dominant estate to a public highway may be the shortest
(Art. 650).

95
The burden of proving the existence of the prerequisites to validly claim a
compulsory right of way lies on the owner of the dominant estate. On the past
subdivision plans by Emmanuel Homes which is bought by Llenado, there is an
indication of an access road through IPAPOs property although it was not properly
paved, a dirt road will suffice. Seeing this, Llenado has failed to comply with the
first requirement. If the servitude requested by Llenado is allowed, other
subdivision developers/owners would be encouraged to hastily prepare a
subdivision plan with fictitious provisions for access roads merely for registration
purposes. Furthermore, if such practice were tolerated, the very purpose for which
Presidential Decree No. 957 was enacted, that is, to protect subdivision buyers
from unscrupulous subdivision owners/developers who renege on their duties to
develop their subdivisions in accordance with the duly approved subdivision plans,
would be defeated.
In order to justify the imposition of the servitude of right of way, there must be a
real, not a fictitious or artificial necessity for it. Mere convenience for the dominant
estate is not what is required by law as the basis for setting up a compulsory
easement. Even in the face of a necessity, if it can be satisfied without imposing
the servitude, the same should not be imposed.
The complaint for easement of right of way filed by Llenado in the lower court did
not contain a prayer for the fixing of the amount that he must pay Floro in the
event that the easement of right of way is constituted. Thus, the existence of the
second requisite has likewise not been established. Private respondent Llenado
admitted that the Ipapo riceland was no longer being cultivated. Indications are
that it has already been abandoned as a ricefield. There was no reason for private
respondent's failure to develop the right of way except the inconvenience and
expenses it would cost him. Hence, the third requisite has not been met. Failing to
establish the existence of the prerequisites under Articles 649 and 650 of the Civil
Code, private respondent Llenado's bid for a compulsory easement of right of way
over Road Lots 4 and 5 of the Floro Park Subdivision must fail.

Quimen v. Court of Appeals


G.R. No. 112331 May 29, 1996, 257 SCRA 163
Bellosillo, J.
FACTS: Petitioner Anastacia Quimen together with her brothers Sotero, Sulpicio,
Antonio and sister Rufina inherited a piece of property situated in Pandi, Bulacan.
They agreed to subdivide the property equally among themselves, as they did,
with the shares of Anastacia, Sotero, Sulpicio and Rufina abutting the municipal
road. Located directly behind the lots of Anastacia and Sotero is the share of their
brother Antonio designated as Lot No. 1448-B-C which the latter divided into two
(2) equal parts, now Lots Nos. 1448-B-6-A and 1448-B-6-B. The latter Lot is behind
the property of Sotero, father of private respondent Yolanda Oliveros. Yolanda
purchased Lot No. 1448-B-6-A from her uncle Antonio through her aunt Anastacia
who was then acting as his administratrix. According to Yolanda, when petitioner
ofered her the property for sale she was hesitant to buy as it had no access to a
public road. But Anastacia prevailed upon her to buy the lot with the assurance
that she would give her a right of way on her adjoining property. Thereafter,
Yolanda constructed a house on the lot she bought using as her passageway to the
public highway a portion of Anastacia's property. But when Yolanda finally ofered
to pay for the use of the pathway Anastacia refused to accept the payment. In fact
she was thereafter barred by Anastacia from passing through her property. Later,
Yolanda purchased the other lot of Antonio Quimen, Lot No. 1448-B-6-B, located
directly behind the property of her parents who provided her a pathway between

96
their house from the lot of Yolanda behind the sari sari store of Sotero, and
Anastacia's perimeter fence. The store is made of strong materials and occupies
the entire frontage of the lot measuring four (4) meters wide and nine meters (9)
long. Although the pathway leads to the municipal road it is not adequate for
ingress and egress. The municipal road cannot be reached with facility because the
store itself obstructs the path so that one has to pass through the back entrance
and the facade of the store to reach the road. Finally, Yolanda filed an action with
the proper court praying for a right of way through Anastacia's property. The report
was that the proposed right of way was at the extreme right of Anastacia's
property facing the public highway, starting from the back of Sotero's sari-sari
store and extending inward by one (1) meter to her property and turning left for
about five (5) meters to avoid the store. However, the trial court dismissed her
complaint. The Court of Appeals reversed the decision declaring that she was
entitled to a right of way on petitioners property and that the way proposed by
Yoland would cause the least damage and detriment to the servient estate.
ISSUE: Whether or not passing through the property of Yolanda's parents is more
accessible to the public road than to make a detour to her property and cut down
the avocado tree standing thereon.
HELD: Yes. The conditions sine quo non for a valid grant of an easement of right of
way are: (a) the dominant estate is surrounded by other immovables without an
adequate outlet to a public highway; (b) the dominant estate is willing to pay the
proper indemnity; (c) the isolation was not due to the acts of the dominant estate;
and, (d) the right of way being claimed is at a point least prejudicial to the servient
estate.
The criterion of least prejudice to the servient estate must prevail over the
criterion of shortest distance although this is a matter of judicial appreciation.
While shortest distance may ordinarily imply least prejudice, it is not always so as
when there are permanent structures obstructing the shortest distance; while on
the other hand, the longest distance may be free of obstructions and the easiest or
most convenient to pass through. In other words, where the easement may be
established on any of several tenements surrounding the dominant estate, the one
where the way is shortest and will cause the least damage should be chosen.
However, as elsewhere stated, if these two (2) circumstances do not concur in a
single tenement, the way which will cause the least damage should be used, even
if it will not be the shortest.
As between a right of way that would demolish a store of strong materials to
provide egress to a public highway, and another right of way which although longer
will only require an avocado tree to be cut down, the second alternative should be
preferred.
Gestopa v. Court of Appeals
G.R. No. 111904, October 5, 2000, 342 SCRA 105
Quisumbing, J.
FACTS: Spouses Danlag were the owners of six parcels of unregistered lands. They
executed three deeds of donation mortis causa, two of which were in favor of
Mercedes Danlag-Pilapil. All deeds contained the reservation of the rights of the
donors to amend, cancel or revoke the donation during their lifetime, and to sell,
mortgage, or encumber the properties donated during the donors' lifetime, if
deemed necessary. The spouses then executed another deed of donation inter
vivos in favor of Mercedes which contained the condition that the donors, Danlag
spouses, shall continue to enjoy the fruits of the land during their lifetime and that

97
the donee enjoy the fruits of the land during their lifetime and that the donee
cannot sell or dispose of the land during the lifetime of the donors without their
prior consent and approval. Consequently, Mercedes caused the transfer of the
parcels of land's tax declaration to her name and paid the taxes on them.
Spouses Danlag sold two parcels of lots to Spouses Agripino and Isabel Gestopa
and executed a deed of revocation recovering the six parcels of land subject to the
deed of donation inter vivos. Mercedes Pilapil filed with the Regional Trial Court
against the Spouses Danlag and Gestopa, for quieting of title over the parcels of
land and alleged that the land was donated to her by Diego Danlag and that she
accepted the donation openly and publicly exercised rights of ownership over the
donated properties, and transferred the tax declarations to her name. She also
alleged that the donation inter vivos was coupled with conditions and, according to
Mercedes, since its perfection, she had complied with all of them; that she had not
been guilty of any act of ingratitude; and that Diego Danlag had no legal basis to
revoke the donation and then in selling the two parcels of land to the Gestopa
spouses.
In their opposition, the spouses Gestopa and the Danlag averred that the deed of
donation was null and void because it was obtained by Mercedes through
machinations and undue influence. Even assuming it was validly executed, the
intention was for the donation to take efect upon the death of the donor and that
the donation was void for it left the donor, Diego Danlag, without any property at
all.
ISSUE: Whether the donation is a donation inter vivos or a donation mortis causa.
HELD: The Court Rules that it was a donation inter vivos. The Court affirmed the
Court of Appeals' decision that the reservation by the donor of lifetime usufruct
indicated that he transferred to Mercedes the ownership over the donated
properties; that the right to sell belonged to the donee, and the donor's right
referred to that of merely giving consent; that the donor changed his intention by
donating inter vivos properties already donated mortis causa; that the transfer to
Mercedes' name of the tax declarations pertaining to the donated properties
implied that the donation was inter vivos; and that Mercedes did not purchase two
of the six parcels of land donated to her.
In ascertaining the intention of the donor, all of the deed's provisions must be read
together. The granting clause shows that Diego donated the properties out of love
and afection for the donee. This is a mark of a donation inter vivos. Second, the
reservation of lifetime usufruct indicates that the donor intended to transfer the
naked ownership over the properties. Third, the donor reserved sufficient
properties for his maintenance in accordance with his standing in society,
indicating that the donor intended to part with the six parcels of land. Lastly, the
donee accepted the donation.
An acceptance clause is a mark that the donation is inter vivos. Acceptance is a
requirement for donations inter vivos. Donations mortis causa, being in the form of
a will, are not required to be accepted by the donees during the donors' lifetime.
The right to dispose of the properties belonged to the donee. The donor's right to
give consent was merely intended to protect his usufructuary interests. The
limitation on the right to sell during the donors' lifetime implied that ownership had
passed to the donees and donation was already efective during the donors'
lifetime. Hence, the moment that it was accepted by Mercedes Danlag-Pilapil,
ownership of the properties was transferred.

98
De Jesus, et. al. v. Homart Corporation, et. al.
G.R. No. 44191 R, August 28, 1974, 19 CA Rep. 831
FACTS: Jesus and Luz Miranda de Jesus are owners of the building located in
Tondo, Manila. They brought an action for damages against Homart Corporation
and Howmill Manufacturing Corporation, owners of the land adjoining the plaintif
on the same street where a sixty storey concrete building was constructed.
Plaintifs allege that the defendants failed to observe the necessary care and
precautions to protect the construction of the plaintifs by depriving it of sufficient
lateral or subjacent support, thereby causing it to sink in some parts; its walls,
ceilings, and floorings to crack in some places; and by the careless manner of
handling the cement used the roofings of the building of the plaintif were
damaged with the accumulated debris piled thereon.
ISSUE: Whether or not proper precautions had been taken by the defendants in
constructing the building in question so as to prevent causing damage to the
building of the plaintif.
HELD: No. Article 684 of the New Civil Code provides No property shall make such
excavations upon his land as to deprive any adjacent land or building sufficient
lateral or subjacent support. A reading of Article 684 shows that the duty of an
adjacent owner not to deprive any adjacent land or building of sufficient lateral or
subjacent support is an absolute one. It does not depend on the degree of care and
precaution made by the proprietor in making the excavation or building on his
land. Plaintifs house which adjoins the seven storey concrete building constructed
by the defendants had sunk by about eight inches. The sinking of the left side of
the house of the plaintifs was due to the weakening of subjacent support and to
the weight of the seven storey concrete building constructed by the defendant, as
the excavation made necessarily disturbed the subjacent soil of the plaintifs land.
Defendants having failed to provide the plaintifs land and house with sufficient
lateral and subjacent support are liable for damages.
La Vista Association, Inc. v. Court of Appeals
G.R. No. 95252, September 5, 1997, 278 SCRA 498
Bellosillo, J.
FACTS: The Tuasons owned a vast tract of land in Quezon City and Marikina, and
when they sold to Philippine Building Corporation a portion of their landholdings, it
was expressly provided in the Deed of Sale with Mortgage that the boundary line
between the property sold and the adjoining property of the Tuasons shall be a
road fifteen (15) meters wide, one-half of which shall be taken from the property
sold to the Philippine Building Corporation and the other half from the portion
adjoining belonging to the Tuasons. Philippine Building Corporation then sold and
assigned with the consent of the Tuasons, the subject parcel of land to ATENEO
which assumed the mortgage and the obligation in the seven and one-half
roadway.
On their part, the Tuasons developed a part of the estate adjoining the portion sold
to Philippine Building Corporation into a residential village known as LA VISTA
Subdivision. Thus the boundary between LA VISTA and the portion sold to ATENEO
was the 15-meter wide roadway known as the Mangyan Road. The Tuasons
developed its 7.5-meter share of the 15-meter wide boundary, while ATENEO
deferred improvement on its share and erected instead an adobe wall on the entire
length of the boundary.

99
ATENEO subsequently sold to Solid Homes Inc. the land which the latter developed
into a subdivision now known as LOYOLA Grand Villas. Solid Homes Inc. now claims
to have an easement of right-of-way along Mangyan Road through which they
could have access to Katipunan Avenue.
LA VISTA however instructed its security guards to prohibit agents and assignees of
Solid Homes, Inc., from traversing Mangyan Road, and even constructed concrete
posts that prevented the residents of LOYOLA from passing through.
Solid Homes, Inc., filed a case before the Regional Trial Court and prayed that LA
VISTA been joined from preventing and obstructing the use and passage of LOYOLA
residents through Mangyan Road. The lower court recognized the easement of
right-of-way along Mangyan Road in favor of Solid Homes, Inc., and ordered LA
VISTA to pay damages. On appeal by LA VISTA, the decision of the lower court was
affirmed.
ISSUE: Whether or not there is an easement of right-of-way over Mangyan Road.
HELD: Yes. The predecessors-in-interest of both LA VISTA and Solid Homes, Inc.,
i.e., the Tuasons and the Philippine Building Corporation, respectively, clearly
established a contractual easement of right-of-way over Mangyan Road. A
voluntary easement is quite evidently manifested in the stipulation in the Deed of
Sale with mortgage executed by them. When the easement was established by
their contract, the parties unequivocally made provisions for its observance by all
whom in the future might succeed them in dominion. It is thus very apparent that
the parties and their respective predecessors-in-interest intended to establish an
easement of right-of-way over Mangyan Road for their mutual benefit, both as
dominant and servient estates.
With this, the free ingress and egress along Mangyan Road created by the
voluntary agreement between Ateneo and Solid Homes, Inc., is thus legally
demandable (Articles 619 and 625, New Civil Code) with the corresponding duty on
the servient estate not to obstruct the same.
LA VISTA contends that there are other routes to LOYOLA from Mangyan Road,
however, this should not be taken into consideration since the opening of an
adequate outlet to a highway can extinguish only legal or compulsory easements,
not voluntary easements like in the case at bar. The fact that an easement by
grant may have also qualified as an easement of necessity does not detract from
its permanency as a property right, which survives the termination of the necessity.
Alcantara v. Reta, Jr.
G.R. No. 136996, December 14, 2001, 372 SCRA 364
Pardo, J.
FACTS: Alcantara and the other petitioners claim that they were tenants or lessees
of the land owned by Reta. The land has been converted into a commercial center
and Reta is threatening to eject them. They claim that since they are legitimate
tenants or lessees of such land, they have the right of first refusal to purchase the
land in accordance with Section 3(g) of Presidential Decree No. 1517, the Urban
Land Reform Act. They also claimed that the amicable settlement executed
between Reta and Ricardo Roble, one of the petitioners, was void ab initio for being
violative of PD No. 1517. On the other hand, Reta claimed that the land is question
is not within the scope of PD No. 1517 since it was not proclaimed as an Urban
Land Reform Zone (ULRZ). Alcantara, among others, then filed complaint for the
exercise of the right of first refusal under PD No. 1517 in the Regional Trial Court.

100
However, such complaint was dismissed and such dismissal was affirmed by the
Court of Appeals. Hence, this petition was filed.
ISSUE: Whether the Alcantara and the other petitioners have the right of first
refusal.
HELD: No. The land involved has not been proclaimed an Urban Land Reform Zone
(ULRZ). In fact, petitioners filed a petition with the National Housing Authority
requesting that said land be declared as an ULRZ. Clearly, the request to have the
land proclaimed as an ULRZ would not be necessary if the property was an ULRZ.
PD No. 1517 pertains to areas proclaimed as ULRZ. Consequently, petitioners
cannot claim any right under the said law since the land involved is not an ULRZ.
To be able to qualify and avail of the rights and privileges granted by the said
decree, one must be: (1) a legitimate tenant of the land for ten (10) years or more;
(2) must have built his home on the land by contract; and, (3) has resided
continuously for the last ten (10) years. Those who do not fall within the said
category cannot be considered "legitimate tenants" and, therefore, not entitled to
the right of first refusal to purchase the property should the owner of the land
decide to sell the same at a reasonable price within a reasonable time.
Reta denies that he has lease agreements with Alcantara and Roble. Alcantara, on
the other hand, failed to present evidence of a lease agreement other than his
testimony in court. Reta allowed Roble to use sixty-two (62) coconut trees for P186
from where he gathered tuba. This arrangement would show that it is a usufruct
and not a lease. Roble was also allowed to construct his house on the land
because it would facilitate his gathering of tuba. This would be in the nature of a
personal easement under Article 614 of the Civil Code. Whether the amicable
settlement is valid or not, the conclusion would still be the same since the
agreement was one of usufruct and not of lease. Thus, Roble is not a legitimate
tenant as defined by PD No. 1517.
With regard to the other petitioners, Reta admitted that he had verbal agreements
with them. This notwithstanding, they are still not the legitimate tenants who can
exercise the right of first refusal under PD No. 1517. From the moment Reta
demanded that the petitioners vacate the premises, the verbal lease agreements,
which were on a monthly basis since rentals were paid monthly, ceased to exist as
there was termination of the lease.
In conclusion, none of the petitioners is qualified to exercise the right of first
refusal under PD No. 1517.
There was also no intention on the part of Reta to sell the property. Hence, even if
the petitioners had the right of first refusal, the situation which would allow the
exercise of that right, that is, the sale or intended sale of the land has not
happened. PD No. 1517 applies where the owner of the property intends to sell it
to a third party.
Republic of the Philippines v. Silim
G.R. No. 140487, April 2, 2001, 356 SCRA 1
Kapunan, J.
FACTS: Respondent Spouses Silim and Mangubat donated a 5,600 square meter
parcel of land in favor of the Bureau of Public Schools of the Municipality of
Malangas, Zamboanga del Sur. In the Deed of Donation, the respondents imposed
the condition that the said property should be used exclusively and forever for

101
school purposes only. This donation was accepted by the District Supervisor of the
Bureau, through an Affidavit of Acceptance and/or Confirmation of Donation.
A school building was thereafter constructed on the donated land. However,
another school building that was also supposed to be allocated for the donated
parcel of land could not be released since the government required that it be built
upon a 1 hectare parcel of land. By reason of this, the District Supervisor and the
vice-mayors wife entered into a Deed of Exchange whereby the donated lot was
exchanged with a bigger lot owned by the latter. Consequently, the school
buildings were constructed on this new school site and the school building
previously erected on the donated land was dismantled and transferred to the new
location. One day, respondents were surprised when he saw the vice-mayor
constructing a house on the donated land.
ISSUES:
1.) Whether or not there was a valid donation despite non-notation of the
acceptance in the Deed of Donation, as required in Article 749.
2.) Whether or not the condition on the donation was violated.
HELD: 1.) Yes. The purpose of the formal requirement for acceptance of a donation
is to ensure that such is duly communicated to the donor. In the case at bar, a
school building was immediately constructed after the donation was executed.
Respondents had knowledge of the existence of the school building put up on the
donated lot. The actual knowledge by respondents of the construction and
existence of the school building fulfilled the legal requirement that the acceptance
of the donation by the donee be communicated to the donor.
2.) No. There was no violation even after the donated lot was exchanged for
another one. The purpose of the donation remains the same, which is for the
establishment of a school. The exclusivity of the purpose was not altered or
afected. In fact, the exchange of the lot for a much bigger one was in furtherance
and enhancement of the purpose of the donation. The acquisition of the bigger lot
paved the way for the release of funds for the construction of Bagong Lipunan
school building which could not be accommodated by the limited area of the
donated lot.
Prosperity Credit Resources, Inc. v. Court of Appeals
G.R. No. 114170, January 15, 1999, 301 SCRA 52
Mendoza, J.
FACTS: Private respondent Metropolitan Fabrics, Inc. (MFI) and petitioner
Prosperity Credit Resources, Inc. (PCRI) executed a Memorandum of Undertaking
(MOU) wherein PCRI acceded to MFIs request to redeem three of the seven lots
foreclosed and won by the former in the ensuing public auction. The MOA was
conditioned upon the agreement that the petitioner shall be given a right of way
on the existing private road which forms part of the area to be redeemed by
private respondents. Later, PCRI filed an injunctive suit against MFI alleging, inter
alia, that the latter, in violation of the terms of the MOU, refused to allow PCRI to
make excavations on one side of the access road for the installation of water. The
trial court granted the petition for the issuance of the writ of preliminary
mandatory injunction. On appeal, the CA set aside the assailed order of the trial
court; hence, this petition for review on certiorari. PCRI contends that it is entitled
to the issuance of the writ of preliminary mandatory injunction as may be gleaned
from the following provision in the MOU: The above cited lot, being an existing
private road, will remain open to ingress and egress for whatever kind of passage
in favor of PROSPERITY FINANCIAL RESOURCES, INC. or its successors=in-interest.

102
ISSUE: Whether or not the RTC committed grave abuse of discretion in issuing a
writ of preliminary mandatory injunction ordering private respondent to allow
petitioner to undertake excavations along the access road for the purpose of
installing water pipes.
Held: Yes. There is no question as to the meaning of the terms ingress and
egress. They give petitioner the right to use the private road as means of entry
into and exit from its property on the northwestern side o f the compound. The
question concerns the meaning of the phrase for whatever kind of passage. The
trial court read this phrase to mean that petitioner had the right to make
excavations on the side of the access road in order to install a network of pipes.
The word passage does not, however; clearly and unmistakably convey a
meaning that includes a right to install water pipes on the access road. The
ordinary meaning of the word, as defined in Websters Dictionary, is that act or
action of passing: movement or transference from one place or point to another.
this legal meaning is not diferent. It means, according to Blacks Law Dictionary,
the act of passing; transit; transition.
Villanueva v. Velasco
G.R. No. 130845, November 27, 2000, 346 SCRA 99
Quisumbing, J.
FACTS: Petitioner Bryan Villanueva is the registered owner of the parcel of land
covered by Transfer Certificate of Title No. 127862 of the Register of Deeds of
Quezon City. He bought it from Pacific Banking Corporation, the mortgagee of said
property. When petitioner bought the parcel of land there was a small house on its
southeastern portion. It occupied one meter of the two-meter wide easement of
right of way the Gabriel spouses granted to the Espinolas, predecessors-in-interest
of private respondents, in a Contract of Easement of Right of Way. Unknown to
petitioner, even before he bought the land, the Gabriels had constructed the
aforementioned small house that encroached upon the two-meter easement.
Petitioner was also unaware that private respondents, Julio Sebastian and Shirley
Lorilla, had filed on May 8, 1991 for easement. As successors-in-interest, Sebastian
and Lorilla wanted to enforce the contract of easement. On August 13, 1991, a writ
of preliminary mandatory injunction was issued, ordering the Gabriels to provide
the right of way and to demolish the small house encroaching on the easement. On
January 5, 1995, Judge Tirso Velasco issued an Alias Writ of Demolition. Meanwhile,
petitioner filed a Third Party Claim with Prayer to Quash Alias Writ of Demolition.
He maintains that the writ of demolition could not apply to his property since he
was not a party to the civil case.

ISSUE: Whether or not the easement on the property binds petitioner.

HELD: Yes. Unlike other types of encumbrance of real property, a servitude like a
right of way can exist even if they are not expressly stated or annotated as an
encumbrance in a Torrens title because servitudes are inseparable from the
estates to which they actively or passively belong. Moreover, Villanueva was bound
by the contract of easement, not only as a voluntary easement but as a legal
easement. A legal easement is mandated by law, and continues to exist unless its
removal is provided for in a title of conveyance or the sign of the easement is

103
removed before the execution of the conveyance conformably with Article 649 in
accordance with Article 617 of the Civil Code.

National Irrigation Administration v. Court of Appeals


G.R. No. 114348, September 20, 2000, 340 SCRA 661
Pardo, J.
FACTS: A free patent over 3 hectares of land in Cagayan was issued and
registered in the name of private respondent Dick Manglapus predecessor-ininterest, Vicente Manglapus. The land was granted to the latter subject to the
provisions of sections 113, 121, 122 and 124 of Commonwealth Act No. 141 which
provide that except in favor of the Government or any of its branches, units, or
institutions, the land hereby acquired shall be inalienable and shall not be subject
to encumbrance for a period of 5 years from the date of this patent and shall not
be liable for the satisfaction of any debt contracted prior to the expiration of that
period. Subsequently, private respondent Manglapus acquired the lot from Vicente
Manglapus by absolute sale and was later registered 11 years later from the
issuance of patent. Meanwhile, petitioner National Irrigation Administration entered
into a contract with Villamar Development Construction. Under the contract,
petitioner NIA was to construct canals in Cagayan. NIA then entered a portion of
petitioners land and made diggings and fillings thereon. Private respondent then
filed a complaint for damages alleging that petitioners diggings and fillings
destroyed the agricultural use of his land and that no reasonable compensation
was paid for its taking.
ISSUE: Whether or not the petitioner NIA should pay Manglapus just compensation
for the taking of a portion of his property for use as easement of a right of way.
HELD: No. We find that NIA is under no obligation. We sustain the appeal. We
agree with NIA that the Transfer Certificate of Title and the Original Certificate of
Title covering the subject parcel of land contained a reservation granting the
government a right of way over the land covered therein.
Under the Original Certificate of Title, there was a reservation and condition that
the land is subject to to all conditions and public easements and servitudes
recognized and prescribed by law, especially thouse mentioned in Sections 109,
110, 111, 112, 113 and 114, Commonwealth Act No. 141, as amended. This
reservation, unlike the other provisos imposed on the grant, was not limited by any
time period and thus is a subsisting condition. Section 112, Commonwealth Act No.
141, provides that lands granted by patent, shall further be subject to a right of
way not exceeding twenty meters in width for public highways, railrods, irrigation,
ditches, aqueducts, telegraphs and telephone lines, and similar works as the
Government or any public or quasi-public service or enterprises, including mining
or forest concessionaires may reasonably require for carrying on their business,
with damages for the improvements only.
Article 619 of the Civil Code provides that Easements are established either by
law or by the will of the owners. The former are called legal and the latter
voluntary easements. In the present case, we find and declare that a legal
easement of a right-of-way exists in favor of the government. The land was
originally public land, and awarded to respondent Manglapus by free patent. The
ruling would be otherwise if the land were originally private property, in which
case, just compensation must be paid for the taking of a part thereof for public use
as an easement of a right of way.

104
Imperial v. Court of Appeals
G.R. No. 112483, October 8, 1999, 316 SCRA 393
Gonzaga Reyes, J.
FACTS: Leoncio Imperial was the owner of a parcel of land with an area of 32,837
sq. m. and located in Albay. On July 7, 1951, Leoncio sold the lot for Php 1.00 to his
acknowledged natural son, petitioner in this case. Petitioner and Victor Imperial,
adopted son of Leoncio, agreed that despite the designation of the contract as
Deed of Absolute Sale, the transaction is in fact a Donation. Two years after,
Leoncio filed a complaint for the Annulment of Donation. It was however resolved
through a compromise agreement under the following terms and conditions: (1)
Leoncio recognized and agreed the legality and validity of the rights of petitioner;
and (2) petitioner agreed to sell a designated 1,000 sq.m. portion of the donated
land.
Leoncio died leaving only two heirs: petitioner and Victor Imperial. On March 8,
1962, Victor was substituted in the complaint for annulment. He moved for the
execution of judgment and it was granted. After 15 years, Victor died and was
survived only by his natural father, Ricardo Villalon. Ricardo Villalon is a lessee of
the portion of the subject property. Villalon died leaving his heirs, Cesar and Teresa
Villalon, respondents in this case. In 1986, respondents filed a complaint for the
annulment of the donation. Allegedly, it impairs the legitime of Victor Imperial.
ISSUES:
1.) Whether or not the respondents have the right to question the inofficious
donation and seek its reduction.
2.) Whether or not the 30-year prescriptive period is applicable in the reduction of
the inofficious donation.
HELD: 1.) Yes. At the time of the substitution, the judgment approving the
compromise agreement has already been rendered. Victor merely participated in
the execution of the compromise judgment. He was not a party to the compromise
agreement. When Victor substituted Leoncio, he was not deemed to have
renounced his legitime. He was therefore not precluded or estopped from
subsequently seeking the reduction. Nor are Victors heirs, upon his death,
precluded from doing so. This is in accordance with Articles 772 and 1053 of the
new Civil Code, to wit:
Article 772. Only those who at the time of the donors death have a right to the
legitime and their heirs and successors in interest may ask for the reduction of the
inofficious donation xxx.
and
Article 1053. If the heir should die without having accepted or repudiated the
inheritance, his rights shall be transmitted to his heirs.
2.) No. Under Article 1144 of the New Civil Code, actions upon an obligation
created by law must be brought within ten years from the right of action accrues.
Thus, the 10-year prescriptive period applies to the obligation to reduce inofficious
donations required under Article 771 of the New Civil Code to the extent that they
impair the legitime of compulsory heirs.
The cause of action to enforce a legitime accrues upon the death of the donordecedent. Clearly so, since only then that the net estate may be ascertained and
on which basis, the legitimes may be determined. It took 24 years since the death

105
of Leoncio to initiate this case. Thus, the action has long prescribed. Not only has
prescription set in, they are also guilty of estoppel and laches. Fifteen years after
the death of Leoncio, Victor died. Ricardo Villalon, Victors sole heir, died four years
later. While Victor was alive, he gave no indication of any interest to contest the
donation of his deceased father.
HIDALGO ENTERPRISES V. BALANDAN
91 PHIL 488
FACTS: Hidalgo Enterprises, an owner of an ice plant factory, kept on their
premises 2 uncovered water tanks, which were unguarded. On April 16, 1948,
children entered the factory premises and swam in one of the water tanks. Mario
Balandan, a boy barely 8 years old, was drowned and sank in the tank. The lower
court decided in favor of the boys parents saying that Hidalgo Enterprises is liable
for damages due to the doctrine of attractive nuisance.
ISSUE: Whether the doctrine of attractive nuisance is applicable in this case?
HELD: A swimming pool or water tank isnt an attractive nuisance for while it is
attractive, it cannot be a nuisance being merely an imitation of a work of nature.
Hence, if small children are drowned in an attractive water tank of another, the
owner is not liable even if there be no guards in the premises.

Acap v. Court of Appeals


G.R. No. 118114, December 7, 1995, 251 SCRA 30
Padilla, J.
FACTS: The title to Lot 1130 of the Cadastral Survey of Hinigaran, Negros
Occidental was evidenced by OCT R-12179. The lot has an area of 13,720 sq. m.
The title was issued and is registered in the name of spouses Santiago Vasquez
and Lorenza Oruma. After both spouses died, their only son Felixberto inherited the
lot. In 1975, Felixberto executed a duly notarized document entitled Declaration
of Heirship and Deed of Absolute Sale in favor of Cosme Pido. Since 1960, Teodoro
Acap had been the tenant of a portion of the said land, covering an area of 9,500
sq. m. When ownership was transferred in 1975 by Felixberto to Cosme Pido, Acap
continued to be the registered tenant thereof and religiously paid his leasehold
rentals to Pido and thereafter, upon Pidos death, to his widow Laurenciana. The
controversy began when Pido died interstate and on 27 November 1981, his
surviving heirs executed a notarized document denominated as Declaration of
Heirship and Waiver of Rights of Lot 1130 Hinigaran Cadastre, wherein they
declared to have adjudicated upon themselves the parcel of land in equal share,
and that they waive, quitclaim all right, interests and participation over the parcel
of land in favor of Edy de los Reyes. The document was signed by all of Pidos
heirs. Edy de los Reyes did not sign said document. It will be noted that at the time
of Cosme Pidos death, title to the property continued to be registered in the name
of the Vasquez spouses. Upon obtaining the Declaration of Heirship with Waiver of
Rights in his favor, de los Reyes filed the same with the Registry of Deeds as part
of a notice of an adverse claim against the original certificate of title. Thereafter,
delos Reyes sought for Acap to personally inform him that he had become the new
owner of the land and that the lease rentals thereon should be paid to him. Delos
Reyes alleged that he and Acap entered into an oral lease agreement wherein Acap
agreed to pay 10 cavans of palay per annum as lease rental. In 1982, Acap
allegedly complied with said obligation. In 1983, however, Acap refused to pay any
further lease rentals on the land, prompting delos Reyes to seek the assistance of

106
the then Ministry of Agrarian Reform (MAR) in Hinigaran, Negros Occidental. The
MAR invited Acap, who sent his wife, to a conference scheduled on 13 October
1983. The wife stated that the she and her husband did not recognize delos
Reyess claim of ownership over the land. On 28 April 1988, after the lapse of four
(4) years, delos Reyes filed a complaint for recovery of possession and damages
against Acap, alleging that as his leasehold tenant, Acap refused and failed to pay
the agreed annual rental of 10 cavans of palay despite repeated demands. On 20
August 1991.
ISSUE: Whether or not the subject declaration of heirship and waiver of rights is a
recognized mode of acquiring ownership by private respondent over the lot in
question.
HELD: An asserted right or claim to ownership or a real right over a thing arising
from a juridical act, however justified, is not per se sufficient to give rise to
ownership over the res. That right or title must be completed by fulfilling certain
conditions imposed by law. Hence, ownership and real rights are acquired only
pursuant to a legal mode or process. While title is the juridical justification, mode is
the actual process of acquisition or transfer of ownership over a thing in question.
Under Article 712 of the Civil Code, the modes of acquiring ownership are generally
classified into two (2) classes, namely, the original mode (i.e., through occupation,
acquisitive prescription, law or intellectual creation) and the derivative mode (i.e.,
through succession mortis causa or tradition as a result of certain contracts, such
as sale, barter, donation, assignment or mutuum).
In the case at bench, the trial court was obviously confused as to the nature and
efect of the Declaration of Heirship and Waiver of Rights, equating the same with
a contract (deed) of sale. They are not the same. In a Contract of Sale, one of the
contracting parties obligates himself to transfer the ownership of and to deliver a
determinate thing, and the other party to pay a price certain in money or its
equivalent. Upon the other hand, a declaration of heirship and waiver of rights
operates as a public instrument when filed with the Registry of Deeds whereby the
intestate heirs adjudicate and divide the estate left by the decedent among
themselves as they see fit. It is in efect an extrajudicial settlement between the
heirs under Rule 74 of the Rules of Court.
Hence, there is a marked diference between a sale of hereditary rights and a
waiver of hereditary rights. The first presumes the existence of a contract or deed
of sale between the parties. The second is, technically speaking, a mode of
extinction of ownership where there is an abdication or intentional relinquishment
of a known right with knowledge of its existence and intention to relinquish it, in
favor of other persons who are co-heirs in the succession. Private respondent,
being then a stranger to the succession of Cosme Pido, cannot conclusively claim
ownership over the subject lot on the sole basis of the waiver document which
neither recites the elements of either a sale, or a donation, or any other derivative
mode of acquiring ownership.
De Luna v. Abrigo
G.R. No. L-57455, January 18, 1990, 181 SCRA 150
Medialdea, J.
FACTS: Prudencio de Luna donated a portion of a lot of the Cadastral Survey of
Lucena to the Luzonian University Foundation. The donation was embodied in a
Deed of Donation Intervivos and made subject to certain terms and conditions and
provided for the automatic reversion to the donor of the donated property in case

107
of violation or non-compliance. The foundation failed to comply with the conditions
of the donation. De Luna "revived" the said donation in favor of the foundation, in a
document entitled "Revival of Donation Intervivos" subject to terms and conditions
which among others, required it to construct a chapel, a nursery and a
kindergarten school in the donated property within five (5) years from execution.
The automatic reversion to the donor of the donated area in case of violation of the
conditions was also provided. The foundation, through its president, accepted the
donation. A "Deed of Segregation" was later executed by De Luna and the
foundation whereby the area donated was adjudicated to the foundation. The heirs
of de Luna later filed a complaint with the trial court alleging that the terms and
conditions of the donation were not complied with by the foundation. Thus, it
prayed for the cancellation of the donation and the reversion of the donated land
to the heirs. The foundation invoked, among others, the defense of prescription of
action. The court dismissed the complaint. It ruled that under Article 764 of the
New Civil Code, actions to revoke a donation on the ground of non-compliance with
any of the conditions of the donation shall prescribe in four years (4) counted from
such non-compliance. In the instant case, the four-year period for filing the
complaint for revocation commenced on April 9, 1976 and expired on April 9, 1980.
Since the complaint was brought on September 23, 1980 or more than five (5)
months beyond the prescriptive period, it was already barred by prescription.
ISSUE: Whether or not the complaint is one for judicial decree of revocation of the
donation in question as contemplated in Article 764 of the New Civil Code and
which prescribes in four (4) years and not an action to enforce a written contract
which prescribes in ten (10) years.
HELD: The donation subject of this case is one with an onerous cause. It was made
subject to the burden requiring the donee to construct a chapel, a nursery and a
kindergarten school in the donated property within five years from execution of the
deed of donation. It is true that under Article 764, actions for the revocation of a
donation must be brought within for (4) years from the non-compliance of the
conditions of the donation. However, the said article does not apply to onerous
donations in view of the specific provision of Article 733 providing that onerous
donations are governed by the rules on contracts. Therefore, the rules on contracts
and the general rules on prescription and not the rules on donations are applicable
in the case at bar.
Furthermore, while the judicial action for the rescission of a contract is generally
not necessary where the contract provides that it may be automatically revoked
and cancelled for violation of any of its terms and conditions, however, where one
of the parties contests or denies the rescission, judicial intervention is necessary
not for purposes of obtaining a judicial declaration rescinding a contract already
deemed rescinded by virtue of an agreement providing for rescission even without
judicial intervention, but in order to determine whether or not the rescission was
proper. Judicial action will be necessary as without it, the extrajudicial resolution
will remain contestable and subject to judicial invalidation, unless attack thereon
should become barred by acquiescence, estoppel or prescription.
In the instant case, trial court was therefore not correct in holding that the
complaint is barred by prescription under Article 764 because Article 764 does not
apply to onerous donations. As provided in the donation executed on April 9, 1971,
compliance with the terms and conditions of the contract of donation, shall be
made within five (5) years from its execution. The complaint which was filed on
September 23, 1980 was then well within the ten (10) year prescriptive period to
enforce a written contract pursuant to Article 1144 par. 1, counted from April 9,
1976.

108
Gonzales v. Court of Appeals
G.R. No. 110335, June 18, 2001, 358 SCRA 598
Melo, J.
FACTS: Deceased spouses Ignacio and Marina Gonzales were registered owners of
two parcels of agricultural land. Petitioners are the successors-in-interest or the
children and grandchildren of the deceased spouses. On the other hand, private
respondents are the farmers and tenants of said spouses who have been
cultivating the parcels of land even before World War II either personally or through
their predecessors-in-interest. Marina Gonzales died intestate and appointed as
administratix was petitioner Lilia Gonzales. Prior to the partition of said estate,
Ignacio Gonzales executed a Deed of Donation in favor of his grandchildren but
was not registered. When Presidential Decree No. 27 took efect, the landholdings
of the said spouses were placed under Operation Land Transfer. Private
respondents were then issued the corresponding Certificates of Land Transfer. The
administratix of the spouses estate, Lilia Gonzales filed an application for retention
requesting that their property be excluded from the Operation Land Transfer.
Initially, it was denied but was approved due to the deed of donation.
ISSUE: Whether or not the property subject of the deed of donation which was not
registered when P.D. No. 27 took efect, should be excluded from the Operation
Land Transfer.
HELD: No. Article 749 of the Civil Code provides inter alia that in order that the
donation of an immovable may be valid, it must be made in a public document,
specifying therein the property donated and the value of the charges which the
done must satisfy. Corollarily, Article 709 of the same Code explicitly states that
the titles of ownership, or other rights over immovable property, which are not
duly inscribed or annotated in the Registry of Property shall not prejudice third
persons. From the foregoing provisions, it may be inferred that as between the
parties to a donation of immovable property, all that is required is for said donation
to be contained in a public document. Registration is not necessary for it to be
contained in a public document. It is not necessary for it to be considered valid and
efective. However, in order to bind third persons, the donation must be registered
in the Registry of Property. In the case at bar, the donation executed by Ignacio
Gonzales in favor of his grand children, although in writing and duly notarized, has
not been registered in accordance with law. For this reason, it shall not be binding
upon private respondents who did not participate in said deed nor had no actual
knowledge thereof.
Reyes v. Mosqueda
G.R. No. L-45262, July 23, 1990
Gutierrez, Jr., J.
FACTS: On May 15, 1969, Dr. Emilio Pascual executed a Deed of Donation of real
property located at 1109-1111 R. Papa St. Tondo, Manila in favor of Ofelia
Parungao, a minor, with her mother, Rosario Duncil, accepting the gift and
donation for and in her behalf. However, Ursula Pascual alleged that Dr. Pascual
during his lifetime on November 2, 1966 executed a Donation mortis causa in her
favor covering the said property. Parungao, upon reaching the age of majority was
able to register the Deed of Donation with the Register of Deeds in Manila and was
issued a TCT.
On September 23, 1976, Ursula executed a deed of absolute sale over the Tondo
property in favor of Benjamin, Oscar, Jose, and Emmanuel Reyes. Benjamin filed a

109
complaint for the declaration of nullity of the TCT of Parungao and/or reconveyance
of the deed of title. The CFI of Manila declared the TCT in the name of Parungao
null and void and ordered the Register of Deeds to cancel the title. On appeal, the
Court of Appeals ruled that the 1966 donation to Ursula was inter vivos, which
meant that the property was already transferred to Ursula at that time.
ISSUE: Whether or not the donation to Ursula was Inter Vivos or Mortis Causa.
HELD: It was a Donation Inter Vivos. The title given by the donor in the deed of
donation is not a determinative factor which makes the donation inter vivos or
mortis causa. It is the body of the document of donation and the statements
contained therein and not the title that should be considered in ascertaining the
intent of the donor. In the case, the donor used the term donation Mortis Causa but
from the stipulations of the deed, it can be clearly inferred that he was actually
executing a donation Inter Vivos to Ursula.
The transfer of ownership over the properties donated to Ursula was immediate
and independent of the death of Dr. Pascual since it was a donation Inter Vivos.
The provision as regards the reservation of properties for the donor's subsistence
in relation to the other provisions of the deed of donation confirms the intention of
the donor to give the naked ownership of the properties to Ursula immediately
after the execution of the deed of donation. Hence, he could not have donated the
property again in 1969 in favor of Parungao since the lot was already transferred to
Ursula at that time.
Liguez v. Court of Appeals
G.R. No. L-11240, December 18, 1957, 102 Phil. 577
Reyes, J.B.L., J.
FACTS: The case began upon complaint filed by petitioner-appellant against the
widow and heirs of the late Salvador P. Lopez to recover a parcel of land in barrio
Davao. Plaintif averred to be its legal owner, pursuant to a deed of donation of
said land, executed in her favor by the late owner, Salvador P. Lopez, on 18 May
1943. The defense interposed was that the donation was null and void for having
an illicit causa or consideration, which was the plaintif's entering into marital
relations with Salvador P. Lopez, a married man; and that the property had been
adjudicated to the appellees as heirs of Lopez by the court of First Instance, since
1949.
It was ascertained by the Court of Appeals that the donated land originally
belonged to the conjugal partnership of Salvador P. Lopez and his wife, Maria Ngo;
that the latter had met and berated Conchita for living maritally with her husband,
sometime during June of 1943; that the widow and children of Lopez were in
possession of the land and made improvements thereon; that the land was
assessed in the tax rolls first in the name of Lopez and later in that of his widow.;
and that the deed of donation was never recorded.
Upon these facts, the Court of Appeals held that the deed of donation was
inoperative, and null and void (1) because the husband, Lopez, had no right to
donate conjugal property to the plaintif appellant; and (2) because the donation
was tainted with illegal cause or consideration, of which donor and donee were
participants.
ISSUE: Whether or not the donation is valid.

110
HELD: In the present case, it is scarcely disputable that Lopez would not have
conveyed the property in question had he known that appellant would refuse to
cohabit with him. The cohabitation was an implied condition to the donation, and
being unlawful, necessarily tainted the donation itself.
The rule that parties to an illegal contract, if equally guilty, will not be aided by the
law but will both be left where it finds them, has been interpreted by this Court as
barring the party from pleading the illegality of the bargain either as a cause of
action or as a defense. Memo auditor propriam turpitudinem allegans.
The appellant seeks recovery of the disputed land on the strength of a donation
regular on its face. To defeat its efect, the appellees must plead and prove that
the same is illegal. But such plea on the part of the Lopez heirs is not receivable,
since Lopez, himself, if living, would be barred from setting up that plea; and his
heirs, as his privies and successors in interest, can have no better rights than
Lopez himself.
Appellees, as successors of the late donor, being thus precluded from pleading the
defense of immorality or illegal causa of the donation, the total or partial
inefectiveness of the same must be decided by diferent legal principles. In this
regard, the Court of Appeals correctly held that Lopez could not donate the entirety
of the property in litigation, to the prejudice of his wife Maria Ngo, because said
property was conjugal in character and the right of the husband to donate
community property is strictly limited by law
Pershing Tan Queto v. Court of Appeals
G.R. No. L-35648, March 27, 1987, 148 SCRA 54
Paras, J.
FACTS: Herein private respondent Restituta Tacalinar Guangco de Pombuena
received the questioned lot from her mother Basilides Tacalinar either as a
purported donation or by way of purchase with P50 as the alleged consideration
thereof. The donation or sale was consummated while Restituta was already
married to her husband Juan Pombuena. Juan then filed for himself and his
supposed co-owner Resitituta an application for a Torrens Title over the land which
was later on granted pronouncing him (married to Resitiuta) as the owner of the
land.
A contract of lease over the lot was entered into between petitioner, Pershing Tan
Queto and Restituta with the consent of her husband for a period of 10 years. The
lease of contract having expired, Restituta filed for unlawful detainer against Tan
Queto. The unlawful detainer case was won by the spouses in the Municipal Court
but on appeal in the CFI the entire case was dismissed because of a barter
agreement whereby Tan Queto became the owner of the disputed lot and the
spouses became the owners of a parcel of land with the house thereon previously
owned before the barter by Tan Queto. After the barter agreement, Tan Queto
constructed on the disputed land a concrete building without any objection from
Restituta. Afterwards Restituta sued both Juan and Tan Queto for reconveyance of
the title over the registered but disputed lot, for annulment of the barter, and for
recovery of the land with damages.
The respondent courts decision which later on was affirmed by the Supreme court
led to the reformation of the Contract of Sale of the disputed lot from Basilides to
Restituta from a sale to a conveyance of the share of Restituta in the future
hereditary estate of her parents. Hence, this petition for a motion for
reconsideration.

111
ISSUE: Whether or not the conveyance of the share of Restituta in the future
hereditary estate of her parents was valid hence a paraphernal property.
HELD: No. The court ruled that the land is conjugal, not paraphernal. The oral
donation of the lot cannot be a valid donation intervivos because it was not
executed in a public instrument (Art. 749, Civil Code), nor as a valid donation
mortis causa for the formalities of a will were not complied with. The allegation
that the transfer was a conveyance to RESTITUTA of her hereditary share in the
estate of her mother (or parents) cannot be sustained for the contractual
transmission of future inheritance is generally prohibited.
The fact is ownership was acquired by both JUAN and RESTITUTA by tradition
(delivery) as a consequence of the contract of sale (See Art. 712, Civil Code) with
P50.00 (then a considerable amount) as the cause or consideration of the
transaction. The lot is therefore conjugal, having been acquired by the spouses
thru onerous title (the money used being presumably conjugal there being no proof
that RESTITUTA had paraphernal funds of her own).
Pajarillo vs. Intermediate Appellate Court
G.R. No. 72908, August 11, 1989, 176 SCRA 340
Cruz, J.
FACTS: Perfecta Balane de Cordero died intestate in 1945 and leaving a tract of 28
hectares of land with buildings and improvements in the Quezon Province. On May
20, 1946, perfectas siblings Juana and Felipe executed a public instrument entitled
Extra-judicial settlement of the estate of the decease Perfecta Balane de
Cordero. In it they disposed that in according to Perfectas wishes and in
consideration of love and afection, the said property be donated to private
respondent Salud Suterio de Matias, Perfectas niece, who will assume the
encumbrance/obligation to the Philippine National Bank in the amount of P 1,000.
In the same document, the done accepted the donation in a public instrument. The
instrument was never registered nor the title transferred to Saluds name although
she immediately took possession of the land. Sometime in 1951, Salud transferred
the possession of the land to her mother Juana, who was then staying with her
brother Claudio and his family. During the period they were occupying the land,
Claudio paid realty taxes thereon. On May 25, 1956, Juana executed a deed of
absolute sale conveying the land to Claudio. Two years later, Claudio had the land
registered in his name. Claudio died in 1961 and his mother in 1963. On June 30,
1965, the private respondents Salud and Pedro Matias filed a complaint for the
reconveyance of the property on the ground that the deed of sale in favour of
Claudio was fictitious and the registration in his name was null and void. Salud
claimed that no compensation was paid by Claudio and that the transaction was
deliberately concealed from her by her brother and the defendants.
ISSUE: Whether or not the extra-judicial settlement was a donation.
HELD: Yes. Felipe and Juana had declared themselves the heirs of Perfecta and the
owners of the property in question. As such, they were free to give the land to
whomever they pleased and for whatever reason they saw fit. Hence, if they
choose to respect Perfectas wishes and carry out her intentions by donating the
land to Salud, there was no legal impediment to their doing so. There is no
question that Felipe and Juana could have simply disregarded their sisters
sentiments and decided not to donate the property to Salud. The fact that they did
no do this speaks well of their integrity and their loyalty to their deceased sister.

112
The extra-judicial settlement also reflects their own afection for Salud which
constituted the valid consideration for their own act of liberality.
Cruz v. Court of Appeals
G.R. No. L-58671, November 22, 1985, 140 SCRA 245
Plana, J.
FACTS: In 1973, Eduvigis Cruz, a childless widow, donated a 235.5 sq. m.
residential lot in San Isidro, Taytay, Rizal together with the two-door apartment
erected thereon to her grandnieces (private respondents Teresita, Lydia and
Cecilia, all surnamed De Leon). The property was accordingly transferred to the
names of private respondents.
In 1974, Cruz judicially adopted Cresencia Ocreto, a minor, after which she
extrajudicailly tried to revoke the donation, but the donee resisted, alleging that:
(1) the property in question was co-owned by Eduvigis Cruz and her brother, the
late Maximo Cruz, grandfather of the donees, hence the latter own 1/2 of the
property by inheritance; and (2) Eduvigis owns another property, an agricultural
land of more than two hectares situated in Barrio Dolores, Taytay, Rizal, hence the
donation did not impair the presumptive legitime of the adoptive child.
Petitioner filed a complaint against the donees for revocation of donation, invoking
Article 760, par. 3 of the NCC. The trial court rendered a decision revoking the
donation. On appal, The Court of Appeals reversed the trial court and dismissed
the complaint.
ISSUE: Whether or not the Court of Appeals correctly dismissed the complaint to
annul the subject donation.
HELD: Yes. In the case of the subsequent adoption of a minor by one who had
previously donated some or all of his properties to another, the donor may sue for
the annulment or reduction of the donation within 4 years from the date of
adoption, if the donation impairs the legitime of the adopted, taking into account
the whole estate of the donor at the time of the donation of the child (Articles 760,
761 and 763 of the NCC). Of course, the burden of proof is on the plaintif-donor,
who must allege and establish the requirements prescribed by law, on the basis of
which annulment or reduction of the donation can be adjudged. Unfortunately, in
the case at bar, the complaint for annulment does not allege that the subject
donation impairs the legitime of the adopted child. Indeed, it contains no indication
at all of the total assets of the donor.
Nor is there proof of impairment of legitime. On the contrary, there is unrebutted
evidence that the donor has another piece of land worth P273,420 in 1977. The
legal situation of petitioner-donor is made worse by the factual finding of the Court
of Appeals that the grandfather of the donees was the owner pro indiviso of onehalf of the donated land, the efect of which is to reduce the value of the donation
which can then more easily be taken from the portion of the estate within the free
disposal of petitioner.
Roman Catholic Archbishop of Manila v. Court of Appeals
G.R. No. 77425, June 19, 1991, 198 SCRA 300
Regalado, J.
FACTS: On August 23, 1930, the spouses Eusebio de Castro and Martina Rieta
executed a deed of donation in favor of herein petitioner Roman Catholic

113
Archbishop of Manila covering a parcel of land located at Cavite. The deed of
donation provides that the donee shall not dispose or sell the property within a
period of 100 years from the execution of the deed of donation, otherwise a
violation of such condition would render ipso facto null and void the donation and
the property would revert to the estate of the donors.
However, on June 30, 1980 while within the prohibitive period to dispose, petitioner
executed a deed of absolute sale of the property subject of the donation in favor of
the petitioner-spouses Florencio and Soledad Ignao in consideration of the sum of
P114,000.00. Hence, private respondents filed a complaint for the nullification of
the deed of donation. In their answer, the petitioners filed a motion to dismiss
based on the grounds that the action has been barred by prescription because the
complaint was filed four years after the sale, and that the complaint states no
cause of action.
ISSUE: Whether or not the deed of donation in favor of the Roman Catholic
Archbishop of Manila may be revoked.
HELD: No. The complaint in the case at bar cannot be barred by prescription
because the applicable prescriptive period is not the 4-year period provided in
Article 764 of the New Civil Code, rather it is the 10-year period ordinary
prescription shall apply because the deed of donation provides for the automatic
reversion of the property to the original owner in case of violation of any condition.
The Court in the previous case of De Luna v. Abrigo has already settled such
prescriptive period.
However, although the action cannot be dismissed on the ground of prescription,
the same should be dismissed for lack of cause of action.
The cause of action of the private respondents is based on the fact that the
petitioner sold the lot during the 50 th year of the prohibitive period of 100 years.
Such prohibitive period imposed by the respondents was unreasonable because
applying in analogy Articles 494 and 870 of the New Civil Code, the donor cannot
order a prohibitive period of disposition exceeding 20 years. As such, the said
condition regarding the prohibitive period being contrary to law shall be considered
as null and void pursuant to Art. 727 of the New Civil Code but the donation shall
remain valid and subsisting. Thus, respondents cannot anymore revoke the
donation, and the sale of the property by the petitioner to the Ignao spouses shall
be valid and with legal efects.
Eduarte v. Court of Appeals
G.R. No. 105944, February 9, 1996, 253 SCRA 391
Francisco, J.
FACTS: Pedro Calapine was the registered owner of a parcel of land with an area of
12,199 square meters. He executed a deed entitled Donation InterVivos ceding
one-half portion thereof to his niece Helen S. Doria. Eventually, the whole parcel of
land was ceded to Doria by Calapine. Doria then donated a portion of 157 square
meters to the Calauan Christian Reformed Church. He also sold, transferred and
conveyed unto the spouses Eduarte the parcel of land, saving the 700 square
meters on which Dorias house was erected. However, Pedro Calapine filed a
complaint against Doria, the Calauan Christian Reformed Church, Inc. and the
spouses Eduarte claiming that his signature to the deed of donation was a forgery.
He prays for the revocation of the donation made in favour of Doria, to declare null
and void the deeds of donation and sale that she had executed in favor of the
Calauan Christian Reformed Church, Inc. and the spouses Eduarte.

114
ISSUE: Whether or not the petitioners are buyers in bad faith of the donated
property.
HELD: No. The rule is well-settled that mere possession cannot defeat the title of a
holder of a registered torrens title to real property. When herein petitioners
purchased the subject property from Helen Doria, the same was already covered
by TCT No. T-23205 under the latter's name. And although Helen Doria's title was
fraudulently secured, such fact cannot prejudice the rights of herein petitioners
absent any showing that they had any knowledge or participation in such
irregularity. Thus, they cannot be obliged to look beyond the certificate of title
which appeared to be valid on its fade and sans any annotation or notice of private
respondents' adverse claim. Contrary therefore to the conclusion of respondent
Court, petitioners are purchasers in good faith and for value as they bought the
disputed property without notice that some other person has a right or interest in
such property, and paid a full price for the same at the time of the purchase or
before they had notice of the claim or interest of some other person in the
property. And having established beyond doubt that Helen Doria fraudulently
secured her title over the disputed property which she subsequently sold to
petitioners, Helen Doria should instead be adjudged liable to private respondents,
and not to petitioners as declared by the trial court and respondent Court of
Appeals, for the resulting damages to the true owner and original plaintif, Pedro
Calapine.
Petition granted.
Quilala v. Alcantara
G.R. No. 132681, December 3, 2001, 371 SCRA 311
Ynares Santiago, J.
FACTS: On February 20, 1981, Catalina Quilala executed a "Donation of Real
Property Inter Vivos" in favor of Violeta Quilala over a parcel of land. The "Donation
of Real Property Inter Vivos" consists of two pages. The first page contains the
deed of donation itself, and is signed on the bottom portion by Catalina Quilala as
donor, Violeta Quilala as donee, and two instrumental witnesses. The second page
contains the Acknowledgment, which states merely that Catalina Quilala personally
appeared before the notary public and acknowledged that the donation was her
free and voluntary act and deed. There appear on the left-hand margin of the
second page the signatures of Catalina Quilala and one of the witnesses, and on
the right-hand margin the signatures of Violeta Quilala and the other witness The
deed of donation was registered with the Register of Deeds and, in due course, TCT
No. 17214 was cancelled and TCT No. 143015 was issued in the name of Violeta
Quilala.
On November 7, 1983, Catalina Quilala died. Violeta Quilala likewise died on May
22, 1984. Petitioner Ricky Quilala alleges that he is the surviving son of Violeta
Quilala. Meanwhile, respondents Gliceria Alcantara, Leonora Alcantara, Ines Reyes
and Juan Reyes, claiming to be Catalina's only surviving relatives within the fourth
civil degree of consanguinity, executed a deed of extrajudicial settlement of estate,
dividing and adjudicating unto themselves the above-described property.
On September 13, 1984, respondents instituted against petitioner and Guillermo T.
San Pedro, the Registrar of Deeds of Manila, an action for the declaration of nullity
of the donation inter vivos. The trial court found that the deed of donation,
although signed by both Catalina and Violeta, was acknowledged before a notary
public only by the donor, Catalina. Consequently, there was no acceptance by
Violeta of the donation in a public instrument, thus rendering the donation null and

115
void. On appeal, the Court of Appeals rendered a decision affirming with
modification the decision of the trial court by dismissing the complaint for lack of
cause of action without prejudice to the filing of probate proceedings of Catalina's
alleged last will and testament.
ISSUE: Whether or not the deed of donation is void for lack of acceptance on the
part of the donee Violeta Quilala.
HELD: No. As stated above, the second page of the deed of donation, on which the
Acknowledgment appears, was signed by the donor and one witness on the lefthand margin, and by the donee and the other witness on the right hand margin.
Surely, the requirement that the contracting parties and their witnesses should
sign on the left-hand margin of the instrument is not absolute. The intendment of
the law merely is to ensure that each and every page of the instrument is
authenticated by the parties. The requirement is designed to avoid the falsification
of the contract after the same has already been duly executed by the parties.
Hence, a contracting party affixes his signature on each page of the instrument to
certify that he is agreeing to everything that is written thereon at the time of
signing.
Simply put, the specification of the location of the signature is merely directory.
The fact that one of the parties signs on the wrong side of the page does not
invalidate the document. The purpose of authenticating the page is served, and
the requirement in the above-quoted provision is deemed substantially complied
with.
In the same vein, the lack of an acknowledgment by the donee before the notary
public does not also render the donation null and void. The instrument should be
treated in its entirety. It cannot be considered a private document in part and a
public document in another part. The fact that it was acknowledged before a
notary public converts the deed of donation in its entirety a public instrument. The
fact that the donee was not mentioned by the notary public in the
acknowledgment is of no moment. To be sure, it is the conveyance that should be
acknowledged as a free and voluntary act. In any event, the donee signed on the
second page, which contains the Acknowledgment only. Her acceptance, which is
explicitly set forth on the first page of the notarized deed of donation, was made in
a public instrument.
Hemedes v. Court of Appeals
G.R. No. 107132, October 8, 1999, 316 SCRA 347
Gonzaga Reyes, J.
FACTS: Jose Hemedes, father of Maxima Hemedes and Enrique D. Hemedes. Jose
Hemedes executed a document entitled "Donation Inter Vivos With Resolutory
Conditions" whereby he conveyed ownership over the subject land, together with
all its improvements, in favor of his third wife, Justa Kausapin, subject to the
following resolutory conditions that upon her death or marriage, the DONEE shall
revert the said property to anyone of Jose Hemedes children.
On September 27, 1960 a "Deed of Conveyance of Unregistered Real Property by
Reversion" conveying to Maxima Hemedes. She had it titled and mortgage it to R &
B Insurance with an annotation of USUFRUCT favor of her stepmother,Justa
Kausapin. Unable to pay the mortgage, R & B Insurance extra-judicially foreclosed
the property. However, Justa Kausapin executed another agreement or Kasunduan
on May 27, 1971 to his stepson, Enrique D. Hemedes. He obtained tax

116
declarations and pay realty taxes from thereon. The Ministry of Agrarian Reform
Office conducted a cadastral survey and indicated Enrique Hemedes as the owner.
Enrique Hemedes sold the property to Dominium Realty Const. Corp.(Dominium), a
sister company of Asia Brewery. Asia Brewery started to introduce some
improvements already when R & B insurance informed them that they are the
owners of the property where these improvements are being built.
ISSUE: Whether or not the kasunduan executed by Justa Kausapin in favor of
Enrique D. Hemedes valid.
HELD: The court dismissed the petition and affirmed the decision of the CA. It
held that Maxima failed to comply with the requirements of Art. 1332 of the civil
code and also failed to repudiate Justa Kausapins allegation that she did not
execute such a deed and she never allowed to use the land as security for the
loan. It was found that the deed of conveyance to Maxima was spurious and it
follows that the original title she had for the property was also null and void so as
the mortgage to R & B Insurance. On the other hand, Kausapin executed an
affidavit to affirm the authenticity of the kasundudan in favor of his stepson,
Enrique Hemedes whom she is dependent from for her financial support.
Siguan v. Lim
G.R. No. 134685, November 19, 1999, 318 SCRA 725
Davide, Jr., C.J.
FACTS: On 2 July 1991, a Deed of Donation conveying parcels of land and
purportedly executed by LIM on 10 August 1989 in favor of her children, Linde,
Ingrid and Neil was registered with the Office of the Register of Deeds of Cebu City.
On 23 June 1993, petitioner filed an accion pauliana against LIM and her children to
rescind the questioned Deed of Donation and to declare as null and void the new
transfer certificates of title issued for the lots covered by the questioned Deed.
Petitioner claimed therein that sometime in July 1991, LIM, through a Deed of
Donation, fraudulently transferred all her real property to her children in bad faith
and in fraud of creditors, including her; that LIM conspired and confederated with
her children in antedating the questioned Deed of Donation, to petitioner's and
other creditors' prejudice; and that LIM, at the time of the fraudulent conveyance,
left no sufficient properties to pay her obligations. The RTC ruled in favor of Siguan
and rescinded the Contract, but was reversed by the CA.
ISSUE: Whether or not the Deed of Donation executed by respondent may be
rescinded for being in fraud of her alleged creditor.
HELD: We resolve these issues in the negative. Art. 1381 of the Civil Code
enumerates the contracts which are rescissible, and among them are "those
contracts undertaken in fraud of creditors when the latter cannot in any other
manner collect the claims due them."
The action to rescind contracts in fraud of creditors is known as accion pauliana.
For this action to prosper, the following requisites must be present: (1) the plaintif
asking for rescission has a credit prior to the alienation, although demandable
later; (2) the debtor has made a subsequent contract conveying a patrimonial
benefit to a third person; (3) the creditor has no other legal remedy to satisfy his
claim; (4) the act being impugned is fraudulent; (5) the third person who received
the property conveyed, if it is by onerous title, has been an accomplice in the
fraud. The general rule is that rescission requires the existence of creditors at the
time of the alleged fraudulent alienation, and this must be proved as one of the

117
bases of the judicial pronouncement setting aside the contract. Without any prior
existing debt, there can neither be injury nor fraud. While it is necessary that the
credit of the plaintif in the accion pauliana must exist prior to the fraudulent
alienation, the date of the judgment enforcing it is immaterial. Even if the
judgment be subsequent to the alienation, it is merely declaratory, with retroactive
efect to the date when the credit was constituted. In the instant case, the alleged
debt of LIM in favor of petitioner was incurred in August 1990, while the deed of
donation was purportedly executed on 10 August 1989. Even assuming arguendo
that petitioner became a creditor of LIM prior to the celebration of the contract of
donation, still her action for rescission would not fare well because the third
requisite was not met. Under Article 1381 of the Civil Code, contracts entered into
in fraud of creditors may be rescinded only when the creditors cannot in any
manner collect the claims due them. It is, therefore, "essential that the party
asking for rescission prove that he has exhausted all other legal means to obtain
satisfaction of his claim. 20 Petitioner neither alleged nor proved that she did so. On
this score, her action for the rescission of the questioned deed is not maintainable
even if the fraud charged actually did exist."

Noceda vs. Court of Appeals


G.R. No. 119730, September 2, 1999, 313 SCRA 504
Gonzaga Reyes, J.
FACTS: Celestino Arbizo died in 1956 leaving behind a parcel of land having an
area of 66,530 square meters. His heirs plaintif Aurora Directo, defendant Rodolfo
Noceda, and Maria Arbizo extrajudicially settled the partition of the land with
Directo getting 11,426 square meters, Noceda got 13,294 square meters, and
Arbizo got 41,810 square meters. Plaintif Directo donated 625 square meters of
her share to defendant Noceda, who is her nephew being the son of her deceased
sister However another extrajudicial settlement-partition was executed. Three
fifths of the said land went to Maria Arbizo while plaintif Directo and defendant
Noceda got only one-fifth each.
Sometime in 1981, Noceda constructed his house on the land donated to him by
Directo. Directo fenced the portion allotted to her in the extrajudicial settlement,
excluding the donated portion, and constructed thereon three huts. But in 1985,
Noceda removed the fence earlier constructed by Directo, occupied the three huts
(3) and fenced the entire land of plaintif Directo without her consent. Directo
demanded from Noceda to vacate her land, but the latter refused. Hence, Directo
filed a complaint for the recovery of possession and ownership and
rescission/annulment of donation, against defendant Noceda
ISSUE: Whether or not the acts of Noceda constitute ingratitude to warrant
revocation of the donation.
HELD: Yes. It was established that petitioner Noceda occupied not only the portion
donated to him by private respondent Aurora Arbizo-Directo but he also fenced the
whole area of Lot C which belongs to private respondent Directo, thus petitioner's
act of occupying the portion pertaining to private respondent Directo without the
latter's knowledge and consent is an act of usurpation which is an ofense against
the property of the donor and considered as an act of ingratitude of a donee
against the donor. The law does not require conviction of the donee; it is enough
that the ofense be proved in the action for revocation.
The action to revoke by reason of ingratitude prescribes within one (1) year to be
counted from the time (a) the donor had knowledge of the fact; (b) provided that it

118
was possible for him to bring the action. It is incumbent upon petitioner to show
proof of the concurrence of these two conditions in order that the one (1) year
period for bringing the action be considered to have already prescribed. No
competent proof was adduced by petitioner to prove his allegation.
Heirs of Cesario Velasquez v. Court of Appeals
G.R. No. 126996, February 15, 2000, 325 SCRA 552
Gonzaga Reyes, J.
FACTS: Spouses Leoncia de Guzman and Cornelio Aquino died intestate sometime
in 1945 and 1947, respectively and were childless, leaving 6 parcels of land
situated in Pangasinan. Leoncia De Guzman was survived by her sisters Anatalia de
Guzman and Tranquilina de Guzman.
Sometime in 1989, the Meneses(heirs of Anatalia de Guzman) filed a complaint for
annulment, partition and damages against the heirs of Cesario Velasquez(son of
Tranquilina de Guzman) for the latters' refusal to partition the properties of the
Spouses Aquino.
The complaint alleged that before Leoncias death, she told that the documents of
donation and partition which she and her husband earlier executed were not
signed by them as it was not their intention to give away all the properties to
Cesario Velasquez because Anatalia de Guzman who is one of her sisters had
several children to support; and that Cesario Velasquez and his mother allegedly
promised to divide the properties equally and to give the plaintifs one-half thereof.
Plaintifs further claim that after the death of Leoncia, defendants forcibly took
possession of all the properties and despite plaintifs' repeated demands for
partition, defendants refused.
Plaintifs prayed for the nullity of the documents covering the properties in
question since they do not bear the genuine signatures of the Aquino spouses, to
order the partition of the properties between plaintifs and defendants in equal
shares and to order the defendants to render an accounting of the produce of the
land from the time defendants forcibly took possession until partition shall have
been efected.
Defendants filed their answer with counterclaim alleging that during the lifetime of
spouses Aquino, they had already disposed of their properties in favor of
petitioners' predecessors-in-interest, Cesario Velasquez and Camila de Guzman,
and petitioners Anastacia and Jose Velasquez in the following manner:
(1) The third and sixth parcels were conveyed to defendants' late parents
Cesario Velasquez and Camila de Guzman, by virtue of a Escritura de
Donation Propter Nuptias dated February 15, 1919;
(2) The second parcel was conveyed to defendants' late parents Cesario
Velasquez and Camila de Guzman by virtue of a deed of conveyance dated
July 14, 1939;
(3) The first parcel was likewise conveyed to defendants Jose Velasquez and
Anastacia Velasquez by virtue of a deed of conveyance (Donation Intervivos)
dated April 10, 1939;
(4) As to the fourth and fifth parcels, the same were owned and possessed by
third parties.
The trial court ruled in favor of the plaintifs, giving credibility to Santiago Meneses
testimony; declaring the Donation Intervivos, the Deed of Sale, the Deed of
Donation, the Deed of Sale to third parties over the 4th and 5th parcels as null and
void insofar as 1/2 of the 6 parcels are concerned legitimately belong to the

119
plaintifs; and ordering the defendants to pay damages. Defendants appealed the
decision to respondent CA which affirmed the same. A motion for reconsideration
was filed by the petitioners but the same was denied.
ISSUE: Whether or not the petitioners have acquired absolute and exclusive
ownership of the properties in question.
HELD: Yes. Private respondent Santiago Meneses failed to prove the nullity of the
Deeds of Conveyance executed by the Aquino spouses in favor of petitioners and
their predecessors-in-interest Cesario Velasquez and Camila de Guzman since he
failed to adduce any evidence to support his claim other than his bare allegations
of its nullity. On the other hand, petitioners were able to show by documentary
evidence that the Aquino spouses during their lifetime disposed of the four parcels
of land subject of the complaint, to wit: (a) Escritura de donation propter nuptias,
(b) Deed of donation inter vivos, (c) Escritura de Compreventa with a P500
consideration: (d) Deed of Conveyance with a consideration of P600 and
confirming in the same Deed the Escritura de donation propter nuptias and
Escritura de compraventa abovementioned. It was reversible error for the court to
overlook the probative value of these notarized documents.
A donation as a mode of acquiring ownership results in an efective transfer of title
over the property from the donor to the donee and the donation is perfected from
the moment the donor knows of the acceptance by the donee. Once a donation is
accepted, the donee becomes the absolute owner of the property donated.
The donation of the first parcel made by the Aquino spouses to petitioners Jose and
Anastacia Velasquez who were then 19 and 10 years old respectively was accepted
through their father Cesario Velasquez, and the acceptance was incorporated in
the body of the same deed of donation and made part of it, and was signed by the
donor and the acceptor. Legally speaking there was delivery and acceptance of the
deed, and the donation existed perfectly and irrevocably. The donation inter vivos
may be revoked only for the reasons provided in Articles 760, 764 and 765 of the
Civil Code.
The donation propter nuptias in favor of Cesario Velasquez and Camila de Guzman
over the third and sixth parcels including a portion of the second parcel became
the properties of the spouses Velasquez since 1919. The deed of donation propter
nuptias can be revoked by the non-performance of the marriage and the other
causes mentioned in Article 86 of the Family Code. The alleged reason for the
repudiation of the deed, i.e., that the Aquino spouses did not intend to give away
all their properties since Anatalia had several children to support is not one of the
grounds for revocation of donation either inter vivos or propter nuptias, although
the donation might be inofficious.
The Escritura compraventa over another portion of the second parcel and the Deed
of conveyance dated July 14, 1939 in favor of Cesario and Camila Velasquez over
the remaining portion of the second parcel is also valid. In fact in the deed of sale,
the Aquino spouses ratified and confirmed the rights and interests of Cesario
Velasquez and Camila de Guzman including the previous deeds of conveyance over
the second parcel in the complaint and such deed of sale became the basis for the
issuance of TCT in the names of Cesario Velasquez and Camila de Guzman. The
best proof of the ownership of the land is the certificate of title and it requires
more than a bare allegation to defeat the face value of TCT which enjoys a legal
presumption of regularity of issuance. Notably, during the lifetime of Cesario
Velasquez, he entered into contracts of mortgage and lease over the property as

120
annotated at the back of the certificate of title which clearly established that he
exercised full ownership and control over the property.
Petitioners were able to establish that these four parcels of land were validly
conveyed to them by the Aquino spouses, hence, they no longer formed part of the
conjugal properties of the spouses at the time of their deaths. As regards the
fourth and fifth parcels, petitioners alleged that these were also conveyed to third
persons and they do not claim any right thereto.
In view of the foregoing, the action of partition cannot be maintained. The
properties sought to be partitioned by private respondents have already been
delivered to petitioners and therefore no longer part of the hereditary estate which
could be partitioned. No co-ownership exists between private respondents and
petitioners.

Você também pode gostar